From 60b48fcfb34abb0e030e0457032a41df53781be7 Mon Sep 17 00:00:00 2001 From: Bbalon Date: Sun, 21 Jun 2020 09:34:42 +0200 Subject: [PATCH] Exam update --- ...ircmes-ELEC1370-exam-2013-Juin-Majeure.tex | 289 ++++++-- ...ircmes-ELEC1370-exam-2013-Juin-Mineure.tex | 351 +++++++++- ...ircmes-ELEC1370-exam-2014-Juin-Majeure.tex | 446 +++++++++++-- ...rcmes-ELEC1370-exam-2014-Septembre-All.tex | 453 ++++++++++--- ...ircmes-ELEC1370-exam-2016-Juin-Majeure.tex | 423 +++++++++--- src/q4/circmes-ELEC1370/exam/2018/2018.mk | 2 + .../exam/2018/Juin/All/Makefile | 2 + .../circmes-ELEC1370-exam-2018-Juin-All.tex | 627 ++++++++++++++++++ .../circmes-ELEC1370/exam/2018/Juin/Juin.mk | 2 + src/q4/circmes-ELEC1370/exam/2020/2020.mk | 2 + .../exam/2020/Juin/All/Makefile | 2 + .../circmes-ELEC1370-exam-2020-Juin-All.tex | 589 ++++++++++++++++ .../circmes-ELEC1370/exam/2020/Juin/Juin.mk | 2 + 13 files changed, 2861 insertions(+), 329 deletions(-) create mode 100644 src/q4/circmes-ELEC1370/exam/2018/2018.mk create mode 100644 src/q4/circmes-ELEC1370/exam/2018/Juin/All/Makefile create mode 100644 src/q4/circmes-ELEC1370/exam/2018/Juin/All/circmes-ELEC1370-exam-2018-Juin-All.tex create mode 100644 src/q4/circmes-ELEC1370/exam/2018/Juin/Juin.mk create mode 100644 src/q4/circmes-ELEC1370/exam/2020/2020.mk create mode 100644 src/q4/circmes-ELEC1370/exam/2020/Juin/All/Makefile create mode 100644 src/q4/circmes-ELEC1370/exam/2020/Juin/All/circmes-ELEC1370-exam-2020-Juin-All.tex create mode 100644 src/q4/circmes-ELEC1370/exam/2020/Juin/Juin.mk diff --git a/src/q4/circmes-ELEC1370/exam/2013/Juin/Majeure/circmes-ELEC1370-exam-2013-Juin-Majeure.tex b/src/q4/circmes-ELEC1370/exam/2013/Juin/Majeure/circmes-ELEC1370-exam-2013-Juin-Majeure.tex index 4574c37ce..bb0645c69 100644 --- a/src/q4/circmes-ELEC1370/exam/2013/Juin/Majeure/circmes-ELEC1370-exam-2013-Juin-Majeure.tex +++ b/src/q4/circmes-ELEC1370/exam/2013/Juin/Majeure/circmes-ELEC1370-exam-2013-Juin-Majeure.tex @@ -1,49 +1,127 @@ \documentclass[fr]{../../../../../../eplexam} \usepackage{../../../../../../eplunits} \usepackage[oldvoltagedirection]{circuitikz} +\usepackage{bodegraph} \usepackage{pgfplots} +\usepackage{amsmath} \usepackage{enumitem} \pgfplotsset{compat=newest} \tikzset{meter/.style={draw,thick,circle,fill=white,minimum size =0.75cm,inner sep=0pt}} \hypertitle{circmes-ELEC1370}{4}{ELEC}{1370}{2013}{Juin}{Majeure} -{Nicolas Verbeek\and Adrien Couplet\and Martin Van Essche\and Guillaume Gilson\and Guillaume Colinet} +{Brieuc Balon} {Claude Oestges, Bruno Dehez and Christophe Craeye} - \section{Question Oestges : phaseurs} -Soit le circuit suivant avec $V_o=5.5\angle\ang{104.1}$ V, +Soit le circuit suivant avec $V_o=5.5\angle 104.1^\circ$ V, \begin{center} \begin{circuitikz} \draw - (0,-2.5) to[american voltage source,l=$6\angle\ang{0}$ V] (0,2.5) + (0,-2.5) to[american voltage source,l=$6\angle 0^\circ$ V] (0,2.5) (2.5,2.5) to[european resistor,l=$Z$] (0,2.5) (2.5,2.5) -- (5,2.5) - (2.5,0) to[L,i=$I_L$,l=j$\SI{2}{\ohm}$,-*] (2.5,2.5) - (2.5,0) to[R,v=$V_o$,l=$\SI{2}{\ohm}$] (2.5,-2.5) - (5,2.5) to[american current source,l=$2\angle\ang{0}$ A] (5,-2.5) + (2.5,0) to[L,i=$I_L$,l=$2j\Omega$,-*] (2.5,2.5) + (2.5,0) to[R,v=$V_o$,l=$2\Omega$] (2.5,-2.5) + (5,2.5) to[american current source,l=$2\angle 0^\circ$ A] (5,-2.5) (0,-2.5) -- (5,-2.5); \end{circuitikz} \end{center} - On demande de calculer \begin{enumerate} \item Le courant $I_L$ dans l'inductance - \item L'impédance $Z$ et les caractéristiques des composants ($R$,$L$,$C$) avec une fréquence de $f=\SI{10}{\kilo\hertz}$. + \item L'impédance $Z$ et les caractéristiques des composants ($R$,$L$,$C$) avec une fréquence de $f=10kHz$ \item La puissance active délivrée par la source de tension - \item Le dipôle équivalent de Thévenin aux bornes de la résistance de $\SI{2}{\ohm}$. - \item Le courant dans une résistance de $\SI{3}{\ohm}$ mise en série avec ce dipôle équivalent. + \item Le dipôle équivalent de Thévenin aux bornes de la résistance de $2\Omega$. + \item Le courant dans une résistance de $3\Omega$ mise en série avec ce dipôle équivalent. \end{enumerate} - \begin{solution} \begin{enumerate} - \item $I_L = 2.75\angle -75.9^\circ$ A - \item $Z = 0.698-4.404j \Rightarrow R=\SI{0.7}{\ohm}$ et $C = \SI{3.614}{\micro\farad}$ - \item $P = \SI{4}{\watt}$ - \item $V_\text{th} = 9.94 \angle 62.41^\circ$ et $Z_\text{th} = Z+2j$ - \item $I = 2.25 \angle 95.45^\circ$ A + \item Tout d'abord il faut remarquer que le sens du courant $I_L$ est dans le même sens que la tension $V_o$ dès lors : + \begin{equation*} + I_L = - \frac{V_o}{2} = 2.75\angle -75.9^\circ [A] + \end{equation*} + \item Pour la suite des calculs nous utiliserons les notations telles qu'illustrées : + \begin{center} + \begin{circuitikz} + \draw + (0,-2.5) to[american voltage source,l=$6\angle 0^\circ$ V, i= $I_s$] (0,2.5) + (0,2.5) to[european resistor,l=$Z$,v=$V_Z$] (2.5,2.5) + (2.5,2.5) -- (5,2.5) + (2.5,0) to[L,i=$I_L$,l=$2j\Omega$,v = $V_L$,-*] (2.5,2.5) + (2.5,0) to[R,v=$V_o$,l=$2\Omega$] (2.5,-2.5) + (5,2.5) to[american current source,l=$2\angle 0^\circ$ A] (5,-2.5) + (0,-2.5) -- (5,-2.5) + (2.5,2.5) node[label ={[font=\footnotesize]above:$A$}]{}; + \end{circuitikz} +\end{center} +Sur base de la relation élémentaire $V=ZI$ nous pouvons écrire : +\begin{equation*} + V_L = 2j *I_L = 5.5\angle 14.1^\circ [V] +\end{equation*} +Nous pouvons effectuer une loi des noeuds sur le noeud A : +\begin{equation*} + I_s = 2\angle 0^\circ - I_L = 2.98\angle 63.49^\circ [A] +\end{equation*} +Nous pouvons effectuer une loi des mailles sur la maille de gauche : +\begin{equation*} + V_Z = 6\angle 0^\circ + V_L - V_o = 13.288\angle -17.493^\circ [V] +\end{equation*} +Sur base de la relation élémentraire $V = ZI$ nous pouvons écrire : +\begin{equation*} + Z = \frac{V_Z}{I_S} = 4.458\angle -80.988 ^\circ = 0.698 - 4.404j [\Omega] \Rightarrow R=0.7[\Omega]~\mbox{et}~ C = \frac{1}{4.404*2\pi*f} = 3.614 [\mu F] +\end{equation*} +\item La puissance\footnote{Nous travaillons avec des phaseurs, ils représentent la valeur éfficace de la tension ou du courant. S'il n'est pas spécifié dans l'énoncé si la valeur de la tension $V_o$ représente la valeur de crête ou la valeur éfficace de la tension, n'hésitez pas à le spécifier dans votre exercice. Nous considérons que $V_o$ correspond à la valeur éfficace dans la suite des calculs. } active délivrée par la source est : +\begin{equation*} + P = \Re(S) = \Re(V*I_S^\ast) = \Re(6\angle 0^\circ * 2.98\angle 63.49^\circ) = 7.98[W] +\end{equation*} +\item Pour calculer le dipôle équivalent, nous devons calculer 2 des 3 données suivantes : $V_{Th}$, $I_N$, $Z_{eq}$. Nous calculerons $V_{Th}$ et $Z_{eq}$ par souci de simplicité. Pour calculer $V_{Th}$, il faut retirer la résistance de $2\Omega$. Nous obtenons donc le circuit +\begin{center} + \begin{circuitikz} + \draw + (0,-2.5) to[american voltage source,l=$6\angle 0^\circ$ V, i= $I_s$] (0,2.5) + (0,2.5) to[european resistor,l=$Z$,v=$V_Z$,-*] (2.5,2.5) + (2.5,2.5) -- (5,2.5) + (2.5,2.5) to [european voltages,open,v^=$V_{Th}$] (2.5,-2.5) + (5,2.5) to[american current source,l=$2\angle 0^\circ$ A] (5,-2.5) + (0,-2.5) -- (5,-2.5) + (2.5,2.5) node[label ={[font=\footnotesize]above:$A$}]{}; + \end{circuitikz} +\end{center} +Sur base de la relation élémentaire $V=ZI$ nous pouvons écrire : +\begin{equation*} + V_Z = Z * 2\angle 0^\circ = 4.458\angle -80.988 ^\circ * 2\angle 0^\circ = 8.916 \angle -80.988 ^\circ [V] +\end{equation*} +Nous pouvons aussi écrire de façon élémentaire : +\begin{equation*} + V_{Th} = 6 - V_Z = 9.93\angle 62.4^\circ [V] +\end{equation*} +Pour calculer $Z_{eq}$, il faut retirer la résistance de $2\Omega$. Nous obtenons donc le circuit : +\begin{center} + \begin{circuitikz} + \draw + (0,2.5) to[european resistor,l=$Z$] (2.5,2.5) + (0,-2.5) -- (0,2.5) + (2.5,0) to[L,l=$2j\Omega$,*-] (2.5,2.5) + (0,-2.5) to [short, -*] (2.5,-2.5); + \end{circuitikz} +\end{center} +De façon évidente : +\begin{equation*} + Z_{Th}= Z +2j = 0.698-2.404j[\Omega] +\end{equation*} +\item Nous pouvons représenter le circuit comme tel : +\begin{center} + \begin{circuitikz} + \draw + (0,-2.5) to[american voltage source,l=$V_{Th}$, i= $I$] (0,2.5) to [european resistor,l=$Z_{eq}$](2.5,2.5) + to [R, l=$3\Omega$] (2.5 ,-2.5) --(0,-2.5); + \end{circuitikz} +\end{center} +Sur base de la relation élementaire $V=ZI$ nous obtenons : +\begin{equation*} + I = \frac{V_{Th}}{Z_{eq}+3} = 2.25\angle 95.42^\circ [A] +\end{equation*} \end{enumerate} \end{solution} - \section{Question Dehez : triphasé} Soit le circuit suivant \begin{center} @@ -52,29 +130,29 @@ \section{Question Dehez : triphasé} \coordinate (s2) at ($ (s1) + ({3*cos(60)},{-3*sin(60)}) $); \coordinate (s3) at ($ (s1) + ({-3*cos(60)},{-3*sin(60)}) $); - \draw (s1) to[V,l=$34\angle\ang{0}\,\si{\volt}$,i=$I_1$,*-*] (s2) to [V,l=$34\angle\ang{-120}\,\si{\volt}$,-*] (s3) to [V,l=$34\angle\ang{-240}\,\si{\volt}$] (s1); - \draw (s1) to [R,l=\SI{2}{\ohm}] ++(5,0) coordinate (l11); + \draw (s1) to[V,l=$34\angle 0^\circ \,V$,i=$I_1$,*-*] (s2) to [V,l=$34\angle -120^\circ \,V$,-*] (s3) to [V,l=$34\angle -240^\circ \,V$] (s1); + \draw (s1) to [R,l=$2\Omega$] ++(5,0) coordinate (l11); \draw (s3) to[short] ++(0,-2) coordinate (p1); \coordinate (l12) at ($ (l11) + ({3*cos(60)},{-3*sin(60)}) $); \coordinate (l13) at ($ (l11) + ({-3*cos(60)},{-3*sin(60)}) $); \coordinate (c1) at ($ (l11) + (0,{-sqrt(3)}) $); - \draw (l11) to[L,l_=\SI{j}{\ohm},-*] (c1); - \draw (c1) to[L,l=\SI{j}{\ohm}] (l12); - \draw (c1) to[L,l_=\SI{j}{\ohm}] (l13); + \draw (l11) to[L,l_=$j\Omega$,-*] (c1); + \draw (c1) to[L,l=$j\Omega$] (l12); + \draw (c1) to[L,l_=$j\Omega$] (l13); - \draw (l13) to[R,l=\SI{2}{\ohm}] (s2); - \draw (l12) to[short] ++(0,-2) to[R,l=\SI{2}{\ohm}] (p1); + \draw (l13) to[R,l=$2\Omega$] (s2); + \draw (l12) to[short] ++(0,-2) to[R,l=$2\Omega$] (p1); \coordinate (l21) at ($ (l11) + (3.5,0) $); \coordinate (l22) at ($ (l21) + ({3*cos(60)},{-3*sin(60)}) $); \coordinate (l23) at ($ (l21) + ({-3*cos(60)},{-3*sin(60)}) $); \coordinate (c2) at ($ (l21) + (0,{-sqrt(3)}) $); - \draw (c2) to[L,l_=\SI{8j}{\ohm},*-] (l21); - \draw (c2) to[L,l=\SI{8j}{\ohm}] (l22); - \draw (c2) to[L,l_=\SI{8j}{\ohm}] (l23); + \draw (c2) to[L,l_=$8j\Omega$,*-] (l21); + \draw (c2) to[L,l=$8j\Omega$] (l22); + \draw (c2) to[L,l_=$8j\Omega$] (l23); \coordinate (d1) at ($ (l21) + (3.5,0) $); \coordinate (d2) at ($ (d1) + ({3*cos(60)},{-3*sin(60)}) $); @@ -84,7 +162,7 @@ \section{Question Dehez : triphasé} \draw (l21) to[short] (d1); \draw (l23) to[short] ++(0,-1) to[short] (p2); \draw (l22) to[short] ++(0,-2) to[short] (p3); - \draw (d1) to[C,l=\SI{-3j}{\ohm},i=$I_2$,*-*] (d2) to[C,l=\SI{-3j}{\ohm},-*] (d3) to[C,l=\SI{-3j}{\ohm}] (d1); + \draw (d1) to[C,l=$-3j\Omega$,i=$I_2$,*-*] (d2) to[C,l=$-3j\Omega$,-*] (d3) to[C,l=$-3j\Omega$] (d1); \draw (l11) node[below left]{$\bullet$}; \draw (l21) node[below right]{$\bullet$}; @@ -92,37 +170,72 @@ \section{Question Dehez : triphasé} \draw (l22) node[below left]{$\bullet$}; \draw (l13) node[below right]{$\bullet$}; \draw (l23) node[above]{$\bullet$}; - \draw [<->,>=stealth] ($ (l11) + (-0.2,0.2) $) to [bend left] node[pos=0.5,above] {\SI{6j}{\ohm}} ($ (l21) + (0.2,0.2) $); - + \draw [<->,>=stealth] ($ (l11) + (-0.2,0.2) $) to [bend left] node[pos=0.5,above] {$6j\Omega$} ($ (l21) + (0.2,0.2) $); \end{circuitikz} \end{center} \begin{enumerate} \item Calculer le facteur de dispersion du couplage magnétique; \item Calculer l'amplitude et la phase de $I_1$ et $I_2$. \end{enumerate} - \begin{solution} \begin{enumerate} - \item $$1- \sigma = 1- \frac{M^2}{L_1 L_2} = -3.5$$ + \item Le facteur de dispersion s'écrit comme : + \begin{equation*} + \sigma = 1 -\frac{M^2}{L_1L_2} = 1 - \frac{(6j)^2}{j*j} = -3.5 + \end{equation*} On a ici un facteur de dispersion négatif (impossible physiquement mais erreur de Mr. Dehez. \textit{Errare humanum est}.) - \item On commence par la transformation des sources et des capacités en modèle étoile.\\ Après cette transformation, on a : - \begin{itemize} - \item $V_s = \frac{34}{\sqrt3} \angle\ang{-30}$ [V] - \item $j\omega C = -j$ [$\Omega$] - \end{itemize} - On passe en monophasé, on trouve un courant $I_1'$ dans la partie de gauche égal à $4.267\angle 34.2^\circ$ [A]. On trouve un courant $I_2 = 3.657 \angle34.23^\circ$ [A]. \\ - Connaissant $I_{1}'$ (courant de ligne) on peut trouver le courant $I_{1}$ (courant de phase) via la relation liant les 2 grandeurs. - On trouve $I_1 = 2.464 \angle 64.23^\circ$ [A]. + \item Nous pouvons réécrire le circuit en un circuit simple monophasé en modifiant la source triphasé en triangle en une source triphasée étoile et la charge capacitive triangle en une charge capacitive étoile. Ce faisant nous obtenons le circuit monophasé suivant. + \begin{center} + \begin{circuitikz} + \draw + (0,0) to [american voltage source,l=$\frac{34}{\sqrt{3}}\angle -30^\circ$ V, i= $I_{1}^{'}$] (0,5) to [R, l = $2\Omega$] (5,5) to [L, l = $j\Omega$](5,0) --(0,0) + (7,5) to [L, l =$j\Omega$,i=$I_{2}^{'}$] (7,0)--(10,0) to[C, l=$-j\Omega$] (10,5) -- (7,5); + \draw [fill=black] (4.8,3)node(a){} circle (2pt); + \draw [fill=black] (7.3,3)node(b){} circle (2pt); + \draw [<->,>=stealth] (a) to [bend left] node[pos=0.5,fill=white] {$8j\Omega$} ++(2,0); + \end{circuitikz} + \end{center} +En faisant la loi des mailles sur les 2 mailles du circuit nous obtenons : +\begin{equation*} + \left \{ +\begin{array}{rcl} +&\frac{34}{\sqrt{3}}\angle -30^\circ&=(2+j)I_{1}^{'} +6j I_{2}^{'} \\ +&0 &= 6jI_{1}^{'} + 7jI_{2}^{'} +\end{array} +\right. +\Leftrightarrow +\left \{ +\begin{array}{rcl} +I_{1}^{'} &=& 4.267\angle 34.23^\circ [A]\\ +I_{2}^{'} &=& 3.657\angle -145.77^\circ [A] +\end{array} +\right. +\end{equation*} +Sachant que les courant $I_{1}^{'}$ et $I_{2}^{'}$ sont les courants résultant de la transformation triangle étoile, nous pouvons trouver les courants $I_1$ et $I_2$ comme tel : +\begin{equation*} +\left \{ +\begin{array}{rcl} +|I_{1} |&=& |\frac{I_{1}^{'}}{\sqrt{3}}|~~ \mbox{et} ~~\phi_{I_{1}} = \phi_{I_{1}^{'}} +30^\circ \\ +|I_{2} |&=& |\frac{I_{2}^{'}}{\sqrt{3}}|~~ \mbox{et} ~~\phi_{I_{2}} = \phi_{I_{2}^{'}} +30^\circ +\end{array} +\right. +\Leftrightarrow +\left \{ +\begin{array}{rcl} +I_1 &=& 2.463\angle 64.23^\circ [A]\\ +I_2 &=& 2.11\angle -115.77^\circ [A] +\end{array} +\right. +\end{equation*} \end{enumerate} \end{solution} - \section{Question Craeye : transitoire} -Soit le circuit commuté ci-dessous. Donnez l'expression temporelle de la tension $V_c$ pour $t>0$ (l'interrupteur passe de la borne $A$ à la borne $B$ en $t=0$). Les valeurs des éléments du circuits sont: $R_o = R_1 = R = \SI{1}{\kilo\ohm}$, $C=\SI{1}{\nano\farad}$ et $L_o = L = \SI{1}{\milli\henry}$. +Soit le circuit commuté ci-dessous. Donnez l'expression temporelle de la tension $V_c$ pour $t>0$ (l'interrupteur passe de la borne $A$ à la borne $B$ en $t=0$). Les valeurs des éléments du circuits sont: $R_o = R_1 = R = 1k\Omega$, $C=1nF$ et $L_o = L = 1mH $. \begin{center} \begin{circuitikz} - \draw (0,0) to[american voltage source,l=\SI{1}{\volt}] ++(0,4) to[R,l=$R_1$] ++(0,4) to[short,-o] ++(1,0) coordinate (B); + \draw (0,0) to[american voltage source,l=$1V$] ++(0,4) to[R,l=$R_1$] ++(0,4) to[short,-o] ++(1,0) coordinate (B); \draw (2,8) to[short] ++(2,0) to[R,l=$R$] ++(2,0) to[L,l=$L$] ++(0,-8) to[short] ++(-6,0); - \draw (2,0) to[american voltage source,l=\SI{2}{\volt}] ++(0,2) to[L,l=$L_o$] ++(0,2) to[R,l=$R_o$] ++(0,2) to[short,-o] ++(0,1) coordinate (A); + \draw (2,0) to[american voltage source,l=$2V$] ++(0,2) to[L,l=$L_o$] ++(0,2) to[R,l=$R_o$] ++(0,2) to[short,-o] ++(0,1) coordinate (A); \draw (1.4,7.4) to[short,o-] (2,8); \draw (4,0) to[short] ++(0,3) to[C,l=$C$,v>=$V_c$] ++(0,2) to[short] ++(0,3); @@ -131,19 +244,91 @@ \section{Question Craeye : transitoire} \draw (B) node[above]{$B$}; \end{circuitikz} \end{center} - \begin{solution} -\[V_c(t) = \frac{1}{2} + \frac{1}{2}e^{-10^6t}(\cos(10^6t)-\sin(10^6t))\] -ou la solution équivalente avec seulement un cosinus -\[ V_c(t) = \frac{1}{2} + 0.707 e^{-10^6t}\cos(10^6t+0.785) \] - +En $t<0$, on peut réécrire le circuit comme : +\begin{center} + \begin{circuitikz} + \draw + (0,0) to[american voltage source,l=$2V$] ++(0,3) to[R, l = $R_o$] (0,6) -- (1,6) to [R, l=$R$,i= $I_L(0^-)$,*-](4,6) -- (4,0)--(0,0) + (1,6) to [european voltages,open,v^=$V_{C}(0^-)$] (1,0) ; + \end{circuitikz} +\end{center} +Sur base du diviseur de tension et la loi des mailles sur l'unique maille nous obtenons : +\begin{equation*} + \left \{ +\begin{array}{rcl} +V_{C}(0^-) &=& \frac{R}{R+R_o} *2[V] = 1[V]\\ +I_L(0^-) &=& \frac{2}{R_0+R} = 10^{-3} [A] +\end{array} +\right. +\end{equation*} +En $t>0$, on peut réécrire le circuit comme : +\begin{center} + \begin{circuitikz} + \draw (0,0) to[american voltage source,l=$1V$] ++(0,4) to[R,l=$R_1$,v =$V_{R_1(s)}$, i =$I_1(s)$] ++(0,4) --(2,8) to [C,l=$C$, i=$I_2(s)$,v=$V_C(s)$](2,0) + (2,8) to [R, l=$R$,v=$V_R(s)$] (5,8) to [L,l=$L$, v=$V_L(s)$](5,0) --(0,0); + \end{circuitikz} +\end{center} +Sur base de la loi des mailles sur la maille de droite nous pouvons écrire : +\begin{align*} + & \Rightarrow &V_C(s) &= V_R(s)+V_L(s) ~~\mbox{avec}~~ V_L(s) = sL(I_1(s)-I_2(s))-LI_L(0^-)\\ + & \Leftrightarrow &V_C(s) &= R(I_1(s)-I_2(s))+sL(I_1(s)-I_2(s))-LI_L(0^-) \\ + & \Leftrightarrow &V_C(s) &= I_1(s)(R+sL)-I_L(0^-)-I_2(R+sL) ~~\mbox{avec}~~I_2(s) = sV_C(s)C -CV_C(0^-)\\ + & \Leftrightarrow & V_C(s) &= I_1(s)(R+sL)-I_L(0^-)-(sV_C(s)C -CV_C(0^-))(R+sL)\\ + & \Leftrightarrow &I_1(s)(R+sL) &= V_C(s) (1+RSC+s^2CL)+LI_L(0^-)-CV_C(0^-)(R+sL) +\end{align*} +\begin{equation*} + \Rightarrow I_1(s)= \frac{V_C(s)(1+RCs+CLs^2)+LI_L(0^-)-RCV_C(0^-)-CLV_C(0^-)s}{R+sL} +\end{equation*} +Sur base de la loi des mailles sur la maille de gauche nous pouvons écrire : +\begin{align*} + &\Rightarrow \frac{1}{s} = R_1I_1(s)+V_C(s)\\ + &\Leftrightarrow \frac{1}{s} = \frac{R_1}{R+sL}(V_C(s)(1+RCs+CLs^2)+LI_L(0^-)-RCV_C(0^-)-CLV_C(0^-)s) + V_C(s)\\ + & \Leftrightarrow R+sL = sR_1V_C(s)(1+RCs+CLs^2)+sR_1((LI_L(0^-))-CV_C(0^-)R-CLV_C(0^-)s)+V_C(s)(sL+R)s\\ + & \Leftrightarrow V_{C}(s)(s(R+R)+s^{2}(R_{1}RC+L)+R_{1}CLs^{3})=R+sL-sR_{1}LI_{L}(0^-)+sRR_{1}CV_{C}(0^-)+s^{2}R_{1}CLV_{C}(0^-) +\end{align*} +\begin{equation*} + \Rightarrow V_C(s) = \frac{R+s(L-R_1LI_L(0^-)+RR_1CV_C(0^-))+s^2R_1CLV_C(0^-)}{s(R_1+R+s(L+R_1RC)+s^2LCR_1)} +\end{equation*} +En substituant les valeurs nous obtenons : +\begin{equation*} + V_C(s) = \frac{1000+10^{-3}s+10^{-9}s^2}{s(2000+2*10^{-3}s+10^{-9}s^2)} = \frac{s^2+10^6s+10^{12}}{s(s^2+2*10^6s+2*10^{12})} +\end{equation*} +En effectuant la décomposition en fraction simple nous avons : +\begin{equation*} + V_C(s) = \frac{A}{s} + \frac{B*10^6}{(s+10^6)^2+10^{12}}+\frac{C*(10^6+s)}{(s+10^6)^2+10^{12})} \Leftrightarrow +\begin{dcases} + A+C =1\\ + 2*10^6 A+10^6B+10^6C = 10^6\\ + 2*10^{12}A = 10^{12}\\ +\end{dcases} +\Leftrightarrow +\begin{dcases} + A= \frac{1}{2}\\ + B=-\frac{1}{2}\\ + C=\frac{1}{2}\\ +\end{dcases} +\end{equation*} +Nous obtenons donc dans le domaine de Laplace : +\begin{equation*} + V_C(s) = \frac{1}{2}(\frac{1}{s} -\frac{10^6}{(s+10^6)^2+10^{12}}+\frac{(10^6+s)}{(s+10^6)^2+10^{12}} +\end{equation*} +Cela correspond dans le domaine temporel à : +\begin{equation*} + V_c(t) = \frac{1}{2} u(t) (1 +e^{-10^6t}(\cos(10^6t)-\sin(10^6t))) +\end{equation*} +La solution équivalente avec seulement un cosinus est +\begin{equation*} + V_c(t) = \frac{1}{2} + 0.707 e^{-10^6t}\cos(10^6t+0.785) +\end{equation*} +Le graphe ressemble à : \begin{center} \begin{tikzpicture} - \begin{axis}[enlargelimits=true,grid=major,ylabel=$V_c(t)$,xlabel=$t$] + \begin{axis}[enlargelimits=true,grid=major,ylabel=$V_c(t)$,xlabel=$t(s)$] \addplot [blue,domain=0:0.00001,samples=200]{0.5 + 0.5*e^(-(10^6)*x)*cos(deg(10^6*x + 0.785))}; \end{axis} \end{tikzpicture} \end{center} \end{solution} -\end{document} +\end{document} \ No newline at end of file diff --git a/src/q4/circmes-ELEC1370/exam/2013/Juin/Mineure/circmes-ELEC1370-exam-2013-Juin-Mineure.tex b/src/q4/circmes-ELEC1370/exam/2013/Juin/Mineure/circmes-ELEC1370-exam-2013-Juin-Mineure.tex index a05d134a6..663a74080 100644 --- a/src/q4/circmes-ELEC1370/exam/2013/Juin/Mineure/circmes-ELEC1370-exam-2013-Juin-Mineure.tex +++ b/src/q4/circmes-ELEC1370/exam/2013/Juin/Mineure/circmes-ELEC1370-exam-2013-Juin-Mineure.tex @@ -1,26 +1,30 @@ \documentclass[fr]{../../../../../../eplexam} \usepackage{../../../../../../eplunits} \usepackage[oldvoltagedirection]{circuitikz} +\usepackage{bodegraph} \usepackage{pgfplots} +\usepackage{amsmath} \usepackage{enumitem} + + \pgfplotsset{compat=newest} \tikzset{meter/.style={draw,thick,circle,fill=white,minimum size =0.75cm,inner sep=0pt}} \hypertitle{circmes-ELEC1370}{4}{ELEC}{1370}{2013}{Juin}{Mineure} -{Nicolas Verbeek\and Adrien Couplet\and Martin Van Essche\and Guillaume Gilson\and Guillaume Colinet} +{Brieuc Balon} {Claude Oestges, Bruno Dehez and Christophe Craeye} \section{Question Oestges : phaseurs} -Soit le circuit suivant opérant à $\SI{20}{\kilo\hertz}$ avec $V_o=2.46 \angle\ang{126.87}$ V, +Soit le circuit suivant opérant à $20kHz$ avec $V_o=2.46 \angle 126.87^\circ$ V, \begin{center} \begin{circuitikz} \draw (0,2.5) to[american controlled current source,l=$2 V_x$] (0,0) (2.5,2.5) -- (0,2.5) - (5,2.5) to [american voltage source,l_=$12\angle\ang{0}$ V] (2.5,2.5) - (2.5,0) to[R,l=$\SI{1}{\ohm}$,-*] (2.5,2.5) - (5,2.5) to[C,l=-j$\SI{1}{\ohm}$,i=$I_C$,v=$V_x$] (5,0) - (5,2.5) to[european resistor,*-,l=j$X$] (7.5,2.5) to [R,l=$\SI{1}{\ohm}$,v=$V_o$] (7.5,0) -- (5,0) + (5,2.5) to [american voltage source,l_=$12\angle 0^\circ$ V] (2.5,2.5) + (2.5,0) to[R,l=$1\Omega$,-*] (2.5,2.5) + (5,2.5) to[C,l=-1j$\Omega$,i=$I_C$,v=$V_x$] (5,0) + (5,2.5) to[european resistor,*-,l=j$X$] (7.5,2.5) to [R,l=$1\Omega$,v=$V_o$] (7.5,0) -- (5,0) (7.5,2.5) to [short,*-o,l=$B$] (8.5,2.5) (7.5,0) to [short,*-o,l=$A$] (8.5,0) (0,0) -- (5,0); @@ -38,13 +42,134 @@ \section{Question Oestges : phaseurs} \end{enumerate} \begin{solution} + Pour la résolution du circuit nous utiliserons les notations : + \begin{center} + \begin{circuitikz} + \draw + (0,2.5) to[american controlled current source,l=$2 V_x$] (0,0) + (2.5,2.5) -- (0,2.5) + (5,2.5) to [american voltage source,l_=$12\angle 0^\circ$ V, i = $I_S$] (2.5,2.5) + (2.5,0) to[R,l=$1\Omega$,i = $I_R$, v = $V_R$,-*] (2.5,2.5) + (5,2.5) to[C,l=-1j$\Omega$,i=$I_C$,v=$V_x$] (5,0) + (5,2.5) to[european resistor,*-,l=j$X$,v =$V_L$] (7.5,2.5) to [R,l=$1\Omega$,i=$I_0$,v=$V_o$] (7.5,0) -- (5,0) + (7.5,2.5) to [short,*-o,l=$B$] (8.5,2.5) + (7.5,0) to [short,*-o,l=$A$] (8.5,0) + (0,0) -- (5,0); + \end{circuitikz} +\end{center} +Sur base de la relation élémentaire $V=ZI$ nous pouvons écrire : +\begin{equation*} + I_o = 2.46\angle 126.87^\circ V +\end{equation*} \begin{enumerate} - \item $I_C = 3.48\angle\ang{-98.13}$ A - \item $X=\SI{1}{\ohm}$ - \item $V_s = 8.57 \angle\ang{-176.7}$ V et $V_\text{eff} = 6.05$ V - \item $P = \SI{10}{\watt}$ - \item $V_\text{th} = V_o$ et $Z_\text{th} = 0.615 \angle 36.87^\circ$ - \item $Z_\text{L} = Z_\text{th}^{*}$ +\item En faisant la loi des mailles sur la maille centrale et une équation des noeuds sur le noeud en bas de la résistance de $1\Omega$ nous pouvons écrire : +\begin{equation*} + \left \{ + \begin{array}{rcl} + V_R + 12 + V_x &=&0 \\ + 2 V_x + I_c + I_o &=& \frac{V_R}{1} + \end{array} + \right. + \Leftrightarrow + \left \{ + \begin{array}{rcl} + V_R + V_x &=& =-12 \\ + 2 V_x -V_R + \frac{V_x}{-j} &=& -2.46\angle 126.87^\circ + \end{array} + \right. +\end{equation*} +\begin{equation*} +\left \{ +\begin{array}{rcl} +V_R + V_x &=& =-12 \\ +V_x (2- \frac{1}{j}) - V_R &=& -2.46\angle 126.87^\circ + +\end{array} +\right. +\Leftrightarrow + \left \{ +\begin{array}{rcl} +V_x &=& = 3.385\angle 172.157^\circ [V] \\ +V_R &=& 8.658\angle -176.94^\circ [V] + +\end{array} +\right. +\end{equation*} +On peut trouver le courant $I_c$ sur base de la relation élémentaire $V=ZI$: +\begin{equation*} + I_c = \frac{V_x}{-j} = 3.38\angle -97.843^\circ [A] +\end{equation*} +\item En faisant une loi des mailles sur la maille de droite nous pouvons écrire : +\begin{equation*} + V_L = V_x - Vo = 2.4068\angle-141.261^\circ [V] +\end{equation*} +On peut trouver $jX$ sur base de la relation élémentaire $V = ZI$ : +\begin{equation*} + jX = \frac{V_L}{I_o}=0.978\angle 91.869^\circ = - 0.032 + 0.978j \simeq 1j \Leftrightarrow X = 1 \rightarrow L = \frac{1}{\omega} =\frac{1}{2\pi \cdot 20000} = 7.957 [\mu H] +\end{equation*} +\item La tenion au borne de la source commandé vaut $V_R$: +\begin{equation*} + V_s = 8.658\angle -176.94^\circ [V] \rightarrow V_{eff} = \frac{V_s}{\sqrt{2}} = 6.12[V] +\end{equation*} +\item En faisant une loi des noeuds sur le noeud de la borne positive de la source de tension nous obtenons : +\begin{equation*} + I_s = 2V_x - I_R = 2V_x - V_R = 2.38\angle 35.55^\circ [A] +\end{equation*} +En considérant que la tension $V_o$ est valeur de crête nous pouvons écrire\footnote{Si c'était la valeur efficace nous n'aurions pas la facteur 2 au dénominateur} : +\begin{equation*} + P = \Re(S) = \frac{VI_s^\ast}{2} =\Re(6* 2.38\angle -35.55^\circ)= 11.61 [W] +\end{equation*} +\item Nous connaissons la tension de Thévenin ($V_{Th} = V_o$) calculons maintenant la résistance équivalente ($Z_{eq}$) sur base du circuit modifié : + + \begin{center} + \begin{circuitikz} + \draw + (2.5,0) to[R,l=$1\Omega$] (2.5,2.5) + (2.5,2.5)--(5,2.5) to[C,l=$-1j\Omega$] (5,0) + (5,2.5) to[european resistor,l=j$X$,] (7.5,2.5) to [R,l=$1\Omega$] (7.5,0) -- (5,0) + (7.5,2.5) to [short,*-o,l=$B$] (8.5,2.5) + (7.5,0) to [short,*-o,l=$A$] (8.5,0) + (2.5,0) -- (5,0); + \end{circuitikz} + \end{center} +Nous pouvons observer que : +\begin{equation*} + Z_{eq} = 1\Omega || (jX\Omega +(-j\Omega||1\Omega)) = 0.447\angle 26.56^\circ [\Omega] +\end{equation*} +Le dipôle équivalent de Thévenin aux bornes $A-B$ est représenté ci-dessous : +\begin{center} + \begin{circuitikz} + \draw + (0,0) to [american voltage source,l=$V_{Th}$] (0,2) to [R,l=$Z_{eq}$](3,2) + (3,0)--(0,0); + \end{circuitikz} + \end{center} +\item L'impédance de charge qui maximiserait le transfert de la puissance est $Z_{charge} = Z_{eq}$. Prouvons l'égalité sur base d'un cas général: +\begin{center} + \begin{circuitikz} + \draw + (0,0) to [american voltage source,l=$V$] (0,2) to [european resistor,l=$Z_1$](3,2) to [european resistor, l=$Z_2$](3,0)--(0,0); + \end{circuitikz} + \end{center} + La puissance de $Z_2$ est donné par : +\begin{equation*} + P_{Z_2}(Z_2) = Z_2 I^2 ~~\mbox{avec}~~I=\frac{V}{Z_1+Z_2} +\end{equation*} +\begin{equation*} + \Leftrightarrow P_{Z_2}(Z_2) = V^2\frac{Z_2}{(Z_1+Z_2)^2} +\end{equation*} +Pour maximiser sa puissance nous allons voir pour quelles valeurs sa dérivée par rapport à $Z_2$ s'annule : +\begin{equation*} + \frac{\partial P_{Z_2}}{\partial Z_2}(Z_2) = V^2\frac{(Z_1+Z_2)^2-2(Z_1+Z_2)Z_2}{(Z_1+Z_2)^2}= V^2\frac{Z_1^2-Z_2^2}{(Z_1+Z_2)^2} =0 +\end{equation*} +\begin{equation*} + \Leftrightarrow Z_1^2=Z_2^2 \Rightarrow Z_1 = Z_2 +\end{equation*} +On peut voir que cette condition amène un maximum en regardant le signe de la dérivée seconde : +\begin{equation*} + \frac{\partial^2 P_{Z_2}}{\partial Z_2}(Z_2)= 2V^2\frac{Z_2-2Z_1}{(Z_1+Z_2)^4} +\end{equation*} +Dont le signe est négatif lorsque $Z_2=Z_1$. \end{enumerate} \end{solution} @@ -64,7 +189,7 @@ \section{Question Oestges : Bode et quadripôles} (8,2.5) to [open,v^=$V_o$] (8,0); \end{circuitikz} \end{center} -Le quadripôle de ce circuit est représenté par la matrice $Z$ suivante à la fréquence de $\SI{26.5}{\kilo\hertz}$ +Le quadripôle de ce circuit est représenté par la matrice $Z$ suivante à la fréquence de $26.5kHz$ \[ \begin{bmatrix} 6-2j & 4-6j \\ 4-6j & 7+2j \end{bmatrix} \] On demande de \begin{enumerate} @@ -73,22 +198,196 @@ \section{Question Oestges : Bode et quadripôles} \item Sachant que $Z_1$ et $Z_3$ sont des impédances de type R-L série ($R_1$, $L_1$ et $R_3$, $L_3$) et que $Z_2$ est une impédance de type R-C série ($R_2$, $C_2$) calculer les différents composants. \item Réécrire la matrice $Z$ du quadripôle en fonction des différent composants trouvés ci-dessus ainsi que de la pulsation $\omega$. \item Tracer le diagramme de Bode du gain $A_\text{vf}$ pour les valeurs calculées des éléments si on branche en sortie du quadripôle une résistance très grande (supposée infinie). De quel type de filtre s’agit-il ? - \item Calculer l'impédance d’entrée du circuit si on branche en sortie du quadripôle une résistance de $\SI{5}{\ohm}$. + \item Calculer l'impédance d’entrée du circuit si on branche en sortie du quadripôle une résistance de $5\Omega$. \end{enumerate} \begin{solution} \begin{enumerate} - \item La matrice $Z$ est la suivante - \[ [Z] = \begin{bmatrix} Z_1 + Z_2 & Z_2 \\ Z_2 & Z_2 + Z_3 \end{bmatrix} \] - \item $Z_1 = 2+4j$, $Z_2 = 4-6j$ et $Z_3 = 3+8j$ - \item $R_1 = \SI{2}{\ohm}$, $L_1=\SI{24}{\micro\henry}$, $R_3 = \SI{3}{\ohm}$, $L_3 = \SI{48}{\micro\henry}$, $R_3 = \SI{4}{\ohm}$, $C_2 = \SI{1}{\micro\farad}$. - \item La matrice $Z$ est la suivante - \[ [Z] = \begin{bmatrix}R_1+R_2+j\omega L_1 + \frac{1}{j\omega C_2} & R_2 + \frac{1}{j\omega C_2} \\ R_2 + \frac{1}{j\omega C_2} & R_2+R_3+j\omega L_3 + \frac{1}{j\omega C_2} \end{bmatrix} \] - \item Le gain s'écrit - \[ A_\text{vf} = \frac{j\omega C_2 R_2 + 1}{(j\omega)^2 L_1 C_2 + j\omega C_2 (R_1+R_2) +1} \] - C'est un filtre passe-bas. - \item $Z_\text{in} = 8.43 \angle\ang{11.09}$ + \item La matrice $Z$ est calculée grâce à : + \begin{equation*} + \begin{bmatrix} + V_i \\ + V_o + \end{bmatrix} = +\begin{bmatrix} +z_i & z_r \\ +z_f & z_o +\end{bmatrix} +\begin{bmatrix} +I_i \\ +I_o +\end{bmatrix} +\end{equation*} +En annulant le courant $I_o$ le circuit devient : +\begin{center} + \begin{circuitikz} + \draw + (2.5,2.5) to [european resistor,l^=$Z_1$,i=$I_i$] (5,2.5) + (5,2.5) to[european resistor,*-,l=$Z_2$] (5,0) + (7.5,2.5) to (5,2.5) + (7,2.5) to [short,-o] (8,2.5) + (5,0) to [short,-o] (8,0) + (5,0) to [short,-o] (2,0) + (2.5,2.5) to [short,-o] (2,2.5) + (2,2.5) to [open,v=$V_i$] (2,0) + (8,2.5) to [open,v^=$V_o$] (8,0); + \end{circuitikz} +\end{center} +Sur base de la relation élémentaire $Z=RI$ nous pouvons écrire : +\begin{equation*} +\left \{ + \begin{array}{rcl} + V_i &=& (Z_1+ Z_2)I_i \\ + V_o &=& Z_2 I_o +\end{array} +\right. +\Leftrightarrow +\left \{ + \begin{array}{rcl} + z_f &=& Z_1+ Z_2 [\Omega]\\ + z_i &=& Z_2 [\Omega] +\end{array} +\right. +\end{equation*} +En annulant le courant $I_i$ le circuit devient : +\begin{center} + \begin{circuitikz} + \draw + (2.5,2.5) to (5,2.5) + (5,2.5) to[european resistor,l=$Z_2$] (5,0) + (7.5,2.5) to[european resistor,-*,l_=$Z_3$,i=$I_o$] (5,2.5) + (7,2.5) to [short,-o] (8,2.5) + (5,0) to [short,-o] (8,0) + (5,0) to [short,-o] (2,0) + (2.5,2.5) to [short,-o] (2,2.5) + (2,2.5) to [open,v=$V_i$] (2,0) + (8,2.5) to [open,v^=$V_o$] (8,0); + \end{circuitikz} +\end{center} + +Sur base de la relation élémentaire $Z=RI$ nous pouvons écrire : +\begin{equation*} +\left \{ + \begin{array}{rcl} + V_i &=& Z_2 I_o \\ + V_o &=& (Z_2+Z_3) Io +\end{array} +\right. +\Leftrightarrow +\left \{ + \begin{array}{rcl} + z_r &=& Z_2 [\Omega]\\ + z_0 &=& Z_2+Z_3 [\Omega] +\end{array} +\right. +\end{equation*} +La matrice Z est donc : +\begin{equation*} + Z = \begin{bmatrix} Z_1 + Z_2 & Z_2 \\ Z_2 & Z_2 + Z_3 \end{bmatrix} +\end{equation*} + +\item Nous pouvons calculer les valeurs complexes de $Z_1$, $Z_2$ et $Z_3$ avec les valeurs données et la matrice Z calculée précédement : +\begin{equation*} + \left \{ + \begin{array}{rcl} + Z_1 +Z_2 &=& 6-2j \\ + Z_2 &=& 4-6j \\ + Z_2+Z_3 &=& 7+2j +\end{array} +\right. +\Leftrightarrow +\left \{ + \begin{array}{rcl} + Z_1 &=& 2+4j [\Omega] \\ + Z_2 &=& 4-6j [\Omega]\\ + Z_3 &=& 3+8j [\Omega] +\end{array} +\right. +\end{equation*} +\item Nous pouvons réécrire les relations calculées précédement sous la forme : +\begin{equation*} +\left\{\begin{matrix*}[l] +Z_1 = 2+4j = R_1+ j\omega L_1 \\ +Z_2 = 4-6j = R_2 + j\frac{1}{\omega C_2} \\ +Z_3 = 3+8j = R_3 +j\omega L_3 +\end{matrix*}\right. + \Leftrightarrow + \left\{\begin{matrix*}[l] + R_1 = 2 [\Omega]\\ + R_2 = 4 [\Omega]\\ + R_3 = 3 [\Omega]\\ + L_1 = \frac{4}{\omega} = \frac{4}{2*\pi*26500} = 24[\mu H]\\ + C_2 = \frac{1}{6\omega} = \frac{1}{6*2*\pi*26500}=1[\mu F]\\ + L_3 =\frac{8}{\omega} = \frac{8}{2*\pi*26500} = 48[\mu H]\\[1ex]\end{matrix*}\right. +\end{equation*} + +\item La matrice $Z$ est la suivante + \[ Z = \begin{bmatrix}R_1+R_2+j\omega L_1 + \frac{1}{j\omega C_2} & R_2 + \frac{1}{j\omega C_2} \\ R_2 + \frac{1}{j\omega C_2} & R_2+R_3+j\omega L_3 + \frac{1}{j\omega C_2} \end{bmatrix} \] + +\item Le gain s'écrit : + \begin{equation*} + A_{vf} = \frac{A_{vfo}}{1-\frac{z_r}{z_l}A_{vfo}} ~~\mbox{avec} ~~A_{vfo}= \frac{z_f}{z_i}\frac{z_L}{z_L+z_o} = \frac{z_f}{z_i}~~\mbox{car} ~~z_L \simeq \infty + \end{equation*} + \begin{equation*} + A_{vf}= \frac{\frac{z_f}{z_i}}{1-\frac{z_r}{z_L}\frac{z_f}{z_i}}= \frac{z_f}{z_i}=\frac{R_2+\frac{1}{j\omega C_2}}{R_1+R_2+j\omega L_1+\frac{1}{j\omega C_2}} = \frac{1+R_2 C_2 j \omega}{1+(R_1+R_2)j\omega C_2 + (j\omega)^2 L_1 C_2} + \end{equation*} + La fonction de transfert peut être réécrite sour la forme : + \begin{equation*} + H(j\omega) = \frac{1 + j\frac{\omega}{\omega_0}}{1+2\xi j\frac{\omega}{\omega_1}+ (j\frac{\omega}{\omega_1})^2}~~ + \mbox{avec}~~ + \left\{\begin{matrix*}[l] +\omega_0 = \frac{1}{R_2C_2} =2.5*10^{5} [\mbox{rad/s}]\\ +\omega_1 =\frac{1}{\sqrt{L_1C_1}} = 204124 [\mbox{rad/s}] \\ +\xi = \frac{\omega_1}{2}(R_1+R_2)C_2 = 0.6124 +\end{matrix*}\right. +\end{equation*} + +\begin{center} + \begin{tikzpicture}[ + gnuplot def/.append style={prefix={}}, +] + +% Grid Style +\tikzset{ + semilog lines/.style={black}, + semilog lines 2/.style={gray,dotted}, + semilog half lines/.style={gray, dotted}, + semilog label x/.style={below,font=\tiny}, + semilog label y/.style={above,font=\tiny} } + +% Magnitude Plot +\begin{scope}[xscale=7/5, yscale=3/50] + \UnitedB + \semilog{0}{8}{-30}{20} + %Asymp + \BodeGraph[green,samples=1000]{0:6.05}{\SOAmpAsymp{1}{0.6124}{204124}} + \BodeGraph[blue,samples=1000]{0:6.4}{-\POAmpAsymp{1}{0.000004}} + %Real + \BodeGraph[red,samples=1000]{0:6.7}{\SOAmp{1}{0.6124}{204124}-\POAmp{1}{0.000004}} +\end{scope} +% Phase plot +\begin{scope}[yshift=-5cm,xscale=7/5,yscale=3/180] + \UniteDegre + \OrdBode{30} + \semilog{0}{8}{-180}{90} + %Asymp + \BodeGraph[green,samples=1000]{0:8}{\SOArgAsymp{1}{0.6124}{204124}} + \BodeGraph[blue,samples=1000]{0:8}{-\POArgAsymp{1}{0.000004}} + %Real + \BodeGraph[red,samples=1000]{0:8}{\SOArg{1}{0.6124}{204124}-\POArg{1}{0.000004}} +\end{scope} +\end{tikzpicture} +\end{center} + C'est un filtre passe-bas. Le bleu correspond au tracé asymptotique du numérateur, le vert correspond au tracé asymptotique du dénominateur et le rouge correspond au tracé de la fonction de transfert. + + \item L'impédance d'entrée s'écrit : + \begin{equation*} + Z_{in} = z_i (1+\frac{z_r}{z_i}A_{if,o})~~ \mbox{avec}~~A_{if,o} = -\frac{z_f}{z_o+z_L} + \end{equation*} + \begin{equation*} + \Leftrightarrow Z_{in}= z_i (1-\frac{z_r}{z_i}\frac{z_f}{z_o+z_L}) = (6-2j)(1-\frac{4-6j}{6-2j}\frac{4-6j}{(7+2j)+5}) = \frac{306}{37}+\frac{60}{37}j = 8.427\angle 11.09^\circ [\Omega] + \end{equation*} + \end{enumerate} \end{solution} - -\end{document} +\end{document} \ No newline at end of file diff --git a/src/q4/circmes-ELEC1370/exam/2014/Juin/Majeure/circmes-ELEC1370-exam-2014-Juin-Majeure.tex b/src/q4/circmes-ELEC1370/exam/2014/Juin/Majeure/circmes-ELEC1370-exam-2014-Juin-Majeure.tex index 94bc7873e..4335d7fc9 100644 --- a/src/q4/circmes-ELEC1370/exam/2014/Juin/Majeure/circmes-ELEC1370-exam-2014-Juin-Majeure.tex +++ b/src/q4/circmes-ELEC1370/exam/2014/Juin/Majeure/circmes-ELEC1370-exam-2014-Juin-Majeure.tex @@ -1,13 +1,15 @@ \documentclass[fr]{../../../../../../eplexam} \usepackage{../../../../../../eplunits} \usepackage[oldvoltagedirection]{circuitikz} +\usepackage{bodegraph} \usepackage{pgfplots} +\usepackage{amsmath} \usepackage{enumitem} \pgfplotsset{compat=newest} \tikzset{meter/.style={draw,thick,circle,fill=white,minimum size =0.75cm,inner sep=0pt}} \hypertitle{circmes-ELEC1370}{4}{ELEC}{1370}{2014}{Juin}{Majeure} -{Nicolas Verbeek\and Adrien Couplet\and Martin Van Essche\and Guillaume Gilson\and Guillaume Colinet} +{Brieuc Balon} {Claude Oestges, Bruno Dehez and Christophe Craeye} \section{Question Oestges : phaseurs} @@ -15,15 +17,15 @@ \section{Question Oestges : phaseurs} \begin{center} \begin{circuitikz} \draw - to[american voltage source,l=$12\angle\ang{0}$] (0,2.5) - (2.5,2.5) to[R,l=$\SI{2}{\ohm}$] (0,2.5) - (2.5,2.5) to [C,l=-j$\SI{1}{\ohm}$] (5,2.5) - (2.5,0) to[european resistor,l=$Z$,-*] (2.5,2.5) - (5,2.5) to[R,v=$V_o$,l=$\SI{1}{\ohm}$] (5,0) + to[american voltage source,l=$12\angle 0^\circ $] (0,2.5) + (2.5,2.5) to[R,l=$2\Omega$] (0,2.5) + (2.5,2.5) to [C,l=-j$1\Omega$] (5,2.5) + (2.5,0) to[european resistor,l=$Z$] (2.5,2.5) + (5,2.5) to[R,v=$V_o$,l=$1\Omega$] (5,0) (0,0) -- (5,0); \end{circuitikz} \end{center} -On mesure une tension $V_o=3\angle\ang{20}$V. On demande de calculer +On mesure une tension $V_o=3\angle 20^\circ$V. On demande de calculer \begin{enumerate} \item La valeur de l'impédance $Z$ \item La valeur de l'inductance ou capacité mise en série avec une résistance de l'impédance $Z$ @@ -32,13 +34,78 @@ \section{Question Oestges : phaseurs} \end{enumerate} \begin{solution} -Solutions finales (contrôlées avec LTSpice): +Pour la résolution du circuit nous utiliserons les notations : +\begin{center} + \begin{circuitikz} + \draw + to[american voltage source,l=$12\angle 0^\circ $] (0,2.5) + (0,2.5)to[R,l=$2\Omega$,i=$I_R$,v=$V_R$] (2.5,2.5) + (2.5,2.5) to [C,l=-j$1\Omega$,v=$V_C$] (5,2.5) + (2.5,2.5)to[european resistor,l=$Z$,i=$I_Z$,v=$V_Z$,*-] (2.5,0) + (5,2.5) to[R,v=$V_o$,l=$1\Omega$,i=$I_o$] (5,0) + (2.5,2.5) node[label={[font=\footnotesize]above:A}]{} + (0,0) -- (5,0); + \end{circuitikz} +\end{center} +Sur base de la relation élémentaire $V=ZI$ nous pouvons écrire : +\begin{equation*} + I_o = \frac{V_o}{1} = 3\angle 70^\circ [A] +\end{equation*} +\begin{equation*} + V_C = -j *I_o = 3\angle 70^\circ [V] +\end{equation*} +Nous pouvons effectuer une loi des mailles sur la maille de droite: +\begin{equation*} + V_Z = V_o + V_C = 3\sqrt{2}\angle -25^\circ [V] +\end{equation*} +Nous pouvons effectuer une loi des mailles sur grande maille: +\begin{equation*} + V_R = 12 - V_o - V_C = 12 - V_Z = 8.349\angle 12.4^\circ [V] +\end{equation*} +Sur base de la relation élémentaire $V=ZI$ nous pouvons écrire : +\begin{equation*} + I_R = \frac{V_R}{2} = 4.175\angle 12.4^\circ [A] +\end{equation*} +Nous pouvons effectuer une loi des noeuds sur le noeud A : +\begin{equation*} + I_Z = I_R - I_o = 1.265\angle -5.878 ^\circ [A] +\end{equation*} \begin{enumerate} - \item $Z = 3.35\angle\ang{-19.12} = 3.165 - 1.097j$ - \item La partie imaginaire est négative, il s'agit d'une capacité - \[ C = \frac{1}{1.097\omega} \] - \item La puissance complexe vaut $S = 25.02 \angle\ang{-12.4} = 24.4 - 5.37j$.\\ Donc $P=\SI{24.4}{\watt}$ et $Q=5.37$ VAR. - \item Il faut que la puissance complexe soit purement réelle, et donc on trouve (par exemple) que $Z=1+j$. + \item Sur base de la relation élémentaire $V=ZI$ nous pouvons écrire : + \begin{equation*} + Z = \frac{V_Z}{I_Z} = 3.354\angle -19.122^\circ = 3.169-1.096j [\Omega] + \end{equation*} + \item Nous avons que : + \begin{equation*} + R= 3.17 [\Omega] et C = \frac{1}{1.096\omega}[F] + \end{equation*} + \item La puissance\footnote{Nous travaillons avec des phaseurs, ils représentent la valeur efficace de la tension ou du courant. S’il n’est pas spécifié dans l’énoncé si la valeur de la tension $V_o$ représente la valeur de crête ou la valeur efficace de la tension, n’hésitez pas à le spécifier dans votre exercice. Nous considérons que $V_o$ correspond à la valeur efficace dans la suite des calculs.} complexe de la source est : + \begin{equation*} + S = 12\angle 0^\circ * I_R^\ast = 12\angle 0^\circ * 4.175\angle -12.4^\circ = 50.1\angle -12.4^\circ =48.93 -10.76j [VA] + \end{equation*} + Donc P = 48.93 [W] et Q = 10.76 [VAR] + + \item Il existe plusieurs méthodes de résolution pour cette question. Celle qui suit est la plus générale. Pour annuler la puissance réactive, il faut que l'impédance équivalent vue par la source soit purement réelle. On remarque assez aisement que : + \begin{equation*} + Z_{eq} = ((1-j)||Z)+2 \Rightarrow \Im( Z_{eq})= 0 + \end{equation*} + \begin{equation*} + \Im( Z_{eq})= \Im((( \frac{1}{1-j}+\frac{R}{+Xj})^{-1})+2) = \Im(( \frac{1}{1-j}+\frac{R}{+Xj})^{-1}) = \Im((\frac{1-j+R+Xj}{(1-j)(R+Xj)})^{-1}) =0 + \end{equation*} + \begin{equation*} + \Leftrightarrow \Im(\frac{(1-j)(R+Xj)}{1+R+j(X-1)})=\Im(\frac{(1-j)(R+Xj)}{1+R+j(X-1)} \frac{(1+R)-j(X-1)}{(1+R)-j(X-1)} )= \Im((1-j)(R+Xj)(1+R-j(X-1)))=0 + \end{equation*} + \begin{equation*} + \Leftrightarrow \Im(R+Xj-Rj+X+R^2+RXj-R^2j+RX+Rj-X+R+jX-RXj+X^2-RX-X^2)=0 + \end{equation*} + \begin{equation*} + -R^2+2X-X^2=0 ~~\mbox{avec}~~[(R\ne -1,X\ne 1),R>0] + \end{equation*} + Les solution de cette équations sont : + \begin{equation*} + [(R=\pm 1,X=1),(R=0,X=0),(R=0,X=2)] + \end{equation*} + Sur base des conditions, la seule solution pour que la puissance réactive fournie par la source soit égale à 0 est $R=X=1$. Il faut que Z = 1+j. \end{enumerate} \end{solution} @@ -55,42 +122,170 @@ \section{Question Oestges : Bode et quadripôles} (5.5,-1.2) to [R,l=$R_3$] (5.5,-3) (opamp.+) to [short,-*] (4,1) (opamp.out) -- (10,0.5) to [short,*-o] (11,0.5) node [above]{$V_\text{out}$} - (-1,-3) -- (11,-3); + (0,-3) -- (11,-3)(4.5,-3) --(4.5,-3.4)node [ground] {}; \end{circuitikz} \end{center} On demande de calculer \begin{enumerate} \item La matrice $G$ associée au quadripôle de ce circuit \item Tracer le diagramme de Bode (amplitude et phase) du coefficient $g_f$ et en déduire la fonction de ce circuit. - \item Déterminer $Z_\text{out}$ si $Z_\text{in} = \SI{100}{\ohm}$. + \item Déterminer $Z_\text{out}$ si $Z_\text{in} = 100\Omega$. \end{enumerate} PS: il manque les données pour tracer le diagramme de Bode correctement - \begin{solution} \begin{enumerate} - \item On obtient les coefficients $g_{r}$ et $g_{o}$ de la matrice G du quadripôle en fermant l'entrée ($V_{i} = 0$) et en plaçant une source de courant test à la sortie. Les coefficients $g_{i}$ et $g_{f} $ s'obtiennent en laissant la sortie ouverte ($i_{0}$ = 0) et en plaçant une source test de tension à l'entrée. \\ - $g_{r} = \frac{i_{i}}{i_{o}}$: on voit que le courant provenant de la source test placée en sortie va entièrement rentré dans l'ampli-op car son impédance de sortie est supposée nulle. Ainsi, les tensions aux bornes + et - de l'amplificateur sont nulles et $i_{i} = 0$\\ - $g_{r} = 0$ \\ - $g_{o}$: même raisonnement que pour $g_{r}$. La tension en sortie du quadripôle est nulle et par conséquent $g_{o} = 0$ \\ - $g_{i} = \frac{i_{i}}{v_{i}}$: en considérant qu'aucun courant ne rentre dans l'ampli-op on trouve: +\item La matrice $G$ est calculée grâce à : \begin{equation*} - v_{i} = i_{i}\left(\frac{R_{2}}{1+j\omega C_{1}R_{2}} + \frac{1}{j\omega C_{2}} \right) - \end{equation*} - et donc - \begin{equation*} - g_{i} = \frac{(j\omega)^{2}C_{1}R_{2}C_{2}+j\omega C_{2}}{1+(R_{2}C_{2} + C_{1}R_{2})j\omega} - \end{equation*} - $g_{f} = \frac{V_{o}}{V_{i}}$: à nouveau en ne considérant qu'aucun courant ne rentre dans l'ampli-op et que les tensions aux bornes de ce dernier sont identiques on trouve: - \begin{equation*} - g_{f} = \left(\frac{R_{4}}{R_{3}}+1\right)\frac{1+j \omega C_{1}R_{2}}{1+j \omega(R_{2}(C_{2} + C_{1}))} + \begin{bmatrix} + I_i \\ + V_o + \end{bmatrix} = + \begin{bmatrix} + g_i & g_r \\ + g_f & g_o + \end{bmatrix} + \begin{bmatrix} + V_i \\ + I_o + \end{bmatrix} \end{equation*} - \item Diagramme de Bode de $g_{f}$: en posant $\omega _{1} = \frac{1}{C_{1}R_{2}}$ et $\omega _{2} = \frac{1}{R_{2}(C_{2}+C_{1})}$, on voit que $\omega_{1} > \omega_{2}$, - \begin{itemize} - \item $log\left(\frac{R_{4}}{R_{3}}+1\right)$ pour $\omega < \omega_{2}$ (pente de 0db/dec) - \item pente de -20dB/dec pour $ \omega_{2}< \omega < \omega_{1}$ - \item pente de 0dB/dec pour $\omega > \omega_{1}$ - \end{itemize} - \item $Z_{out}=0$ (deux façons de s'en convaincre, intuitivement l'impédance de sortie d'un ampli op idéal est nul, l'autre façon est de regarder le tableau des caractéristiques externes) +En annulant la tension $V_i$ le circuit devient : +\begin{center} + \begin{circuitikz} + \draw + (0,-3)to[short,i=$I_i$] + (0,1) -- (1,1) + to [C,l=$C_1$,*-] (4,1) + (1,1) -- (1,2.5) to [R,l=$R_2$] (4,2.5) -- (4,1) to [C,l=$C_2$] (4,-3) + (8,0.5) node[op amp, yscale=-1] (opamp) {} + (opamp.-) -- (5.5,0) -- (5.5,-1.2) to [R,l=$R_4$,*-] (10,-1.2) -- (10,0.5) + (5.5,-1.2) to [R,l=$R_3$] (5.5,-3) + (opamp.+) to [short,-*] (4,1) + (opamp.out) -- (10,0.5) to [short,i<=$I_o$,*-o] (11,0.5) + to [open,v^=$V_o$,-o] (11,-3)-- + (0,-3) (4.5,-3) --(4.5,-3.4)node [ground] {}; + \end{circuitikz} +\end{center} +Comme nous avons un amplificateur opérationnel que l'on considère comme idéal, son impédance d'entrée vaut $0\Omega$. Le courant $I_0$ passe donc entièrement dans l'amplificateur opérationnel et aucun courant ne passe dans le reste du circuit. Dès lors tout le circuit est à 0V. Nous obtenons donc : +\begin{equation*} +\left \{ + \begin{array}{rcl} + I_i &=& 0\cdot I_o \\ + V_o &=& 0 \cdot I_o +\end{array} +\right. +\Leftrightarrow +\left \{ + \begin{array}{rcl} + g_r &=& 0 \\ + g_o &=& 0 [\Omega] +\end{array} +\right. +\end{equation*} +En annulant le courant $I_o$ le circuit devient : +\begin{center} + \begin{circuitikz} + \draw + (0,-3)to[open,v^>=$V_i$,o-o] + (0,1) -- (1,1) + to [C,l=$C_1$,*-] (4,1) + (1,1) -- (1,2.5) to [R,l=$R_2$] (4,2.5) -- (4,1) to [C,l=$C_2$] (4,-3) + (8,0.5) node[op amp, yscale=-1] (opamp) {} + (opamp.-) -- (5.5,0) -- (5.5,-1.2) to [R,l=$R_4$,*-] (10,-1.2) -- (10,0.5) + (5.5,-1.2) to [R,l=$R_3$] (5.5,-3) + (opamp.+) to [short,-*] (4,1) + (opamp.out) -- (10,0.5) to [short,*-o] (11,0.5) + to [open,v^=$V_o$,-o] (11,-3)-- + (0,-3) (4.5,-3) --(4.5,-3.4)node [ground] {}; + \end{circuitikz} +\end{center} +Étant donné que l'amplificateur opérationnel est en rétroaction négative ($V^+=V^-$) et qu'il est considéré comme parfait ($i^-=i^+=0A$) nous obtenons: +\begin{equation*} +\left \{ + \begin{array}{rcl} + V_i &=& (\frac{R_2}{1+R_2C_1j\omega}+\frac{1}{j\omega C_2})I_i \\ + V_o &=& \frac{R_4+R_3}{R_3} V^- ~~\mbox{avec}~~V^-=V^+ = \frac{C_2}{(C_1||R_2)+C_2} V_i +\end{array} +\right. +\Leftrightarrow +\left \{ + \begin{array}{rcl} + I_i &=& j\omega C_2 \cdot \frac{1+R_2C_1j\omega}{1+R_2(C_1+C_2)j\omega} V_i\\ + V_o &=& \frac{R_4+R_3}{R_3} \frac{1+R_2C_1j\omega}{1+R_2(C_1+C_2)j\omega} V_i +\end{array} +\right. +\end{equation*} +\begin{equation*} + \Rightarrow +\left \{ + \begin{array}{rcl} + g_i &=& j\omega C_2 \cdot \frac{1+R_2C_1j\omega}{1+R_2(C_1+C_2)j\omega} [\Omega^{-1}]\\ + g_f &=& \frac{R_4+R_3}{R_3} \frac{1+R_2C_1j\omega}{1+R_2(C_1+C_2)j\omega} +\end{array} +\right. +\end{equation*} +La matrice G est donc : +\begin{equation*} + G = \begin{bmatrix}j\omega C_2 \cdot \frac{1+R_2C_1j\omega}{1+R_2(C_1+C_2)j\omega} & 0 \\ \frac{R_4+R_3}{R_3} \frac{1+R_2C_1j\omega}{1+R_2(C_1+C_2)j\omega} & 0 \end{bmatrix} +\end{equation*} +\item La fonction de transfert du gain $g_f$ peut être réécrite sous la forme générale : +\begin{equation*} +H(j\omega) = K \frac{j\frac{\omega}{\omega_0}}{1+j\frac{\omega}{\omega_1}}~~ +\mbox{avec}~~ +\left\{\begin{matrix*}[l] +K= \frac{R_3+R_4}{R_4}\\ +\omega_0 = \frac{1}{R_2C_1}\\ +\omega_1 = \frac{1}{R_2(C_1+C_2)} \\ +\end{matrix*}\right. +\end{equation*} +Nous allons effectuer le diagramme de Bode pour les valeurs de composants $C_1=1nF$,$C_2=100nF$,$R_2=R_4=10k\Omega$,$R_3=1k\Omega$. Cependant on peut remarque assez simplement pour n'importe quelle valeur de composant que comme $\omega_0>\omega_1$ on aura donc : +\begin{itemize} + \item Au niveau du gain : un gain égale à $20\log(K)$ avant $\omega_1$, gain qui décroit de $-20dB/dec$ entre $\omega_1$ et $\omega_0$ et un gain qui croit de $0dB/dec$ au-delà de $\omega_0$. + \item Au niveau du déphasage : un déphasage de $0^\circ$ avant $\omega_1$, un déphasage asymptotique\footnotetext{Qui sera atteint ou non selon les valeurs des capacités.} entre $\omega_1$ et $\omega_0$ et un déphasage de $0^\circ$ au-delà de $\omega_0$. +\end{itemize} +Le diagramme de Bode de la fonction de transfert du gain $g_f$ avec les paramètre définie précédemment est représentée ci-dessous : +\begin{center} + \begin{tikzpicture}[ + gnuplot def/.append style={prefix={}}, +] +% Grid Style +\tikzset{ + semilog lines/.style={black}, + semilog lines 2/.style={gray,dotted}, + semilog half lines/.style={gray, dotted}, + semilog label x/.style={below,font=\tiny}, + semilog label y/.style={above,font=\tiny} } +% Magnitude Plot +\begin{scope}[xscale=7/5, yscale=3/50] + \UnitedB + \semilog{0}{8}{-30}{40} + %Asymp + \BodeGraph[green,samples=1000]{0:4.5}{\POAmpAsymp{1}{0.00101}} + \BodeGraph[blue,samples=1000]{0:6}{-\POAmpAsymp{0.09091}{0.00001}} + %Real + \BodeGraph[red,samples=1000]{0:8}{\POAmp{1}{0.00101}-\POAmp{0.09091}{0.00001}} +\end{scope} +% Phase plot +\begin{scope}[yshift=-4cm,xscale=7/5,yscale=3/180] + \UniteDegre + \OrdBode{30} + \semilog{0}{8}{-90}{90} + %Asymp + \BodeGraph[green,samples=1000]{0:8}{\POArgAsymp{1}{0.00101}} + \BodeGraph[blue,samples=1000]{0:8}{-\POArgAsymp{0.09091}{0.00001}} + %Real + \BodeGraph[red,samples=1000]{0:8}{\POArg{1}{0.00101}-\POArg{0.09091}{0.00001}} +\end{scope} +\end{tikzpicture} +\end{center} + C'est un filtre passe-bas. Le bleu correspond au tracé asymptotique du numérateur, le vert correspond au tracé asymptotique du dénominateur et le rouge correspond au tracé de la fonction de transfert. +\item Comme nous avons un amplificateur opérationnel que l’on considère comme idéal, nous pouvons supposer que $Z_{out}=0\Omega$. Si cela ne vous apparaît pas vous pouvez toujours calculer l'impédance de sortie au moyen de la table des caractéristiques externes: +\begin{equation*} + Z_{out} = g_o-g_r\frac{g_fz_G}{1+g_iz_G}~~ \mbox{avec}~~ z_G = 100[\Omega] +\end{equation*} +\begin{equation*} + \Leftrightarrow Z_{out} = 0-0\cdot \frac{g_fz_G}{1+g_iz_G} = 0[\Omega] +\end{equation*} \end{enumerate} \end{solution} @@ -100,13 +295,13 @@ \section{Question Dehez : circuit magnétique couplé} \draw[american currents] (0,0) to [american voltage source,l=$V_s$] (0,3) (0,0) -- (3,0) - (3,3) to [R,v^=$V_x$,l_=$\SI{5}{\kilo\ohm}$] (3,0) - (3,3) to [R, l_=$\SI{1}{\kilo\ohm}$] (0,3) + (3,3) to [R,v^=$V_x$,l_=$5K\Omega$] (3,0) + (3,3) to [R, l_=$1k\Omega$] (0,3) (3,0) -- (6,0); \draw[american currents] (6,3) to [cI_=$0.04V_x$] (6,0) (6,0) to [short, -o] (9,0) - (8,0) to [R,l=$\SI{10}{\kilo\ohm}$] (8,3) + (8,0) to [R,l=$10k\Omega$] (8,3) (9,0) to [open,v_>=$V_o$] (9,3) (9,3) to [short, o-] (8,3) to [short, *-] (6,3); @@ -115,13 +310,76 @@ \section{Question Dehez : circuit magnétique couplé} On demande de \begin{enumerate} \item Représenter le dipôle équivalent de Thévenin de ce circuit - \item Supposons qu'on connecte ce circuit à un transformateur idéal avec rapport $a:1$ et une résistance de $\SI{16}{\ohm}$. Quelle valeur de $a$ maximise la puissance fournie à cette résistance? + \item Supposons qu'on connecte ce circuit à un transformateur idéal avec rapport $a:1$ et une résistance de $16\Omega$. Quelle valeur de $a$ maximise la puissance fournie à cette résistance? \end{enumerate} \begin{solution} \begin{enumerate} - \item $V_\text{oc} = -333.3 V_s$ et $R_\text{th} = \SI{10}{\kilo\ohm}$. - \item On obtient un maximum de puissance pour $a=25$. + \item Nous pouvons calculer $V_x$ sur base du diviseur de tension. + \begin{equation*} + V_x = \frac{5000}{1000+5000} V_s = \frac{5}{6} V_s [V] + \end{equation*} + Sur base de la relation élémentaire $V=ZI$ nous pouvons écrire : + \begin{equation*} + V_0 =- 10000*0.04V_x= -400 V_x = -\frac{1000}{3}V_s [V] + \end{equation*} + On remarque assez simplement que la résistance équivalente correspond à $10k\Omega$. Il est aussi possible d'arriver à cette conclusion en calculer le courant $I_N$ qui passe dans le résistance de $10k\Omega$ : + \begin{equation*} + I_N = 0.04V_x = 0.04*\frac{5}{6}V_s = \frac{1}{30}V_s \Rightarrow R_{eq}= - \frac{V_{Th}}{I_N}= 10[k\Omega] + \end{equation*} + Avec le signe négatif venant du fait que le courant $I_N$ est dans le même sens que $V_{Th}$. Le circuit équivalent peut être représenté comme ceci : + \begin{center} + \begin{circuitikz} + \draw + (0,0) to [american voltage source,l=$V_{Th}:\frac{1000}{3}V_s$] (0,3) to [R,l=$R_{eq}:10k\Omega$](3,3) + (0,0)--(3,0); + \end{circuitikz} + \end{center} + \item Nous pouvons représenté le circuit comme ceci : + +\begin{center} + \begin{circuitikz} + \draw + (0,0) to [american voltage source,l=$V_{Th}:\frac{1000}{3}V_s$] (0,2.05) to [R,l=$R_{eq}:10k\Omega$](3,2.05) + (0,-0.05)--(3,-0.05) + (4,1) node [transformer](T){} + (T.base) node{a:1} + (5.05,2.05) --(6,2.05) to[R,l=$16\Omega$](6,-0.05)--(5.05,-0.05); + \end{circuitikz} +\end{center} +En faisant passer la résistance du secondaire vers le primaire on obtient : +\begin{center} + \begin{circuitikz} + \draw + (0,0) to [american voltage source,l=$V_{Th}:\frac{1000}{3}V_s$] (0,2) to [R,l=$R_{eq}:10k\Omega$](3,2) to [R, l=$16a^2 \Omega$](3,0)--(0,0); + \end{circuitikz} + \end{center} +Pour que la puissance soit maximale au niveau de la seconde résistance il faut que $R_{eq}= 16a^2$ et donc que $a = 25$ tours. Prouvons l'égalité sur base d'un cas général: +\begin{center} + \begin{circuitikz} + \draw + (0,0) to [american voltage source,l=$V$] (0,2) to [european resistor,l=$Z_1$](3,2) to [european resistor, l=$Z_2$](3,0)--(0,0); + \end{circuitikz} + \end{center} + La puissance de $Z_2$ est donné par : +\begin{equation*} + P_{Z_2}(Z_2) = Z_2 I^2 ~~\mbox{avec}~~I=\frac{V}{Z_1+Z_2} +\end{equation*} +\begin{equation*} + \Leftrightarrow P_{Z_2}(Z_2) = V^2\frac{Z_2}{(Z_1+Z_2)^2} +\end{equation*} +Pour maximiser sa puissance nous allons voir pour quelles valeurs sa dérivée par rapport à $Z_2$ s'annule : +\begin{equation*} + \frac{\partial P_{Z_2}}{\partial Z_2}(Z_2) = V^2\frac{(Z_1+Z_2)^2-2(Z_1+Z_2)Z_2}{(Z_1+Z_2)^2}= V^2\frac{Z_1^2-Z_2^2}{(Z_1+Z_2)^2} =0 +\end{equation*} +\begin{equation*} + \Leftrightarrow Z_1^2=Z_2^2 \Rightarrow Z_1 = Z_2 +\end{equation*} +On peut voir que cette condition amène un maximum en regardant le signe de la dérivée seconde : +\begin{equation*} + \frac{\partial^2 P_{Z_2}}{\partial Z_2}(Z_2)= 2V^2\frac{Z_2-2Z_1}{(Z_1+Z_2)^4} +\end{equation*} +Dont le signe est négatif lorsque $Z_2=Z_1$. \end{enumerate} \end{solution} @@ -141,27 +399,95 @@ \section{Question Craeye : transitoire} (3.6,5) to [C,l=$C$] (3.6,0); \end{circuitikz} \end{center} -On demande la tension $V_o$ en $t>0$ avec les données numériques suivantes : $R_1=R_2=\SI{1}{\ohm}$, $L=\SI{1}{\milli\henry}$, $C = \SI{1}{\milli\farad}$ +On demande la tension $V_o$ en $t>0$ avec les données numériques suivantes : $R_1=R_2=1\Omega$, $L=1MH$, $C = 1mF$ \begin{solution} -Les conditions initiales sur le courant de l'inductance et sur la tension aux bornes de la capacité sont le suivantes: -\[ V_{L}(0-) = \SI{1}{\volt} \] -\[ I_{L}(0-) = \SI{1}{\ampere} \] -En incluant les conditions initiales, on obtient l'expression suivante pour la tension $V_{0}$ dans le domaine de Laplace: +En $t<0$, on peut réécrire le circuit comme : +\begin{center} + \begin{circuitikz} + \draw + (0,0) to[american voltage source,l=2V] (0,2.5) -- (0,5) to [R,l=$R_1$,i=$I_L(0^-)$] (6,5) + to [R,l=$R_2$] (6,0) + (4,5) to [european voltages,open,v^=$V_{C}(0^-)$] (4,0) (0,0)--(6,0) + (6.5,3.6) to [open, v^=$V_o$] (6.5,1); + \end{circuitikz} +\end{center} +Sur base du diviseur de tension et la loi des mailles sur l’unique maille nous obtenons : \begin{equation*} - V_{0}(s) = \frac{10^{-6}s^{2}+2\cdot 10^{-3}s +1}{s(10^{-6}s^{2} + 2\cdot 10^{-3}s +2)} + \left \{ +\begin{array}{rcl} +V_{C}(0^-) &=& \frac{R_2}{R_1+R_2} *2[V] = 1[V]\\ +I_L(0^-) &=& \frac{2}{R_1+R_2} = 1 [A] +\end{array} +\right. \end{equation*} -Qu'on peut développer en fractions simples: +En $t>0$, on peut réécrire le circuit comme : +\begin{center} + \begin{circuitikz} + \draw + (0,0) -- (5,0) + (3.6,5)--(5,5) + (-1.5,0) -- (0,0) + (-1.5,0) to[american voltage source,l=1V] (-1.5,5) -- (0,5) + (0,5) to [R,l=$R_1$,v=$V_{R_1}(s)$,i=$I_1(s)$] (1.8,5) + (1.8,5) to [L,l=$L$,v=$V_L(s)$] (3.6,5) + (5,5) to [R,l=$R_2$,i=$I_2(s)$] (5,0) + (5.5,3.6) to [open, v^=$V_o(s)$] (5.5,1) + (3.6,5) to [C,l=$C$,v=$V_C(s)$] (3.6,0); + \end{circuitikz} +\end{center} +Sur base de la loi des mailles sur la maille de droite nous pouvons écrire : +\begin{align*} + & \Rightarrow &V_o(s) &=V_C(s) ~~\mbox{avec}~~ V_C(s) = \frac{I_1(s)-I_2(s)}{sC}+\frac{V_C(0^-)}{s}\\ + & \Leftrightarrow &V_o(s) &= \frac{I_1(s)-I_2(s)}{sC}+\frac{V_C(0^-)}{s} \\ + & \Leftrightarrow &V_o(s) &= \frac{I_1(s)}{sC}-\textcolor{red}{\frac{R_2}{R_2}}\text{\footnotemark}\frac{I_2(s)}{sC} + \frac{V_C(0^-)}{s}\\ + & \Leftrightarrow & \frac{I_1(s)}{sC} &= V_o(s)(1+\frac{1}{R_2Cs})-\frac{V_o(0^-)}{s} +\end{align*} +\footnotetext{$^1$Petit trick pour arriver à exprimer $I_1(s)$ en fonction de $V_o(s)$.} \begin{equation*} - V_{0}(s) = \frac{0.5}{s} + \frac{5 \cdot 10^{-7}s + 10^{-3}}{10^{-6}s^{2} + 2\cdot 10^{-3}s + 2} + \Rightarrow I_1(s) = sCV_o(s)(1+\frac{1}{R_2sC})-V_C(0^-)C \end{equation*} -On obtient ainsi l'expression analytique de la solution: -\[ V_o(t) = \left[\frac{1}{2} + 0.708 e^{-10^3t} \cos(10^3 t + 0.785) \right]u(t) \] - -En utilisant les propriétés $ f(s)= \frac{s-b}{(s-b)^{2}+a^{2}} \rightarrow F(t) = e^{bt} cos(at)$ et $f(s) = \frac{1}{(s-b)^{2}+a^{2}} \rightarrow F(t) = \frac{e^{bt} sin(at)}{a}$ des tables de transformées inverses de Laplace, on peut réécrire la solution sous la forme suivante: -\[V_o(t) = \left[\frac{1}{2} + \frac{1}{2}e^{-10^3t}\cos(10^3t) + \frac{1}{2}e^{-10^3t}\sin(10^3t) \right]u(t) \] - -Le graphe ressemble à +Sur base de la loi des mailles sur la maille totale nous pouvons écrire : +\begin{align*} + & \Rightarrow &\frac{1}{s} &= I_1(s)(R_1+Ls)-I_L(0^-)+ V_o(s)\\ + & \Leftrightarrow &\frac{1}{s} &= sCV_o(s)(1+\frac{1}{R_2Cs})(R_1+Ls)-V_C(0^-)C(R_1+Ls)-LI_L(0^-)+V_o(s)\\ + & \Leftrightarrow &1 &= sV_o(s)(1+sCR_1+s^2LC+\frac{R_1}{R_2}+\frac{sL}{R_2})-sV_C(0^-)CR_1-LI_L(0^-)s-s^2LCV_C(0^-) +\end{align*} +\begin{equation*} + \Rightarrow V_o(s) = \frac{1+s(V_C(0^-)CR_1+LI_L(0^-))+s^2LCV_C(0^-)}{s(1+\frac{R_1}{R_2}+s(R_1C+\frac{L}{R_2})+s^2LC} +\end{equation*} +En substituant les valeurs nous obtenons : +\begin{equation*} + V_o(s) = \frac{1+2*10^{-3}+10^{-6}}{s(2+2*10^{-3}+10^{-6}s^2} = \frac{s^2+s*2*10^{3}+10^6}{s(2*10^6+2*10^3+s^2)} +\end{equation*} +En effectuant la décomposition en fraction simple nous avons : +\begin{equation*} + V_o(s) = \frac{A}{s} + \frac{B*10^3}{(s+10^3)^2+10^{6}}+\frac{C*(10^3+s)}{(s+10^3)^2+10^{6})} \Leftrightarrow +\begin{dcases} + A+C =1\\ + 2*10^3 A+10^3B+10^3C = 2*10^3\\ + 2*10^{6}A = 10^{6}\\ +\end{dcases} +\Leftrightarrow +\begin{dcases} + A= \frac{1}{2}\\ + B=\frac{1}{2}\\ + C=\frac{1}{2}\\ +\end{dcases} +\end{equation*} +Nous obtenons donc dans le domaine de Laplace : +\begin{equation*} + V_o(s) = \frac{1}{2}(\frac{1}{s} +\frac{10^3}{(s+10^3)^2+10^{6}}+\frac{(10^3+s)}{(s+10^3)^2+10^{6}} +\end{equation*} +Cela correspond dans le domaine temporel à : +\begin{equation*} + V_o(t) = \frac{1}{2} u(t)(1 +e^{-10^3t}(\cos(10^3t)-\sin(10^3t)) +\end{equation*} +La solution équivalente avec seulement un cosinus est : +\begin{equation*} + V_o(t) = \left[\frac{1}{2} + 0.708 e^{-10^3t} \cos(10^3 t + 0.785) \right]u(t) +\end{equation*} +Le graphe ressemble à : \begin{center} \begin{tikzpicture} \begin{axis}[enlargelimits=true,grid=major,ylabel=$V_o(t)$,xlabel=$t$] @@ -169,7 +495,5 @@ \section{Question Craeye : transitoire} \end{axis} \end{tikzpicture} \end{center} - \end{solution} - -\end{document} +\end{document} \ No newline at end of file diff --git a/src/q4/circmes-ELEC1370/exam/2014/Septembre/All/circmes-ELEC1370-exam-2014-Septembre-All.tex b/src/q4/circmes-ELEC1370/exam/2014/Septembre/All/circmes-ELEC1370-exam-2014-Septembre-All.tex index 377dcb6a3..13f3a4d1a 100644 --- a/src/q4/circmes-ELEC1370/exam/2014/Septembre/All/circmes-ELEC1370-exam-2014-Septembre-All.tex +++ b/src/q4/circmes-ELEC1370/exam/2014/Septembre/All/circmes-ELEC1370-exam-2014-Septembre-All.tex @@ -1,45 +1,100 @@ \documentclass[fr]{../../../../../../eplexam} \usepackage{../../../../../../eplunits} \usepackage[oldvoltagedirection]{circuitikz} +\usepackage{bodegraph} \usepackage{pgfplots} +\usepackage{amsmath} \usepackage{enumitem} \pgfplotsset{compat=newest} \tikzset{meter/.style={draw,thick,circle,fill=white,minimum size =0.75cm,inner sep=0pt}} - \hypertitle{circmes-ELEC1370}{4}{ELEC}{1370}{2014}{Août}{All} -{Nicolas Verbeek\and Adrien Couplet\and Martin Van Essche\and Guillaume Gilson\and Guillaume Colinet} +{Brieuc Balon} {Claude Oestges, Bruno Dehez and Christophe Craeye} - \section{Question Oestges : phaseurs} On considère le circuit suivant \begin{center} \begin{circuitikz} \draw to[european resistor,l=$jX$, -*] (0,2.5) - (2.5,2.5) to[american current source,l=$2\angle\ang{0}$A] (0,2.5) - (2.5,2.5) to [american voltage source,l=$12\angle\ang{0}$V] (5,2.5) - (2.5,0) to[L,l_=j$\SI{4}{\ohm}$,-*] (2.5,2.5) - (5,2.5) to[R,v=$V_o$,l=$\SI{2}{\ohm}$,*-] (5,0) + (2.5,2.5) to[american current source,l=$2\angle 0^\circ$A] (0,2.5) + (2.5,2.5) to [american voltage source,l=$12\angle 0^\circ$V] (5,2.5) + (2.5,0) to[L,l_=j$4\Omega$,-*] (2.5,2.5) + (5,2.5) to[R,v=$V_o$,l=$2\Omega$,*-] (5,0) (0,0) -- (5,0) (0,2.5) -- (0,4) to[R,l=$R$] (5,4) -- (5,2.5); \end{circuitikz} \end{center} -On mesure une tension $V_o=5.5\angle\ang{-101.1}$V. On demande de calculer +On mesure une tension $V_o=5.5\angle -101.1^\circ$V. On demande de calculer \begin{enumerate} \item Les valeurs de $R$ et de $X$ (ces valeurs sont réelles) \item la tension (amplitude et phase) aux bornes de la source de courant de 2A, ainsi que sa valeur efficace. \item la puissance délivrée par la source de tension \end{enumerate} - \begin{solution} -Solutions finales (contrôlées avec LTSpice): +Pour la suite des calculs nous utiliserons les notations telles qu’illustrées : +\begin{center} + \begin{circuitikz} + \draw + to[european resistor,l=$jX$,v=$V_X$,i=$I_X$,-*] (0,2.5) + (2.5,2.5) node [above]{$C$}{} to[american current source,l=$2\angle 0^\circ$A,v>=$V_C$] (0,2.5) node [left]{$B$}{} + (2.5,2.5) to [american voltage source,l=$12\angle 0^\circ$V,i=$I_S$] (5,2.5) + (2.5,0) node[below]{$A$}{} to[L,l^=j$4\Omega$,i<=$I_L$,v>=$V_L$,-*] (2.5,2.5) + (5,2.5) to[R,v^=$V_o$,l_=$2\Omega$,i=$I_o$,*-] (5,0) + (0,0) -- (5,0) + (0,2.5) -- (0,4) to[R,l_=$R$,v^=$V_R$,i=$I_R$] (5,4) -- (5,2.5); + \end{circuitikz} +\end{center} \begin{enumerate} - \item $R=\SI{11.38}{\ohm}$ et $X=\SI{0.562}{\ohm}$. - \item $V_s = 14.85\angle\ang{25.74}$ V et $I_s = 3.33 \angle\ang{-101.27}$ A - \item $P=\SI{-3.9}{\watt}$ + \item Sur base de la relation élémentaire V = ZI nous pouvons écrire : + \begin{equation*} + I_o = \frac{V_o}{2} = 2.75\angle -101.1^\circ [A] + \end{equation*} + Nous pouvons effectuer une loi des mailles sur la maille en bas à droite : + \begin{equation*} + V_L = V_o - 12\angle 0^\circ = 14.13\angle -157.54^\circ [V] + \end{equation*} + Sur base de la relation élémentaire V = ZI nous pouvons écrire : + \begin{equation*} + I_L = \frac{V_L}{4j} = 3.532\angle 112.455^\circ [A] + \end{equation*} + Nous pouvons effectuer une loi des noeuds sur le noeud $A$ : + \begin{equation*} + I_X = I_L +I_o = 1.962\angle 163.23^\circ [A] + \end{equation*} + Nous pouvons effectuer une loi des noeuds sur le noeud $B$ : + \begin{equation*} + I_R = I_X + 2\angle 0^\circ = 0.58\angle 77.91^\circ [A] + \end{equation*} + Nous pouvons effectuer une loi des mailles sur la maille globale : + \begin{equation*} + V_o + V_R +V_X = 0 \Leftrightarrow 5.5\angle -101.1^\circ + RI_R + jXI_X = 0 + \end{equation*} + Nous avons une équation pour deux inconnues. Seulement il faut se rappeler que ces équations sont des équations complexes et donc que nous avons 2 équations pour 2 inconnues. Le système donne donc : + \begin{equation*} + \left\{\begin{matrix*}[l] + -1.059+0.121R -0.566X = 0\\ + -5.3971+0.567R - 1.878X = 0 + \end{matrix*}\right. + \Leftrightarrow + \left\{\begin{matrix*}[l] + R = 11.38 \\ + X = 0.561 + \end{matrix*}\right. + \end{equation*} + \item Nous pouvons effectuer une loi des mailles sur la maille en bas à gauche : + \begin{equation*} + V_C = V_R + 12\angle 0^\circ = R I_R + 12\angle 0^\circ = 14.86\angle 25.75^\circ [V] \Rightarrow V_{eff} = |\frac{V_C}{\sqrt{2}}| = 10.5 [V] + \end{equation*} + Nous pouvons effectuer une loi des noeuds sur le noeud $C$ : + \begin{equation*} + I_S = -(2\angle 0^\circ + I_L) = 3.33\angle -101.28^\circ [A] + \end{equation*} + La puissance\footnote{Nous travaillons avec des phaseurs, ils représentent la valeur efficace de la tension ou du courant. S'il n'est pas spécifié dans l'énonce si la valeur de la tension $V_o$ représente la valeur de crête ou la valeur efficace de la tension, n'hésitez pas à le spécifier dans votre exercice. Nous considérons que $V_o$ correspond à la valeur efficace dans la suite des calculs. } active délivrée par la source est : +\begin{equation*} + P = \Re(S) = \Re(V\cdot I_S^\ast) = \Re(12\angle 0^\circ \cdot 3.33\angle 101.28^\circ) = -7.816[W] +\end{equation*} \end{enumerate} \end{solution} - \section{Question Oestges : Bode et quadripôles} On considère le circuit de filtrage suivant \begin{center}\centering @@ -57,29 +112,173 @@ \section{Question Oestges : Bode et quadripôles} (8,0.5) to [short,*-o] (9,0.5) node [above]{$V_\text{out}$}; \end{circuitikz} \end{center} -où l'amplificateur opérationnel est considéré comme idéal et $C_1 = C_2 = \SI{10}{\nano\farad}$, $R_1 = \SI{5}{\kilo\ohm}$ et $R_2 = \SI{1}{\kilo\ohm}$. On demande de +où l'amplificateur opérationnel est considéré comme idéal et $C_1 = C_2 = 10nF$, $R_1 = 5k\Omega$ et $R_2 = 1k\Omega$. On demande de \begin{enumerate} \item Déterminer l'expression analytique (sans remplacer par les valeurs) de la matrice \textbf{G} du quadripôle ainsi formé. \item Tracer le diagramme de Bode (amplitude et phase) du gain $g_f$ (pour les valeurs données des éléments). Quelle est la fonction de ce circuit (passe-haut, passe-bas, passe-bande) ? Quelle est la bande passante du filtre ? - \item Si la charge est une impédance de $\SI{1}{\kilo\ohm}$, déterminer la valeur de l’impédance d’entrée $Z_\text{in}$, respectivement à 1, 10 et $\SI{100}{\kilo\hertz}$. + \item Si la charge est une impédance de $1k\Omega$, déterminer la valeur de l’impédance d’entrée $Z_\text{in}$, respectivement à 1, 10 et $100kHz$. \end{enumerate} \begin{solution} \begin{enumerate} - \item La matrice \textbf{G} du quadripôle - \[ G = - \begin{pmatrix} - g_i & g_r\\ - g_f & g_o - \end{pmatrix} - = \begin{pmatrix} - \frac{j\omega C_1}{1+j\omega R_1 C_1} & 0\\ - \frac{-j \omega R_2 C_1}{(1+j \omega C_2 R_2)(1+j \omega C_1 R_1)} & 0 - \end{pmatrix} \] - \item Pour les valeurs données, nous avons $g_f = \frac{-j \omega 10^{-5}}{(1+5j \omega 10^{-5})(1+j \omega 10^{-5})} $ - Il s'agit d'un filtre passe bande. - \item Via le tableau des caractéristiques externes, on trouve - \[ Z_\text{in} = \frac{1}{g_i} = \frac{1+j\omega R_1 C_1}{j\omega C_1} \] + \item La matrice $G$ est calculée grâce à : + \begin{equation*} + \begin{bmatrix} + I_i \\ + V_o + \end{bmatrix} = +\begin{bmatrix} +g_i & g_r \\ +g_f & g_o +\end{bmatrix} +\begin{bmatrix} +V_i \\ +I_o +\end{bmatrix} +\end{equation*} +En annulant la tension $V_i$ le circuit devient : +\begin{center}\centering + \begin{circuitikz} + \draw + (9,-1) to [short,-*] (4,-1)--(0,-1) to[short,i=$I_i$](0,1) + to [R, l=$R_1$] (2,1) + (6,0.5) node[op amp] (opamp) {} + (opamp.-) to [C,l_=$C_1$] (2,1) + (opamp.+) to [short] (4,0) + (opamp.out) to [short] (8,0.5) + (4.5,1) to [short,*-*] (4.5,2.5) to [R,l=$R_2$,-*] (8,2.5) -- (8,0.5) + (4.5,2.5) -- (4.5,4) to [C,l=$C_2$] (8,4) -- (8,2.5) + (4,0) -- (4,-1.5) node [ground] {} + (8,0.5) to [short,i<=$I_o$,*-o] (9,0.5) to [open,v^=$V_o$,-o] (9,-1); + \end{circuitikz} +\end{center} +Comme nous avons un amplificateur opérationnel que l'on considère comme idéal, son impédance d'entrée vaut $0\Omega$. Le courant $I_0$ passe donc entièrement dans l'amplificateur opérationnel et aucun courant ne passe dans le reste du circuit. Dès lors tout le circuit est à 0V. Nous obtenons donc : +\begin{equation*} +\left \{ + \begin{array}{rcl} + I_i &=& 0\cdot I_o \\ + V_o &=& 0 \cdot I_o +\end{array} +\right. +\Leftrightarrow +\left \{ + \begin{array}{rcl} + g_r &=& 0 \\ + g_o &=& 0 [\Omega] +\end{array} +\right. +\end{equation*} +En annulant le courant $I_o$ le circuit devient : +\begin{center}\centering + \begin{circuitikz} + \draw + (0,-1) to [short,o-*] (4,-1) to [short,-o] (9,-1) + (0,1) to [short,o-] (1,1) + (0,-1) to [open,v^>=$V_i$,-o](0,1) + (0.5,1)to [R, l=$R_1$,i=$I_i$] (2.5,1) + (6,0.5) node[op amp] (opamp) {} + (opamp.-) to [C,l_=$C_1$] (2.5,1) + (opamp.+) to [short] (4,0) + (opamp.out) to [short] (8,0.5) + (4.5,1) to [short,*-*] (4.5,2.5) to [R,l=$R_2$,-*] (8,2.5) -- (8,0.5) + (4.5,2.5) -- (4.5,4) to [C,l=$C_2$] (8,4) -- (8,2.5) + (4,0) -- (4,-1.5) node [ground] {} + (8,0.5) to [short,*-o] (9,0.5) to [open,v^<=$V_o$,-o](9,-1); + \end{circuitikz} +\end{center} +Étant donné que l'amplificateur opérationnel est en rétroaction négative ($V^+=V^-=0V$). Dès lors, sur base de la relation élémentaire $V = ZI$ nous pouvons écrire : +\begin{equation*} +\left \{ + \begin{array}{rcl} + V_i &=& (r_1+\frac{1}{j\omega C_1}I_i \\ + V_o &=& -(j\omega C_2 +\frac{1}{R_2})^{-1} I_i +\end{array} +\right. +\Leftrightarrow +\left \{ + \begin{array}{rcl} + I_i &=& \frac{j\omega C_1}{1+R_1C_1j\omega} V_i\\ + V_o &=& - \frac{R_2C_1j\omega}{(1+R_2C_2j\omega)(1+R_1C_1j\omega)}V_i +\end{array} +\right. +\end{equation*} +\begin{equation*} + \Rightarrow +\left \{ + \begin{array}{rcl} + g_i &=& \frac{j\omega C_1}{1+R_1C_1j\omega} [\Omega^{-1}]\\ + g_f &=& -\frac{R_2C_1j\omega}{(1+R_2C_2j\omega)(1+R_1C_1j\omega)} +\end{array} +\right. +\end{equation*} +La matrice G est donc : +\begin{equation*} + G = \begin{bmatrix} \frac{j\omega C_1}{1+R_1C_1j\omega} & 0 \\ - \frac{R_2C_1j\omega}{(1+R_2C_2j\omega)(1+R_1C_1j\omega)} & 0 \end{bmatrix} +\end{equation*} +\item La fonction de transfert du gain $g_f$ peut être réécrite sous la forme générale : +\begin{equation*} +H(j\omega) = K \frac{j\frac{\omega}{\omega_0}}{1+2\xi_1 j\frac{\omega}{\omega_1}+ (j\frac{\omega}{\omega_1})^2}~~ +\mbox{avec}~~ +\left\{\begin{matrix*}[l] +K= -1\\ +\omega_0 = \frac{1}{R_2C_1} = 10^5[\mbox{rad/s}]\\ +\omega_1 = \frac{1}{\sqrt{R_1R_2C_1C_2}} =44721 [\mbox{rad/s}] \\ +\xi_1 = \frac{\omega_1}{2}(R_1C_1+R_2C_2)=1.3416\\ +\end{matrix*}\right. +\end{equation*} +Le diagramme de Bode de la fonction de transfert du gain $g_f$ est représentée ci-dessous : +\begin{center} + \begin{tikzpicture}[ + gnuplot def/.append style={prefix={}}, +] +% Grid Style +\tikzset{ + semilog lines/.style={black}, + semilog lines 2/.style={gray,dotted}, + semilog half lines/.style={gray, dotted}, + semilog label x/.style={below,font=\tiny}, + semilog label y/.style={above,font=\tiny} } +% Magnitude Plot +\begin{scope}[xscale=7/5, yscale=3/50] + \UnitedB + \semilog{0}{8}{-50}{30} + %Asymp + \BodeGraph[green,samples=1000]{0:5.9}{\SOAmpAsymp{1}{1.3416}{44721}} + \BodeGraph[blue,samples=1000]{3.5:6.5}{-\IntAmp{100000}} + %Real + \BodeGraph[red,samples=1000]{2.5:6.8}{\SOAmp{1}{1.3416}{44721} - \IntAmp{100000}} +\end{scope} +% Phase plot +\begin{scope}[yshift=-4cm,xscale=7/5,yscale=3/180] + \UniteDegre + \OrdBode{30} + \semilog{0}{8}{-270}{0} + %Asymp + \BodeGraph[green,samples=1000]{0:8}{\SOArgAsymp{1}{1.3416}{44721}} + \BodeGraph[blue,samples=1000]{0:8}{ \IntArg{-100000} } + %Real + \BodeGraph[red,samples=1000]{0:8}{\SOArg{1}{1.3416}{44721} + \IntArg{-100000}} +\end{scope} +\end{tikzpicture} +\end{center} +C'est un filtre passe bande dont la bande passante est donnée par : +\begin{equation*} + BDW = 2\xi \omega_1 = 119852[rad/s] +\end{equation*} + \item L'impédance d'entrée s'écrit : + \begin{equation*} + Z_{in} = \frac{1}{g_i}\frac{1}{1+g_r\cdot A_{if,o}} ~~ \mbox{avec}~~A_{if,o} = -\frac{g_f}{g_i}\frac{1}{z_L+g_o} + \end{equation*} + \begin{equation*} + \Leftrightarrow Z_{in}= \frac{1}{g_i}\frac{1}{1-0\cdot \frac{g_f}{g_i}\frac{1}{z_L+0}}= \frac{1}{g_i}=\frac{1+R_1C_1j\omega}{C_1j\omega} [\Omega] + \end{equation*} + \begin{equation*} + Z_{in} = + \left\{\begin{matrix*}[l] + 5000-10^5j[\Omega] ~~\mbox{si}~~\omega = 1kHz\\ + 5000-10^4j[\Omega] ~~\mbox{si}~~\omega = 10kHz\\ 5000-10^3j[\Omega] ~~\mbox{si}~~\omega = 100kHz +\end{matrix*}\right. + \end{equation*} \end{enumerate} \end{solution} @@ -87,16 +286,16 @@ \section{Question Dehez : circuit magnétiques couplés} On considère le circuit suivant \begin{center} \begin{circuitikz} - \draw (0,0) to[R,l=\SI{12}{\ohm},*-] ++(-2,0) to[american voltage source,l=$12\angle\ang{0}\,\si{\volt}$] ++(0,2) to[R,l=\SI{2}{\ohm}] ++(2,0) to[R,l=\SI{4}{\ohm}] ++(0,-2); - \draw (0,0) to[R,l=\SI{4}{\ohm},-*] ++(0,-2) to[short] ++(-2,0) to[american voltage source,l=$24\angle\ang{0}\,\si{\volt}$,-*] ++(0,2); - \draw (0,0) to[C,l=\SI{-j2}{\ohm}] ++(3,0) coordinate (b1) to[L,l_=\SI{j4}{\ohm}] ++(0,-2) to[short] ++(-3,0); + \draw (0,0) to[R,l=\SI{12}{\ohm},*-] ++(-2,0) to[american voltage source,l=$12\angle 0^\circ$V] ++(0,2) to[R,l=$2\Omega$] ++(2,0) to[R,l=$4\Omega$] ++(0,-2); + \draw (0,0) to[R,l=$4\Omega$,-*] ++(0,-2) to[short] ++(-2,0) to[american voltage source,l=$24\angle 0^\circ$V,-*] ++(0,2); + \draw (0,0) to[C,l=$-2j\Omega$] ++(3,0) coordinate (b1) to[L,l_=$4j\Omega$] ++(0,-2) to[short] ++(-3,0); \draw (b1) node[below left]{$\bullet$}; - \draw ($ (b1) + (1,-2) $) coordinate (b4) to[L,l_=\SI{j3}{\ohm}] ++(0,2) coordinate (b2) to[C,l=\SI{-j1}{\ohm},-*] ++(3,0) to[short, -o] ++(1,0) coordinate (v1) node[right]{$+$}; - \draw (b4) to[short,-*] ++(3,0) coordinate (c) to[R,l=\SI{2}{\ohm}] ++(0,2); + \draw ($ (b1) + (1,-2) $) coordinate (b4) to[L,l_=$3j\Omega$] ++(0,2) coordinate (b2) to[C,l=$-1j\Omega$,-*] ++(3,0) to[short, -o] ++(1,0) coordinate (v1) node[right]{$+$}; + \draw (b4) to[short,-*] ++(3,0) coordinate (c) to[R,l=$2\Omega$] ++(0,2); \draw (c) to[short,-o] ++(1,0) coordinate (v2) node[right]{$-$}; \draw (b4) node[above right]{$\bullet$}; - \draw [<->,>=stealth] ($ (b1) + (-0.2,0.2) $) to [bend left] node[pos=0.5,above] {\SI{-j1}{\ohm}} ($ (b2) + (0.2,0.2) $); + \draw [<->,>=stealth] ($ (b1) + (-0.2,0.2) $) to [bend left] node[pos=0.5,above] {$-1j\Omega$} ($ (b2) + (0.2,0.2) $); \draw [<-,>=stealth] ($ (v1) + (0,-0.2) $) to [bend left] node[pos=0.5,right] {$V_o$} ($ (v2) + (0,0.2) $); \end{circuitikz} \end{center} @@ -104,19 +303,19 @@ \section{Question Dehez : circuit magnétiques couplés} \item Calculer le facteur de dispersion entre les deux inductances couplées magnétiquement. \item Calculer l'amplitude et la phase de la tension $V_o$. \end{enumerate} -On souhaite mesurer l’amplitude de cette tension au moyen d’un voltmètre idéal, mais dont le fond d’échelle, de \SI{100}{\milli\volt}, n’est à priori pas adapté. L’usage d’un transformateur (supposé idéal) est alors envisagé pour adapter la tension mesurée par l’ampèremètre. +On souhaite mesurer l’amplitude de cette tension au moyen d’un voltmètre idéal, mais dont le fond d’échelle, de $100mV$, n’est à priori pas adapté. L’usage d’un transformateur (supposé idéal) est alors envisagé pour adapter la tension mesurée par l’ampèremètre. \begin{center} \begin{circuitikz} - \draw (0,0) to[R,l=\SI{12}{\ohm},*-] ++(-2,0) to[american voltage source,l=$12\angle\ang{0}\,\si{\volt}$] ++(0,2) to[R,l=\SI{2}{\ohm}] ++(2,0) to[R,l=\SI{4}{\ohm}] ++(0,-2); - \draw (0,0) to[R,l=\SI{4}{\ohm},-*] ++(0,-2) to[short] ++(-2,0) to[american voltage source,l=$24\angle\ang{0}\,\si{\volt}$,-*] ++(0,2); - \draw (0,0) to[C,l=\SI{-j2}{\ohm}] ++(3,0) coordinate (b1) to[L,l_=\SI{j4}{\ohm}] ++(0,-2) to[short] ++(-3,0); + \draw (0,0) to[R,l=$12\Omega$,*-] ++(-2,0) to[american voltage source,l=$12\angle 0^\circ$V] ++(0,2) to[R,l=$2\Omega$] ++(2,0) to[R,l=$4\Omega$] ++(0,-2); + \draw (0,0) to[R,l=$4\Omega$,-*] ++(0,-2) to[short] ++(-2,0) to[american voltage source,l=$24\angle 0^\circ$V,-*] ++(0,2); + \draw (0,0) to[C,l=$-2j\Omega$] ++(3,0) coordinate (b1) to[L,l_=$4j\Omega$] ++(0,-2) to[short] ++(-3,0); \draw (b1) node[below left]{$\bullet$}; - \draw ($ (b1) + (1,-2) $) coordinate (b4) to[L,l_=\SI{j3}{\ohm}] ++(0,2) coordinate (b2) to[C,l=\SI{-j1}{\ohm},-*] ++(3,0) to[short, -o] ++(1,0) coordinate (v1); - \draw (b4) to[short,-*] ++(3,0) coordinate (c) to[R,l=\SI{2}{\ohm}] ++(0,2); + \draw ($ (b1) + (1,-2) $) coordinate (b4) to[L,l_=$3j\Omega$] ++(0,2) coordinate (b2) to[C,l=$-1j\Omega$,-*] ++(3,0) to[short, -o] ++(1,0) coordinate (v1); + \draw (b4) to[short,-*] ++(3,0) coordinate (c) to[R,l=$2\Omega$] ++(0,2); \draw (c) to[short,-o] ++(1,0) coordinate (v2); \draw (b4) node[above right]{$\bullet$}; - \draw [<->,>=stealth] ($ (b1) + (-0.2,0.2) $) to [bend left] node[pos=0.5,above] {\SI{-j1}{\ohm}} ($ (b2) + (0.2,0.2) $); + \draw [<->,>=stealth] ($ (b1) + (-0.2,0.2) $) to [bend left] node[pos=0.5,above] {$-1j\Omega$} ($ (b2) + (0.2,0.2) $); \draw [<-,>=stealth] ($ (v1) + (0,-0.2) $) to[] node[pos=0.5,right] {$V_o$} ($ (v2) + (0,0.2) $); \draw (v1) to[short,o-] ++(1,0) coordinate (t1) to[L] ++(0,-2) coordinate (t3) to[short,-o] ++(-1,0); @@ -133,66 +332,60 @@ \section{Question Dehez : circuit magnétiques couplés} \begin{enumerate}[resume] \item Calculer la valeur idéale que devrait prendre le rapport de transformation $n$ de ce transformateur pour avoir une mesure correspondant au fond d’échelle du voltmètre. \end{enumerate} - \begin{solution} \begin{enumerate} - \item Le coefficient de dispersion $\sigma$ est définit comme - \[ \sigma = 1 - \frac{M^2}{L_1L_2} = 1 - \frac{1}{12} = \frac{11}{12} \] - \item + \item Le facteur de dispersion s’écrit comme + \begin{equation*} + \sigma = 1 -\frac{M^2}{L_1L_2} = 1 - \frac{(-j)^2}{3j\cdot 4j} = \frac{11}{12} + \end{equation*} + \item Pour la suite des calculs nous utiliserons les notations telles qu’illustrées : \begin{center} \begin{circuitikz} - \draw (0,0) to[R,i>_=$I_2$,l=\SI{12}{\ohm},*-] ++(-2,0) to[american voltage source,l=$12\angle\ang{0}\,\si{\volt}$] ++(0,2) to[R,l=\SI{2}{\ohm},i=$I_1$] ++(2,0) to[R,l=\SI{4}{\ohm}] ++(0,-2); - \draw (0,0) to[R,l=\SI{4}{\ohm},-*] ++(0,-2) to[short] ++(-2,0) to[american voltage source,l=$24\angle\ang{0}\,\si{\volt}$,-*] ++(0,2); - \draw (0,0) to[C,l=\SI{-j2}{\ohm},i=$I_3$] ++(3,0) coordinate (b1) to[L,l_=\SI{j4}{\ohm}] ++(0,-2) to[short] ++(-3,0); + \draw (0,0) to[R,i<=$I_2$,l=$12\Omega$,*-] ++(-2,0) to[american voltage source,l=$12\angle 0^\circ$V] ++(0,2) to[R,l_=$6\Omega$,i=$I_1$] ++(2,0) -- ++(0,-2); + \draw (0,0) to[R,l=$4\Omega$,-*] ++(0,-2) to[short] ++(-2,0) to[american voltage source,l=$24\angle 0^\circ$V,-*] ++(0,2); + \draw (0,0) to[C,l=$-2j\Omega$,i=$I_3$] ++(3,0) coordinate (b1) to[L,l_=$4j\Omega$] ++(0,-2) to[short] ++(-3,0); \draw (b1) node[below left]{$\bullet$}; - \draw ($ (b1) + (1,-2) $) coordinate (b4) to[L,l_=\SI{j3}{\ohm}] ++(0,2) coordinate (b2) to[C,l=\SI{-j1}{\ohm},i<_=$I_4$,-*] ++(3,0) to[short, -o] ++(1,0) coordinate (v1) node[right]{$+$}; - \draw (b4) to[short,-*] ++(3,0) coordinate (c) to[R,l=\SI{2}{\ohm}] ++(0,2); + \draw ($ (b1) + (1,-2) $) coordinate (b4) to[L,l_=$3j\Omega$] ++(0,2) coordinate (b2) to[C,l=$-j\Omega$,i>_=$I_4$,-*] ++(3,0) to[short, -o] ++(1,0) coordinate (v1) node[right]{$+$}; + \draw (b4) to[short,-*] ++(3,0) coordinate (c) to[R,l=$2\Omega$] ++(0,2); \draw (c) to[short,-o] ++(1,0) coordinate (v2) node[right]{$-$}; \draw (b4) node[above right]{$\bullet$}; - - \draw [<->,>=stealth] ($ (b1) + (-0.2,0.2) $) to [bend left] node[pos=0.5,above] {\SI{-j1}{\ohm}} ($ (b2) + (0.2,0.2) $); + \draw [<->,>=stealth] ($ (b1) + (-0.2,0.2) $) to [bend left] node[pos=0.5,above] {$-j\Omega$} ($ (b2) + (0.2,0.2) $); \draw [<-,>=stealth] ($ (v1) + (0,-0.2) $) to [bend left] node[pos=0.5,right] {$V_o$} ($ (v2) + (0,0.2) $); - \draw (-1,1) node[scale=2]{$\circlearrowright$}; \draw (-1,-1) node[scale=2]{$\circlearrowright$}; \draw (1.5,-1) node[scale=2]{$\circlearrowright$}; \draw (5.5,-1) node[scale=2]{$\circlearrowright$}; - \end{circuitikz} \end{center} - En effectuant quatre fois la loi des mailles, on obtient le système suivant: - \[ \begin{cases} - 12 - 6I_1 - 12I_2 = 0 \\ - 24 + 12I_2 - 4(I_1-I_2-I_3) = 0 \\ - 4(I_1-I_2-I_3) + 2j I_3 - 4j I_3 - j I_4 = 0 \\ - 3j I_4 + j I_3 - j I_4 + 2I_4 = 0 - \end{cases} \] - Soit encore, en réécrivant ce système sous forme matricielle: - \[ \begin{bmatrix} - 6 & 12 & 0 & 0 \\ - 4 & -16 & -4 & 0 \\ - 4 & -4 & -4-2j & -j \\ - 0 & 0 & j & 2+2j - \end{bmatrix} \begin{bmatrix} - I_1 \\ I_2 \\ I_3 \\ I_4 - \end{bmatrix} = \begin{bmatrix} - 12 \\ 24 \\ 0 \\ 0 - \end{bmatrix} \] - D'où, en résolvant: - \[ \begin{cases} - I_1 &= 4.95\angle\ang{-13.43}\,\si{\ampere} \\ - I_2 &= 1.52\angle\ang{157.77}\,\si{\ampere} \\ - I_3 &= 5.61\angle\ang{-37.88}\,\si{\ampere} \\ - I_4 &= 1.99\angle\ang{-172.87}\,\si{\ampere} - \end{cases} \] - On peut alors déterminer la tension $V_o$: - \[ V_o = -2I_4 = 3.97\angle\ang{7.13}\,\si{\ampere} \] - \textbf{Note:} Une méthode équivalente (plus simple au niveau calculs) est de d'abord trouver le dipôle équivalent de Thévenin de la partie gauche du circuit (on trouve $V_\text{th} = 16$V et $Z_\text{th} = 2-2j$), et puis résoudre le système (réduit). - \item On souhaite observer au voltmètre une tension ne dépassant pas les \SI{100}{\milli\volt}. Pour cela nous utiliserons la formule liant les tensions et les nombres de spires de part et d'autre d'un transformateur - \[ \frac{V_1}{V_2} = \frac{N_1}{N_2} \Leftrightarrow \frac{V_o}{V_m} = n\] - On pose la condition: - \[ V_m < \SI{100}{\milli\volt} \Rightarrow \frac{V_o}{n} < \SI{100}{\milli\volt} \Rightarrow n > \frac{V_o}{0.1} = 39.7 \] - On en conclut que pour avoir une mesure correspondant au fond d'échelle du voltmètre, il faut que le nombre de spire $n$ côté circuit vaille au minimum 40 spires. + Nous pouvons effectuer quatre lois des mailles sur les quatre mailles indiquées: + \begin{equation*} + \begin{dcases} + 12 = 6I_1 - 12I_2 \\ + 24 = 12I_2 + 4(I_1+I_2-I_3) \\ + 2jI_3 -jI_4 = 4(I_1+I_2-I_3)\\ + -jI_3+I_4(2+2j) = 0\\ + \end{dcases} + \Leftrightarrow + \begin{dcases} + I_1 =4.945\angle-13.43^\circ [A] \\ + I_2 = 1.518^\circ [A] \\ + I_3 = 5.61\angle-37.875^\circ [A]\\ + I_4 = 1.984\angle 7.125^\circ [A]\\ + \end{dcases} + \end{equation*} + Sur base de la relation élémentaire $V=ZI$ nous pouvons écrire : + \begin{equation*} + V_o = 2\Omega \cdot I_4 = 3.968\angle 7.125^\circ [V] + \end{equation*} + \item Le fond d'échelle correspond à la valeur maximale que peut 'lire' l'appareil de mesure. Il faut donc que la tension mesurée par le voltmètre ne dépasse pas 100mV. La formule qui lie les tensions et le nombre de spires est donnée par : + \begin{equation*} + \frac{V_{primaire}}{V_{secondaire}} = \frac{N_{primaire}}{N_{secondaire}} + \Leftrightarrow \frac{V_o}{V_{voltmètre}}= \frac{n}{1} ~~\mbox{avec}~~V_{voltmètre}< 0.1[V] + \end{equation*} + \begin{equation*} + \Leftrightarrow \frac{V_o}{n}< 0.1 = V_{voltmètre} \Leftrightarrow n >\frac{V_o}{0.1} = \frac{3.968}{0.1} = 39.68 + \end{equation*} + Il faut que le nombre de spires soit au moins supérieur à 40. \end{enumerate} \end{solution} @@ -211,20 +404,88 @@ \section{Question Craeye : transitoire} \end{circuitikz} \end{center} On demande la tension $V_o$ en $t>0$ avec les données numériques suivantes : -$R=\SI{2.5}{\kilo\ohm}$, $L=\SI{0.5}{\milli\henry}$, $C = \SI{0.5}{\nano\farad}$ +$R=2.5k\Omega$, $L=0.5mH$, $C = 0.5nF$ \begin{solution} -L'expression analytique de la solution est -\[ V_o(t) = \left[1-\frac{1}{3} e^{-4\cdot10^6 t} + \frac{4}{3} e^{-10^6 t}\right]u(t) \] - +En $t<0$, on peut réécrire le circuit comme : +\begin{center} + \begin{circuitikz} + \draw + (0,0) to[american voltage source,l=2V,i=$I_L(0^-)$] (0,5) to [R,l=$R$] (1.8,5) + (1.8,5) to [L,l=$L$] (4,5) + (5,3.6) to [open, v^=$V_C(0^-)$] (5,1) + (0,0) --(4,0) --(4,2) + (4,3)--(4,5); + \end{circuitikz} +\end{center} +Le condensateur est chargé et nous obtenons : +\begin{equation*} + \left \{ +\begin{array}{rcl} +V_{C}(0^-) &=& 2[V]\\ +I_L(0^-) &=& 0 [A] +\end{array} +\right. +\end{equation*} +En $t>0$, on peut réécrire le circuit comme : +\begin{center} + \begin{circuitikz} + \draw + (0,0) -- (4,0) + (-1.5,0) -- (0,0) + (-1.5,0) to[american voltage source,l=1V,i=$I(s)$] (-1.5,5) + to [R,l=$R$,v=$V_R(s)$] (1.8,5) + (1.8,5) to [L,l=$L$,v=$V_L(s)$] (4,5) + (5,3.6) to [open, v^=$V_o(s)$] (5,1) + (4,5) to [C,l=$C$] (4,0); + \end{circuitikz} +\end{center} +Sur base de la loi des mailles sur la maille globale nous pouvons écrire : +\begin{align*} + & \Rightarrow &\frac{1}{s} &= V_R(s)+V_L(s)+V_o(s)\\ + & \Leftrightarrow &\frac{1}{s} &= RI(s) +V_L(s)+V_o(s) ~~\mbox{avec}~~ V_L(s) = sLI(s)-LI_L(0^-)\\ + & \Leftrightarrow &\frac{1}{s} &= (R+sL)I(s)-LI_L(0^-) +V_o(s) ~~\mbox{avec}~~I(s) = I_C(s) = sCV_o(s)-CV_C(0^-)\\ + & \Leftrightarrow & \frac{1}{s} &= (R+sL)(sCV_o(s)-CV_C(0^-))+V_o(s)-LI_L(0^-) \\ + & \Leftrightarrow &\frac{1}{s} &= RsCV_o(s) +s^2CLV_o(s)-RCV_C(0^-)-sCLV_C(0^-)+V_o(s)-LI_L(0^-)\\ + & \Leftrightarrow & 1 &= sV_o(s) (sRC+s^2LC +1)-sRCV_C(0^-)-s^2CLV_C(0^-)-sLI_L(0^-) +\end{align*} +\begin{equation*} + \Rightarrow V_o(s)= \frac{1+s(RCV_C(0^-)-LI_L(0^-))+s^2LCV_C(0^-)}{s(1+sRC+s^2LC)} +\end{equation*} +En substituant les valeurs nous obtenons : +\begin{equation*} + V_o(s) = \frac{1+2.5\cdot 10^{-6}s+5\cdot 10^{-13}s^2}{s(1+1.25\cdot 10^{-6}s +2.5\cdot 10^{-13}s^2)} = 2\frac{s^2+5\cdot10^6s+2\cdot 10^{12}}{s(s^2+5\cdot 10^6s+4\cdot 10^{12})} +\end{equation*} +En effectuant la décomposition en fraction simple nous avons : +\begin{equation*} + V_C(s) = \frac{A}{s} + \frac{B}{s+10^6}+\frac{C}{s+4\cdot 10^{6}} \Leftrightarrow +\begin{dcases} +A+B+C =2\\ + 5\cdot 10^6 A+ 4\cdot 10^6B+10^6C = 10^7\\ + 4\cdot 10^{12}A = 4\cdot 10^{12}\\ +\end{dcases} +\Leftrightarrow +\begin{dcases} + A= 1\\ + B= \frac{4}{3}\\ + C=-\frac{1}{3}\\ +\end{dcases} +\end{equation*} +Nous obtenons donc dans le domaine de Laplace : +\begin{equation*} + V_o(s) = \frac{1}{s} + \frac{4}{3} \frac{1}{s+10^6}-\frac{1}{3} \frac{1}{s+4\cdot 10^{6}} +\end{equation*} +Cela correspond dans le domaine temporel à : +\begin{equation*} + V_o(t) = \left[1+\frac{4}{3} e^{-10^6 t}-\frac{1}{3} e^{-4\cdot10^6 t} \right]u(t) +\end{equation*} Le graphe ressemble à \begin{center} \begin{tikzpicture} - \begin{axis}[enlargelimits=true,grid=major,ylabel=$V_o(t)$,xlabel=$t$] + \begin{axis}[enlargelimits=true,grid=major,ylabel=$V_o(t)$,xlabel=$t(s)$] \addplot [blue,domain=0:0.000012,samples=200]{1-(1/3)*e^(-4*(10^6)*x)+(4/3)*e^(-10^6*x)}; \end{axis} \end{tikzpicture} \end{center} \end{solution} - -\end{document} +\end{document} \ No newline at end of file diff --git a/src/q4/circmes-ELEC1370/exam/2016/Juin/Majeure/circmes-ELEC1370-exam-2016-Juin-Majeure.tex b/src/q4/circmes-ELEC1370/exam/2016/Juin/Majeure/circmes-ELEC1370-exam-2016-Juin-Majeure.tex index ae065e496..26d48973e 100644 --- a/src/q4/circmes-ELEC1370/exam/2016/Juin/Majeure/circmes-ELEC1370-exam-2016-Juin-Majeure.tex +++ b/src/q4/circmes-ELEC1370/exam/2016/Juin/Majeure/circmes-ELEC1370-exam-2016-Juin-Majeure.tex @@ -1,24 +1,28 @@ \documentclass[fr]{../../../../../../eplexam} \usepackage{../../../../../../eplunits} \usepackage[oldvoltagedirection]{circuitikz} +\usepackage{bodegraph} \usepackage{pgfplots} -\pgfplotsset{compat=newest} +\usepackage{amsmath} +\usepackage{enumitem} + +\pgfplotsset{compat=newest} +\tikzset{meter/.style={draw,thick,circle,fill=white,minimum size =0.75cm,inner sep=0pt}} \hypertitle{circmes-ELEC1370}{4}{ELEC}{1370}{2016}{Juin}{Majeure} -{Nicolas Verbeek\and Adrien Couplet\and Martin Van Essche\and Guillaume Gilson\and Guillaume Colinet} +{Brieuc Balon} {Claude Oestges, Bruno Dehez and Christophe Craeye} - \section{Question Oestges : phaseurs} -Soit le circuit ci-dessus, avec $I_o = 10\angle\ang{350}$. +Soit le circuit suivant avec $I_o = 10\angle 350^\circ$ [A]. \begin{center} \begin{circuitikz} \draw to[european resistor,l=$Z_\alpha$, i_>=$I_x$, -*] (0,2.5) to[american current source,l=$2\angle 0^\circ$A] (2.5,2.5) (2.5,0) to[american voltage source, l=$12\angle 0^\circ$V,-*] (2.5,2.5) - (2.5,2.5) to[R, l=$\SI{1}{\ohm}$] (5,2.5) - to[R,l=$\SI{1}{\ohm}$, i_>=$I_o$,*-] (5,0) + (2.5,2.5) to[R, l=$1\Omega$] (5,2.5) + to[R,l=$1\Omega$, i_>=$I_o$,*-] (5,0) (0,0) -- (5,0) - (0,2.5) to[R,l=$\SI{1}{\ohm}$] (0,5) -- (2.5,5) to[L,l=$j\SI{1}{\ohm}$,*-] (2.5,2.5) - (2.5,5) -- (5,5) to[C,l=$-j\SI{1}{\ohm}$] (5,2.5); + (0,2.5) to[R,l=$1\Omega$] (0,5) -- (2.5,5) to[L,l=$1j\Omega$,*-] (2.5,2.5) + (2.5,5) -- (5,5) to[C,l=$-1j\Omega$] (5,2.5); \end{circuitikz} \end{center} On demande: @@ -29,12 +33,79 @@ \section{Question Oestges : phaseurs} \item La puissance de la source de tension \end{enumerate} \begin{solution} -Solutions finales (contrôlées avec LTSpice): +Pour la suite des calculs nous utiliserons les notations telles qu’illustrées : +\begin{center} + \begin{circuitikz} \draw + to[european resistor,l_=$Z_\alpha$, i>_=$I_x$,v^=$V_Z$, -*] (0,2.5) node [left]{$A$}{} to[american current source,v^>=$V_C$,l_=$2\angle 0^\circ$A] (2.5,2.5) + (2.5,0) to[american voltage source, l_=$12\angle 0^\circ$V,i=$I_S$,-*] (2.5,2.5) + (2.5,2.5) to[R, l=$1\Omega$,v_=$V_2$,i=$I_2$] (5,2.5)node [right]{$B$}{} + to[R,l_=$1\Omega$, i_>=$I_o$,v^=$V_o$,*-] (5,0) + (0,0) to [short, -*] (2.5,0) node[below]{$D$}{} -- (5,0) + (0,2.5) to[R,l=$1\Omega$,v_>=$V_5$,i<=$I_5$] (0,5) -- (2.5,5) node [above]{$C$}{}to[L,l=$1j\Omega$,i<=$I_4$,v>=$V_4$,*-] (2.5,2.5) + (2.5,5) -- (5,5) to[C,l_=$-1j\Omega$,i<_=$I_3$,v^>=$V_3$] (5,2.5); + \end{circuitikz} +\end{center} \begin{enumerate} - \item $I_x = 2.17\angle 83.02^\circ$ A - \item $Z_\alpha = 3.98\angle 39.5^\circ \Omega$ - \item $V_s = 10.35\angle 44.67^\circ$ V - \item $P = -\SI{57.48}{\watt}$ +\item Sur base de la relation élémentaire V = ZI nous pouvons écrire : +\begin{equation*} + V_o = I_o \cdot 1\Omega = 10\angle 350^\circ [A] +\end{equation*} +Nous pouvons effectuer une loi des mailles sur la maille en bas à droite : +\begin{equation*} + V_2 = 12\angle 0^\circ - V_0 = 2.765\angle 38.9^\circ [V] +\end{equation*} +Sur base de la relation élémentaire V = ZI nous pouvons écrire : +\begin{equation*} + I_2 = \frac{V_2}{1\Omega} = 2.765\angle 38.9^\circ [A] +\end{equation*} +Nous pouvons effectuer une loi des noeuds sur le noeud $B$ : +\begin{equation*} + I_3 = I_2 - I_o = 8.44\angle 155.71^\circ [A] +\end{equation*} +Sur base de la relation élémentaire V = ZI nous pouvons écrire : +\begin{equation*} + V_3 = -j \cdot I_3 = 8.44\angle 65.71^\circ [V] +\end{equation*} +Nous pouvons effectuer une loi des mailles sur la maille en haut à droite : +\begin{equation*} + V_4 = V_2 +V_3 = 10.98\angle 59.19^\circ [V] +\end{equation*} +Sur base de la relation élémentaire V = ZI nous pouvons écrire : +\begin{equation*} + I_4 = \frac{V_4}{j} = 10.98\angle -30.81^\circ [A] +\end{equation*} +Nous pouvons effectuer une loi des noeuds sur le noeud centrale : +\begin{equation*} + I_S = I_2 + I_4 - 2\angle 0^\circ = 10.34\angle -22.08^\circ [A] +\end{equation*} +Nous pouvons effectuer une loi des noeuds sur le noeud $D$ : +\begin{equation*} + I_x = I_o - I_S = 2.168\angle 83.01^\circ [A] +\end{equation*} +\item Nous pouvons effectuer une loi des noeuds sur le noeud $C$ : +\begin{equation*} + I_5 = I_4+I_3 = 2.766\angle -51.08^\circ [A] +\end{equation*} +Sur base de la relation élémentaire V = ZI nous pouvons écrire : +\begin{equation*} + V_5 = 1\Omega \cdot I_5 = 2.766\angle -51.08^\circ [V] +\end{equation*} +Nous pouvons effectuer une loi des mailles sur la grande maille: +\begin{equation*} + V_Z = V_3 + V_5 -V_o = 8.63\angle 122.51^\circ [V] +\end{equation*} +Sur base de la relation élémentaire V = ZI nous pouvons écrire : +\begin{equation*} + Z_\alpha = \frac{V_Z}{I_x} = 3.98\angle 39.5^\circ = 3.07+2.53j[\Omega] +\end{equation*} +\item Nous pouvons effectuer une loi des mailles sur la maille en bas à gauche : +\begin{equation*} + V_C = V_4+V_5 = 10.352\angle 44.67^\circ [V] +\end{equation*} +\item La puissance\footnote{Nous travaillons avec des phaseurs, ils représentent la valeur éfficace de la tension ou du courant. S'il n'est pas spécifié dans l'énoncé si la valeur de la tension $V_o$ représente la valeur de crête ou la valeur éfficace de la tension, n'hésitez pas à le spécifier dans votre exercice. Nous considérons que $V_o$ correspond à la valeur éfficace dans la suite des calculs. } active délivrée par la source est : +\begin{equation*} + P = \Re(S) = \Re(V\cdot I_S^\ast) = \Re(12\angle 0^\circ \cdot 10.34\angle 22.08^\circ) = 114.98[W] +\end{equation*} \end{enumerate} \end{solution} @@ -68,10 +139,112 @@ \section{Question Oestges : Bode et quadripôles} \begin{enumerate} \item La fonction de transfert $\frac{V_\text{out}}{V_\text{in}}$ \item Le diagramme de Bode de la fonction de transfert trouvée au point précédent - \item L'impédance de sortie $Z_\text{out}$ en sachant que $Z_\text{in}=\SI{50}{\ohm}$ + \item L'impédance de sortie $Z_\text{out}$ en sachant que $Z_{in} = 50\Omega$. \end{enumerate} -En vous aidant des données numériques suivantes : $R_1 = \SI{21.6}{\kilo\ohm}$, $R_2 = \SI{28.8}{\kilo\ohm}$, $R_3 = \SI{10}{\kilo\ohm}$, $R = \SI{30}{\kilo\ohm}$, $C = \SI{1}{\nano\farad}$ -\nosolution +En vous aidant des données numériques suivantes : $R_1 = 21.6k\Omega$, $R_2 = 28.8k\Omega$, $R_3 = 10k\Omega$, $R = 30k\Omega$, $C = 1nF$. +\begin{solution} + \begin{enumerate} + Pour la suite des calculs nous utiliserons les notations telles qu’illustrées : + \begin{center} + \begin{circuitikz}[scale = 0.9, transform shape] + \draw + (-1,1) node [above]{$V_\text{in}$} to [short,o-] (-0.5,1) + to [R, l=$R_1$,v =$ $] (2,1) node[label ={[font=\footnotesize]below:$V_x$}]{} + (6,0.5) node[op amp] (opamp) {} + (2,1)to [C,l^=$C$,v=$ $] (opamp.-) + (opamp.+) to [short] (4,0) + (opamp.out) to [short] (8,0.5) + (4.5,1) node[label ={[font=\footnotesize]below:$B$}]{} to [short,*-] (4.5,2.5) to [R,l=$R_2$,v=$ $,-*] (8,2.5) + (4,0) -- (4,-0.5) node [ground] {} + (2,1) -- (2,4) to [C,l=$C$,v=$ $] (8,4) to [short,-*] (8,0.5) node[label ={[font=\footnotesize]below:$V_{out1}$}]{} to [R,l=$R_3$,v=$ $,-*] (11,0.5) node[label ={[font=\footnotesize]above:$A$}]{} -- (12,0.5) + (-0.5,1) to [short,*-] (-0.5,-2) to [R,l=$R$,v=$ $] (11,-2) -- (11,0.5) + + (14,0) node[op amp] (opamp2) {} + (opamp2.-) to [short,-*] (12,0.5) + (opamp2.+) to [short] (12,-0.5) + (12,-0.5) -- (12,-1) node [ground] {} + (opamp2.out) to [short,-*] (15.5,0) + + (12,0.5) -- (12,2) to [R, l=$R$,v=$ $] (15.5,2) -- (15.5,0) to [short,-o] (16,0) node [above]{$V_\text{out}$}; + \end{circuitikz} +\end{center} +Étant donné que les amplificateurs opérationnels sont en rétroaction négative et qu'ils sont considérés comme idéaux, nous pouvons dire que la tensions aux bornes positives ($A$ et $B$) et négatives de chacun des amplificateurs est nulle.\\ +\item Nous pouvons effectuer une loi des noeuds sur le noeud $B$ sachant que le courant $i^- =0$: +\begin{equation*} + V_x j\omega C = -\frac{V_{out1}}{R_2} \Leftrightarrow V_x = -\frac{V_{out1}}{R_2j\omega C} +\end{equation*} +Nous pouvons effectuer une loi des noeuds sur le noeud $V_x$ : +\begin{align*} + & &\frac{V_{in}-V_x}{R_1} &= (V_x -V_{out1})j\omega C +V_x j\omega C\\ + &\Leftrightarrow &\frac{V_{in}}{R_1} &= V_x (\frac{1}{R_1} + 2j\omega C) -V_{out1} j\omega C\\ + &\Leftrightarrow &V_{in} &= V_x (1 + 2R_1j\omega C) -V_{out1} j\omega CR_1\\ + &\Leftrightarrow &V_{in} &= -V_{out1}(\frac{1+2R_1Cj\omega}{R_2j\omega C}+R_1j\omega C) +\end{align*} +\begin{equation*} + \Rightarrow V_{out1} = -\frac{R_2j\omega C}{1+2R_1Cj\omega +R_1R_2C^2(j\omega)^2} V_{in} +\end{equation*} +Nous pouvons effectuer une loi des noeuds sur le noeud $A$ sachant que le courant $i^- =0$: +\begin{align*} + & &-\frac{V_{out}}{R} &=\frac{V_{out1}}{R_3} +\frac{V_{in}}{R} \\ + &\Leftrightarrow &-V_{out}R_3 &= R_3 V_{in} + R V_{out1}\\ + &\Leftrightarrow &-V_{out} R_3 &= R_3 V_{in} - \frac{RR_2j\omega C}{1+2R_1Cj\omega+R_1R_2C^2(j\omega)^2} V_{in} \\ + &\Leftrightarrow &-V_{out}R_3 &= V_{in}(\frac{R_3+2R_1R_3Cj\omega - RR_2j\omega C + R_1R_2R_3C^2(j\omega)^2}{1+2R_1Cj\omega+R_1R_2C^2(j\omega)^2} +\end{align*} +\begin{equation*} + \Rightarrow \frac{V_{out}}{V_{in}}= -(\frac{1+Cj\omega(2R_1-\frac{RR_2}{R_3})+R_1R_2C^2(j\omega)^2}{1+2R_1Cj\omega +R_1R_2C^2(j\omega)^2} +\end{equation*} +La fonction de transfert peut être réécrite sous la forme générale : +\begin{equation*} +H(j\omega) = K \frac{1 + 2\xi_0 j\frac{\omega}{\omega_0}+ (j\frac{\omega}{\omega_0})^2}{1+2\xi_1 j\frac{\omega}{\omega_1}+ (j\frac{\omega}{\omega_1})^2}~~ +\mbox{avec}~~ +\left\{\begin{matrix*}[l] +K= -1\\ +\omega_0 = \frac{1}{\sqrt{R_1R_2C^2}} =40093 [\mbox{rad/s}]\\ +\omega_1 = \frac{1}{\sqrt{R_1R_2C^2}} =40093 [\mbox{rad/s}] \\ +\xi_0 = \frac{\omega_0}{2}C (2R_1-\frac{RR_2}{R_3})=-0.866\text{\footnotemark}\\ +\xi_1 = \frac{\omega_1}{2}2R_1C = 0.866 +\end{matrix*}\right. +\end{equation*} +\footnotetext{$^1$Et oui les $\xi$ peuvent prendre des valeurs négatives} +\item Le diagramme de Bode de la fonction de transfert est représentée ci-dessous : +\begin{center} + \begin{tikzpicture}[ + gnuplot def/.append style={prefix={}}, +] +% Grid Style +\tikzset{ + semilog lines/.style={black}, + semilog lines 2/.style={gray,dotted}, + semilog half lines/.style={gray, dotted}, + semilog label x/.style={below,font=\tiny}, + semilog label y/.style={above,font=\tiny} } +% Magnitude Plot +\begin{scope}[xscale=7/5, yscale=3/50] + \UnitedB + \semilog{0}{8}{-30}{30} + %Asymp + \BodeGraph[green,samples=1000]{0:5.35}{\SOAmpAsymp{1}{0.866}{40093}} + \BodeGraph[blue,samples=1000]{0:5.35}{-\SOAmpAsymp{1}{-0.866}{40093}} + %Real + \BodeGraph[red,samples=1000]{0:8}{\SOAmp{1}{0.866}{40093}-\SOAmp{-1}{-0.866}{40093}} +\end{scope} +% Phase plot +\begin{scope}[yshift=-9cm,xscale=7/5,yscale=3/180] + \UniteDegre + \OrdBode{30} + \semilog{0}{8}{-180}{360} + %Asymp + \BodeGraph[green,samples=1000]{0:8}{\SOArgAsymp{1}{0.866}{40093}} + %Pas possible d'avoir la phase d'une constante donc on bidouille un peu + \BodeGraph[blue,samples=1000]{0:8}{-\SOArgAsymp{1}{-0.866}{40093}-\IntArg{-100000}-\IntArg{-100000}} + %Real + \BodeGraph[red,samples=1000]{0:8}{\SOArg{1}{0.866}{40093}-\SOArg{-1}{-0.866}{40093}} +\end{scope} +\end{tikzpicture} +\end{center} +C'est un filtre passe-tout qui engendre un déphasage au niveau de sa fréquence de coupure. Le bleu correspond au tracé asymptotique du numérateur, le vert correspond au tracé asymptotique du dénominateur et le rouge correspond au tracé de la fonction de transfert. + \end{enumerate} +\end{solution} \section{Question Dehez : pont de Wheatstone} Soit le pont de Wheatstone un peu modifié. @@ -86,16 +259,15 @@ \section{Question Dehez : pont de Wheatstone} \draw (0,0) to [sinusoidal voltage source, l=$V_s\; \omega_0$] (0, \y) to (\x, \y) % Left half bridge - to [american resistor, l_=$R$, *-*] (\x-\dx,\y-\dy) % Top left resistor + to [american resistor, l_=$R$, *-*] (\x-\dx,\y-\dy) node[label ={[font=\footnotesize]left:$A$}]{}% Top left resistor to [vR, l_=$Z_m$, -*] (\x,\y-2*\dy); % Bottom left resistor % Right half bridge \draw (\x,\y) to [american resistor, l=$R_x$] (7.5, 4.5) % Top right resistor - to [L, l=$L_x$,-*] (\x+\dx, \y-\dy) + to [L, l=$L_x$,-*] (\x+\dx, \y-\dy)node[label ={[font=\footnotesize]right:$B$}]{} to [american resistor, l_=$R$, -*] (\x,\y-2*\dy) % Bottom right resistor % Draw connection to (-) terminal of voltage source to (\x, 0) to (0,0); - % Draw voltmeter \draw (\x-\dx, \y-\dy) to [short, -o] (5.5,3) (5.5,3.5) to [open, v^=$V_{ab}$] (6.5,3.5) @@ -112,27 +284,43 @@ \section{Question Dehez : pont de Wheatstone} \begin{solution} \begin{enumerate} - \item - Ce circuit est un cas généralisé du pont de Wheatstone appelé pont de Maxwell\footnote{\url{https://fr.wikipedia.org/wiki/Pont_de_Maxwell}}. Celui-ci permet de mesurer la valeur d'une inductance inconnue grâce à une résistance et un condensateur étalonné ($Z_m$). - - Soit $A$ le point entre $R$ et $Z_m$, et $B$ le point entre $L_x$ et $R$. Par un pont diviseur de tension il est possible de calculer la tension en ces points: - \[ V_A = V_S\cdot\frac{Z_m}{Z_m+R} \qquad \text{et} \qquad V_B = V_S\cdot\frac{R}{R+R_x+j\omega L_x} \] - On obtient alors pour $V_{AB}$: - \[ V_{AB} = V_A - V_B = V_S\cdot\left(\frac{Z_m}{Z_m+R} - \frac{R}{R+R_x+j\omega L_x}\right) \] - On définit ensuite $Z_m$ de manière à ce que $V_{AB} = 0$: - \begin{align*} - \frac{Z_m}{Z_m+R} &= \frac{R}{R+R_x+j\omega L_x}\\ - Z_m &= \frac{R^2}{R_x + j\omega L_x} - \end{align*} - $Z_m$ étant la mise en parallèle d'une résistance $R_m$ et d'une capacité $C_m$: - \[ Z_m = \frac{R_m}{1+j\omega R_mC_m} = \frac{\frac{R^2}{R_x}}{1+j\omega \frac{L_x}{R_x}} \] - Les composants sont alors liés par les relations: - \[ R_x = \frac{R^2}{R_m} \qquad \text{et} \qquad L_x = R^2C_m \] - - \item Le résultat est indépendant de la fréquence de la source. - - \item La mesure de $V_{AB}$ se fait à l'aide d'un amplificateur d'instrumentation. -\end{enumerate} + \item Ce circuit est un cas généralisé du pont de Wheatstone appelé pont de Maxwell\footnote{\url{https://fr.wikipedia.org/wiki/Pont_de_Maxwell}}. Celui-ci permet de mesurer la valeur d'une inductance inconnue grâce à une résistance et un condensateur étalonné ($Z_m$). Lorsque le pont est à l'équilibre, on peut établir la relation : + \begin{equation*} + \frac{R}{Z_m}=\frac{R_x+j\omega L_x}{R} + \end{equation*} + Cette relation peut être trouvée en calculer la tension $V_{ab}$. Sur base des 2 diviseurs de tensions nous avons : + \begin{align*} + \begin{dcases} + V_A= \frac{R}{Z_m+R}V_s\\ + V_B= \frac{R}{R+R_x+j\omega L_x}V_s\\ + \end{dcases} + \Rightarrow V_{ab} = V_A - V_B = (\frac{R}{Z_m+R} - \frac{R}{R+R_x+j\omega L_x})V_s + \end{align*} + Quand le pont est équilibré, $V_{ab} =0$ et donc : + \begin{equation*} + \frac{R}{Z_m+R} =\frac{R}{R+R_x+j\omega L_x} \Leftrightarrow \frac{Z_m+R}{R} =\frac{R+R_x+j\omega L_x}{R} \Leftrightarrow \frac{R}{Z_m}=\frac{R_x+j\omega L_x}{R} + \end{equation*} + Sachant que $Z_m$ est la mise en parallèle d'une capacité $C_m$ et d'une résistance $R_m$ peut réécrire la relation comme : + \begin{align*} + &\Rightarrow \frac{R}{(\frac{1}{R_m}+j\omega C_m)^{-1}} = \frac{R_x+j\omega L_x}{R}\\ + &\Leftrightarrow \frac{R(1+j\omega C_mR_m)}{R_m} =\frac{R_x+j\omega L_x}{R}\\ + &\Leftrightarrow R^2+j\omega R^2C_mR_m =R_m R_x +j\omega R_mL_x + \end{align*} + \begin{equation*} + \Rightarrow + \begin{dcases} + R^2= R_mR_x\\ + R^2C_mR_m= R_mL_x\\ + \end{dcases} + \Leftrightarrow + \begin{dcases} + R_x= \frac{R^2}{R_m} \\ + L_x = R^2C_m\\ + \end{dcases} + \end{equation*} + \item La relation ne dépend pas de $\omega$ + \item Il faut utiliser un galvanomètre pour mesurer $V_{ab}$ car le galvanomètre permet de voir les fluctuations du courant. Quand le pont est à l'équilibre, il n'y a aucun courant qui passe dans le galvanomètre donc son aiguille est immobile. + \end{enumerate} \end{solution} \section{Question Craeye : transitoire} @@ -150,69 +338,116 @@ \section{Question Craeye : transitoire} (6,5) to [open, v^=$v_o$] (6,0); \end{circuitikz} \end{center} - On demande la tension $V_o$ en $t=0^+$ avec les données numériques suivantes: -$R=\SI{1}{\kilo\ohm}$, $L=\SI{1}{\milli\henry}$, $C = \SI{1}{\nano\farad}$ - +$R=1k\Omega$, $L=1mH$, $C =1nF$ \begin{solution} -Soit $I_o$ le courant traversant l'inductance $L$ et la résistance $R$, et $V_s$ la tension fournie en entrée par une des deux sources de tension. On observe: -\[ V_o(t) = RI_o(t) + L\frac{dI_o(t)}{dt} \qquad \text{et} \qquad I_o(t) = \frac{V_s-V_o(t)}{R} + C\frac{d\left(V_s-V_o(t)\right)}{dt} \] -En combinant les deux équations, on obtient: -\[ V_s - V_o(t) + RC \frac{d(Vs-V_o(t))}{dt} + \frac{L}{R}\frac{d(V_s-V_o(t))}{dt} + LC \frac{d^2(V_s-V_o(t))}{dt^2} = V_o(t)\] -Dès lors, en utilisant la transformée de Laplace unilatérale et ses propriétés de dérivation, on écrit: -\begin{equation*} -\begin{split} -V_o(s) & = (\frac{L}{R}+RC)(s(V_s(s)-V_o(s))-(V_s(0^-) - V_o(0^-))) \\ -& + LC(s^2(V_s(s)-V_o(s)) -s(V_s(0^-)-V_o(0^-)) - (V'_s(0^-) - V'_o(0^-))) \\ -& +V_s(s) - V_o(s) \\ -\end{split} -\end{equation*} -En considérant que le circuit était à l'équilibre en $t<0$, on peut définir les conditions initiales du circuit: -\[ V_o(0^-) =V_s(0^-).\frac{R}{2R} = \frac{1}{2} \qquad \text{et} \qquad V'_o(0^-)=0\] -\[ V_s(0^-) = 1 \qquad \text{et} \qquad V'_s(0^-) = 0\] -En appliquant ces conditions initiales, on réécrit: -\begin{equation*} -\begin{split} -2V_o(s) & = (\frac{L}{R}+RC)(s(V_s(s)-V_o(s))-\frac{1}{2}) \\ -& + LC(s^2(V_s(s)-V_o(s)) -\frac{1}{2}s ) \\ -& +V_s(s) \\ -\end{split} -\end{equation*} -On exprime finalement $V_o(s)$: -\[V_o(s) = \frac{V_s(s).(1 + s(RC+\frac{L}{R})+s^2LC)-\frac{1}{2}((RC+\frac{L}{R})+sLC)}{(2 + s(RC+\frac{L}{R})+s^2LC)}\] -En remplaçant $V_s(s)$ par $\frac{2}{s}$, on obtient: -\[ V_o(s) = \frac{\frac{2}{s} + 2\left(RC + \frac{L}{R}\right) + 2sLC - \frac{1}{2}\left(\left(RC+\frac{L}{R}\right)+sLC\right)}{2 + s\left(RC+\frac{L}{R}\right) + s^2LC} \] -Par soucis de lisibilité, nous considérons $a = LC$ et $b = RC + (L/R)$: -\begin{align*} - V_o(s) &= \frac{\frac{2 + 2sb + 2s^2a - s\frac{b}{2} - s^2\frac{a}{2}}{s}}{2 + sb + s^2a}\\ - &= \frac{s^2\frac{3a}{2} + s\frac{3b}{2} + 2}{s(s^2a + sb + 2)} +En $t<0$, on peut réécrire le circuit comme : +\begin{center} + \begin{circuitikz} + \draw + (5,2.5) to[R,l=$R$] (5,0) + (0,0) -- (5,0) + (0,0) to[american voltage source,l=1V] (0,5) to [open,v<=$V_C(0^-)$,*-*] (5,5) to[short,i=$I_L(0^-)$] (5,2.5) + (0,5) -- (0,6.5) to [R,l=$R$] (5,6.5) -- (5,5); + \end{circuitikz} +\end{center} +Sur base du diviseur de tension et la loi des mailles sur l’unique maille nous obtenons : +\begin{equation*} + \left \{ +\begin{array}{rcl} +V_{C}(0^-) &=& \frac{R}{R+R} \cdot 1[V] = 0.5[V]\\ +I_L(0^-) &=& \frac{1}{R+R} = 5\cdot 10^{-4} [A] +\end{array} +\right. +\end{equation*} +En $t>0$, on peut réécrire le circuit comme : +\begin{center} + \begin{circuitikz} + \draw + (5,2.5) to[R,l=$R$,v=$V_{R^{'}}(s)$] (5,0) + (-1.5,0) -- (5,0) + (-1.5,0) to[american voltage source,l=2V] (-1.5,5) + --(0,5) to [C,l=$C$,*-,v_<=$V_C(s)$] (5,5) + (5,5) to[L,l=$L$,i=$I_2(s)$,v_=$V_L(s)$,*-] (5,2.5) + (0,5) -- (0,6.5) to [R,l_=$R$,i_=$I_1(s)$,v^=$V_R(s)$] (5,6.5) -- (5,5) + (6,5) to [open, v^=$V_o(s)$] (6,0); + \end{circuitikz} +\end{center} +Sur base de la loi des mailles sur la maille du haut nous pouvons écrire : +\begin{align*} + & \Rightarrow &RI_1(s) &= V_C(s)~~\mbox{avec}~~ V_C(s) = \frac{I_2(s)-I_1(s)}{Cs}+\frac{V_C(0^-)}{s}\\ + & \Leftrightarrow &RI_1(s) &= \frac{I_2(s)-I_1(s)}{Cs}+\frac{V_C(0^-)}{s}\\ + & \Leftrightarrow &I_1(s)(R+\frac{1}{Cs}) &= \frac{V_C(0^-)}{s}+\frac{I_2(s)}{Cs}\\ + & \Leftrightarrow &I_1(s) &= \frac{I_2(s)+CV_C(0^-)}{Cs}\frac{Cs}{1+sRC} \end{align*} -Le polynôme du second degré au dénominateur a comme racines: -\[ \begin{cases} - s_1 &= \num{-1e6}+\num{1e6}j\\ - s_2 &= \num{-1e6}-\num{1e6}j -\end{cases} \] -On effectue une réduction en fractions simples de $V_o(s)$: +\begin{equation*} + \Rightarrow I_1(s)= \frac{I_2(s)+CV_C(0^-)}{1+sRC} +\end{equation*} +En réécrivant $V_o(s)$ : \begin{align*} - V_o(s) &= \frac{1}{a}\frac{s^2\frac{a}{2} + s\frac{b}{2} + 2}{s\left(s+\alpha-\beta j\right)\left(s+\alpha-\beta j\right)} \\ - &= \frac{1}{a}\left[\frac{\num{1e-12}}{s} + \frac{\frac{1}{4\sqrt{2}}\times10^{-12}\angle{\ang{135}}}{\left(s+\alpha-\beta j\right)} + \frac{\frac{1}{4\sqrt{2}}\times10^{-12}\angle{\ang{-135}}}{\left(s+\alpha+\beta j\right)}\right] \\ - &= \frac{1}{s} + \frac{\frac{1}{4\sqrt{2}}\angle{\ang{135}}}{\left(s+\alpha-\beta j\right)} + \frac{\frac{1}{4\sqrt{2}}\angle{\ang{-135}}}{\left(s+\alpha+\beta j\right)} + & \Rightarrow &V_o(s) &= V_{R^{'}}(s) + V_L(s)~~\mbox{avec}~~ V_L(s) = sLI_2(s)-LI_L(0^-)\\ + & \Leftrightarrow &V_o(s) &= RI_2(s) +sLI_2(s)-LI_L(0^-)\\ + & \Leftrightarrow &V_o(s) &= I_2(s)(R+sL) - LI_L(0^-) \\ + & \Leftrightarrow &I_2(R+sL) &= V_o(s) + LI_L(0^-) \end{align*} -La tension $V_o(t)$ est ensuite obtenu en appliquant la transformée de Laplace inverse à $V_o(s)$: -\[ v_o(t) = \left[1 + \frac{1}{2\sqrt{2}}\cdot e^{-10^6t}\cdot\cos\left(10^6t + \ang{135}\right)\right]u(t) \] - -On peut également écrire cette solution finale sous la forme d'une somme de sinus et de cosinus : -\[V_o(t) = \left[1 + \frac{1}{2} \cdot e^{-10^6t}\cdot\cos(10^6t) + \frac{1}{2} \cdot e^{-10^6t} \cdot\sin(10^6t)\right]u(t)\] +\begin{equation*} + \Rightarrow I_2(s)= \frac{V_o(s)+LI_L(0^-)}{R+sL} +\end{equation*} +Exprimons $I_1(s)$ en fonction de $V_o(s)$ : +\begin{equation*} + I_1(s) = \frac{V_o(s)+LI_L(0^-)+(R+sL)CV_C(0^-)}{(R+sL)(1+sRC)} +\end{equation*} +En effectuant une loi des mailles sur la maille globale nous pouvons écrire: +\begin{align*} + & \Rightarrow &\frac{2}{s} &= V_R(s) + V_o(s)\\ + & \Leftrightarrow &\frac{2}{s} &= RI_1(s)+V_o(s)\\ + & \Leftrightarrow &\frac{2}{s} &= R(\frac{V_o(s)+LI_L(0^-)+RCV_C(0^-)+sLCV_C(0^-)}{R+sR^2C+sL+s^2RLC})+V_o(s)\\ + & \Leftrightarrow &2R+2R^2Cs+2Ls+2RCLs^2 &= Rs(V_o(s)+LI_L(0^-)+RCV_C(0^-))+s^2RLCV_C(0^-)\\ + &&& +s(R+s(R^2C+L)+s^2RLC)V_o(s)\\ + & \Leftrightarrow &V_o(s)(2R+s(R^2C+L)+s^2RLC) &= 2R+s(2R^2C+2L-I_L(0^-)RL\\ + &&& -R^2CV_C(0^-))+s^2(2RLC-RLCV_C(0^-)) +\end{align*} +\begin{equation*} + \Rightarrow V_o(s)= \frac{2R+s(2R^2C+2L-RLI_L(0^-)-R^2CV_C(0^-))+s^2(2RCL-RLCV_C(0^-))}{s(2R+s(R^2C+L)+s^2RCL)} +\end{equation*} +En substituant les paramètres par leurs valeurs nous obtenons : +\begin{equation*} + V_o(s) = \frac{2000+3\cdot 10^{-3}s+ 1.5\cdot 10^{-9}s^2}{s(2000+2\cdot 10^{-3}s+10^{-9}s^2)}= 1.5\cdot \left[\frac{\frac{4}{3}10^{12}+2\cdot 10^{-6}s+s^2}{s(2\cdot 10^{12}+2\cdot 10^{6}s+s^2)}\right] +\end{equation*} +En effectuant la décomposition en fraction simple nous avons : +\begin{equation*} + V_C(s) = \frac{A}{s} + \frac{B\cdot 10^{6}}{(s+10^6)^2+10^{12}}+\frac{C(10^6+s)}{(s+10^{6})^2+10^{12}} \Leftrightarrow +\begin{dcases} +A+C =1.5\\ + 2\cdot 10^6 A+ 10^6B+10^6C = 3\cdot 10^6\\ + 2\cdot 10^{12}A = 2\cdot 10^{12}\\ +\end{dcases} +\Leftrightarrow +\begin{dcases} + A= 1\\ + B= \frac{1}{2}\\ + C=\frac{1}{2}\\ +\end{dcases} +\end{equation*} +Nous obtenons donc dans le domaine de Laplace : +\begin{equation*} + V_o(s) = \frac{1}{s} + \frac{1}{2} (\frac{10^6}{(s+10^6)^2+10^{12}}+ \frac{10^6+s}{(s+\cdot 10^{6})^2+10^{12}}) +\end{equation*} +Cela correspond dans le domaine temporel à : +\begin{equation*} + V_o(t) = \left[1+\frac{1}{2}\cdot e^{-10^6t}(\cos(10^6t)+\sin(10^6t))\right]u(t) +\end{equation*} +La solution équivalente avec seulement un cosinus est : +\begin{equation*} + V_o(t)= \left[1 + \frac{1}{2\sqrt{2}}\cdot e^{-10^6t}\cdot\cos\left(10^6t + 135^\circ \right)\right]u(t) +\end{equation*} +Le graphe ressemble à \begin{center} \begin{tikzpicture} - \begin{axis}[enlargelimits=true,grid=major,ylabel=$V_o(t)$,xlabel=$t$] + \begin{axis}[enlargelimits=true,grid=major,ylabel=$V_o(t)$,xlabel=$t(s)$] \addplot [blue,domain=0:0.00001,samples=200]{1+(1/2)*e^(-(10^6)*x)*cos(10^6*x)+(1/2)*e^(-10^6*x)*sin(10^6*x)}; \end{axis} \end{tikzpicture} \end{center} \end{solution} - - - - -\end{document} +\end{document} \ No newline at end of file diff --git a/src/q4/circmes-ELEC1370/exam/2018/2018.mk b/src/q4/circmes-ELEC1370/exam/2018/2018.mk new file mode 100644 index 000000000..7870b0cf7 --- /dev/null +++ b/src/q4/circmes-ELEC1370/exam/2018/2018.mk @@ -0,0 +1,2 @@ +YEAR=2018 +include ../../../exam.mk diff --git a/src/q4/circmes-ELEC1370/exam/2018/Juin/All/Makefile b/src/q4/circmes-ELEC1370/exam/2018/Juin/All/Makefile new file mode 100644 index 000000000..a519d4a83 --- /dev/null +++ b/src/q4/circmes-ELEC1370/exam/2018/Juin/All/Makefile @@ -0,0 +1,2 @@ +MINMAJ=All +include ../Juin.mk diff --git a/src/q4/circmes-ELEC1370/exam/2018/Juin/All/circmes-ELEC1370-exam-2018-Juin-All.tex b/src/q4/circmes-ELEC1370/exam/2018/Juin/All/circmes-ELEC1370-exam-2018-Juin-All.tex new file mode 100644 index 000000000..99340d83b --- /dev/null +++ b/src/q4/circmes-ELEC1370/exam/2018/Juin/All/circmes-ELEC1370-exam-2018-Juin-All.tex @@ -0,0 +1,627 @@ +\documentclass[fr]{../../../../../../eplexam} +\usepackage{../../../../../../eplunits} +\usepackage[oldvoltagedirection]{circuitikz} +\usepackage{bodegraph} +\usepackage{pgfplots} +\usepackage{amsmath} +\usepackage{enumitem} +\usepackage{xcolor} + + +\pgfplotsset{compat=newest} +\tikzset{meter/.style={draw,thick,circle,fill=white,minimum size =0.75cm,inner sep=0pt}} +\hypertitle{circmes-ELEC1370}{4}{ELEC}{1370}{2018}{Juin}{Majeure} +{Brieuc Balon} +{Claude Oestges, Bruno Dehez and Christophe Craeye} +\section{Question Oestges : phaseurs} +Soit le circuit suivant fonctionnant à la fréquence de $1kHz$: +\begin{center} + \begin{circuitikz} + \draw + (0,0)--(6,0) + (0,0) to [american controlled current source,l=$4I_x$](0,3) + to[R,l_=$1\Omega$,v^>=$V_x$](0,6) + (-1,6) to[short,o-](6,6) + (-1,3) to[short,o-](0,3) + to[R,l_=$1\Omega$](3,3) + to[R,l_=$1\Omega$,i>^=$I_o$](6,3) + (3,6) to [C,l^=$-j\Omega$](3,3) + (6,6)to[R,l_=$1\Omega$,i>^=$I_x$](6,3) + (3,0) to[american voltage source, l_=$4\angle 0^\circ$V](3,3) + (6,0) to[american controlled voltage source,l_=$2V_x$](6,3); + \end{circuitikz} +\end{center} +On demande: +\begin{enumerate} + \item Calculer le courant $I_o$ (amplitude et phase) + \item La puissance délivrée par la source indépendante + \item L'équivalent de Norton aux bornes de la résistance en haut à gauche (Question indépendante des autres) +\end{enumerate} +\begin{solution} +Pour la suite des calculs nous utiliserons les notations telles qu’illustrées : +\begin{center} + \begin{circuitikz} + \draw + (0,0)--(6,0) + (0,0) to [american controlled current source,l=$4I_x$](0,3) + to[R,l_=$1\Omega$,v^>=$V_x$,i_<=$I_1$](0,6) + (-1,6) to[short,o-](6,6) + (-1,3) to[short,o-*](0,3) + to[R,l_=$1\Omega$,i>_=$I_3$,v^=$V_3$](3,3) + to[R,l_=$1\Omega$,i_>=$I_o$,v^<=$V_o$](6,3) + (3,6) to [C,l^=$-j\Omega$,i<^=$I_2$,v>=$V_2$,*-](3,3) + (6,6)to[R,l_=$1\Omega$,i>_=$I_x$,v^<=$V_{x^{'}}$](6,3) + (3,0) to[american voltage source, l_=$4\angle 0^\circ$V,i>_=$I_S$](3,3) + (6,0) to[american controlled voltage source,l_=$2V_x$](6,3) + (3,6) node[above,color = orange]{$A$}{} + (0,3)node[below left,color = teal]{$B$}{} + %Flèches + (4.7,4.5) node[scale=2,color=brown]{$\circlearrowright$}{} + (1.3,4.5)node[scale=2,color=red]{$\circlearrowright$}{} + (4.8,1.5)node[scale=2,color=blue]{$\circlearrowright$}{}; + \end{circuitikz} +\end{center} +\begin{enumerate} + \item Nous pouvons effectuer 3 lois des mailles sur les mailles indiquées et 2 lois des noeuds sur les 2 noeuds indiqués : +\begin{equation*} + \begin{dcases} + \textcolor{teal}{0 = I_1-I_3+4I_x}\\ + \textcolor{orange}{I_2 = I_1+I_x}\\ + \textcolor{red}{jI_2 = I_1+I_3}\\ + \textcolor{brown}{I_x = I_o+jI_2}\\ + \textcolor{blue}{4 = I_o +2V_x}\\ + V_x = I_1\\ + \end{dcases} + \Leftrightarrow + \begin{bmatrix} + 0 & 1 & 0 &-1 & 4 \\ + 0 & 1 & -1 & 0 & 1\\ + 0 & 1 & -j & 1 & 0\\ + 1 & 0 & j & 0 & -1\\ + 1 & 2 & 0 & 0 & 0\\ + \end{bmatrix} + \begin{bmatrix} + I_o\\ + I_1\\ + I_2\\ + I_3\\ + I_x\\ + \end{bmatrix} + = + \begin{bmatrix} + 0\\ + 0\\ + 0\\ + 0\\ + 4\\ + \end{bmatrix} + \Leftrightarrow + \begin{dcases} + I_o = \frac{4}{3}\angle -126.869^\circ [A]\\ + I_1 = 2.45\angle 12.529^\circ [A]\\ + I_2 = 1.1925\angle 26.565^\circ [A]\\ + I_3 = 2.9814\angle 169.695^\circ [A]\\ + I_x = \frac{4}{3} \angle 180^\circ [A]\\ + \end{dcases} +\end{equation*} +Le courant $I_o=\frac{4}{3}\angle -126.869^\circ$ [A] +\item La puissance\footnote{Les phaseurs expriment la valeur efficace de la tension/courant qu'ils représentent.} délivrée par la source indépendante peux s'écrire comme : +\begin{align*} + P &= \Re(V\cdot I_S^\ast) ~~\mbox{avec}~~I_S = I_0+I_2-I_3 = 3.373\angle -18.43^\circ [A]\\ + &=\Re(4\angle 0^\circ \cdot 3.373\angle 18.43^\circ) = 12.8[W] +\end{align*} + +\item Pour calculer l'équivalent de Norton aux bornes de la résistance, nous allons calculer le courant $I_N$ et la tension $V_{Th}$\footnote{Étant donné que nous avons des sources dépendantes nous ne pouvons pas calculer la résistance équivalente $Z_{eq}$.}. \\ +Pour calculer le courant $I_N$, le circuit peut être représenté comme : +\begin{center} + \begin{circuitikz} + \draw + (0,0)--(6,0) + (0,0) to [american controlled current source,l=$4I_x$](0,3) + to[short,i<_=$I_N$](0,6) + (-1,6) to[short,o-](6,6) + (-1,3) to[short,o-*](0,3) + to[R,l_=$1\Omega$,i>_=$I_3$,v^=$V_3$](3,3) + to[R,l_=$1\Omega$,i_>=$I_o$,v^<=$V_o$](6,3) + (3,6) to [C,l^=$-j\Omega$,i<^=$I_2$,v>=$V_2$,*-](3,3) + (6,6)to[R,l_=$1\Omega$,i>_=$I_x$,v^<=$V_x$](6,3) + (3,0) to[american voltage source, l^=$4\angle 0^\circ$V,i>_=$I_S$](3,3) + (6,0) --(6,3) + (3,6) node[above,color = orange]{$A$}{} + (0,3)node[below left,color = teal]{$B$}{} + %Flèches + (4.7,4.5) node[scale=2,color=brown]{$\circlearrowright$}{} + (1.3,4.5)node[scale=2,color=red]{$\circlearrowright$}{} + (4.5,1.5)node[scale=2,color=blue]{$\circlearrowright$}{}; + \end{circuitikz} +\end{center} +Nous pouvons effectuer 3 lois des mailles sur les mailles indiquées et 2 lois des noeuds sur les 2 noeuds indiqués : +\begin{equation*} + \begin{dcases} + \textcolor{teal}{I_3 = I_N+4I_x}\\ + \textcolor{orange}{I_2 = I_N+I_x}\\ + \textcolor{red}{I_3=jI_2}\\ + \textcolor{brown}{I_o =I_x-jI_2}\\ + \textcolor{blue}{I_o = 4\angle 0^\circ}\\ + \end{dcases} + \Leftrightarrow + \begin{bmatrix} + 0 & 1 & -4 & -1 \\ + 1 & 0 & -1 & -1 \\ + -j & 1 & 0 & 0 \\ + -j & 0 & 1 & 0 \\ + \end{bmatrix} + \begin{bmatrix} + I_2\\ + I_3\\ + I_x\\ + I_N\\ + \end{bmatrix} + = + \begin{bmatrix} + 0\\ + 0\\ + 0\\ + 4\\ + \end{bmatrix} + \Leftrightarrow + \begin{dcases} + I_2 = 5.366\angle 116.565^\circ [A]\\ + I_3 = 5.366\angle -153.435^\circ [A]\\ + I_x = 2.529\angle -108.435^\circ [A]\\ + I_N = 7.375\angle 102.529^\circ [A]\\ + \end{dcases} +\end{equation*} +\begin{equation*} + I_N = 7.375\angle 102.529^\circ = -\frac{8}{5}+\frac{36}{5}j [A] +\end{equation*} +Pour calculer la tension $V_{Th}$, le circuit peut être représenté comme : +\begin{center} + \begin{circuitikz} + \draw + (0,0)--(3,0)to[short,i=$I_x$](6,0) + (0,0) to [american controlled current source,l=$4I_x$](0,3) + (-1,3) to[open,v^>=$V_{Th}$](-1,6) + (-1,6) to[short,o-](6,6) + (-1,3) to[short,o-*](0,3) + to[R,l_=$1\Omega$,v^=$V_3$](3,3) + to[R,l_=$1\Omega$,i_>=$I_o$,v^<=$V_o$](6,3) + (3,6) to [C,l^=$-j\Omega$,v>=$V_2$,*-](3,3) + (6,6)to[R,l_=$1\Omega$,v^<=$V_{x^{'}}$](6,3) + (3,0) to[american voltage source, l^=$4\angle 0^\circ$V,i>_=$I_S$](3,3) + (6,0) to[american controlled voltage source,l_=$2V_{Th}$](6,3) + %Flèches + (4.7,4.5) node[scale=2,color=brown]{$\circlearrowright$}{} + (4.5,1.5)node[scale=2,color=blue]{$\circlearrowright$}{}; + \end{circuitikz} +\end{center} +On peut remarquer que : +\begin{equation*} + V_x = -(4I_x -jI_x)=I_x(j-4) +\end{equation*} +Nous pouvons effectuer 2 lois des mailles sur les mailles indiquées : +\begin{equation*} + \begin{dcases} + \textcolor{brown}{I_o = I_x(1-j)}\\ + \textcolor{blue}{4=I_o+2V_{Th}}\\ + \end{dcases} + \Leftrightarrow + \begin{dcases} + \textcolor{brown}{0 = I_o +I_x (j-1)}\\ + \textcolor{blue}{4= I_o+2(j-4)I_x}\\ + \end{dcases} + \Leftrightarrow + \begin{bmatrix} + 1 & j-1 \\ + 1 & 2(j-4)\\ + \end{bmatrix} + \begin{bmatrix} + I_o\\ + I_x\\ + \end{bmatrix} + = + \begin{bmatrix} + 0\\ + 4\\ + \end{bmatrix} + \Leftrightarrow + \begin{dcases} + I_o = \frac{4}{5}\angle 143.13^\circ [A]\\ + I_x = 0.565\angle -171.87^\circ [A]\\ + \end{dcases} +\end{equation*} +\begin{equation*} + V_{Th} =I_x(j-4) = 2.33\angle -5.906^\circ = \frac{58}{25}-\frac{6}{25}j[V] +\end{equation*} +Nous pouvons maintenant déterminer l'impédance équivalente $Z_{eq}$ : +\begin{equation*} + Z_{eq} = \frac{V_{Th}}{I_N} = \frac{\frac{58}{25}-\frac{6}{25}j}{ -\frac{8}{5}+\frac{36}{5}j} =-(\frac{1}{10}+\frac{3}{10}j)=0.316\angle -108.435^\circ [\Omega] +\end{equation*} +Le circuit équivalent de Norton aux bornes de la résistance peut-être représenté comme : +\begin{center} + \begin{circuitikz} + \draw + (0,0) to [american current source,l=$I_N$] (0,3)--(1,3) + (1,3) to[european resistor,l=$Z_{eq}$](1,0) + (0,0) to[short,-o](2,0) + (1,3)to[short,-o](2,3); + \end{circuitikz} +\end{center} +\end{enumerate} +\end{solution} +\section{Question Oestges : quadripôles} +Soit le circuit suivant avec l'amplificateur opérationnel idéal: +\begin{center} + \begin{circuitikz} + \draw + (6,-0.5) node[op amp] (opamp) {} + (0,0) node[above]{$V_i$}{} to[R, l_=$1k\Omega$,o-](3,0) + to[R,l_=$500\Omega$] (3,-2) node[ground]{} + (3,0) to [C,l=$0.1\mu F$](3,3) + (3,0) to [C,l_=$0.1\mu F$](opamp.-) + (opamp.+) to[short] (4.5,-1)--(4.5,-1.5)node[ground]{} + (4.5,0) to[R,l_=$10k\Omega$] (4.5,3) + (3,3)--(7,3)--(7,-0.5)--(opamp.out) + (7,-0.5) to[short,-o] (8,-0.5)node[above]{$V_o$}{}; + \end{circuitikz} +\end{center} +Déterminer $Z_{in}$ si on met un charge $z_L=1k\Omega$ à la sortie de l'amplificateur opérationnel avec : $R_1 = 1k\Omega$, $R_2=500\Omega$, $R_3=10k\Omega$ et $C=0.1\mu F$. +\begin{solution} +Pour déterminer $Z_{in}$, nous allons utiliser la représentation quadripôle Z. La matrice $Z$ est calculée grâce à : +\begin{equation*} + \begin{bmatrix} + V_i \\ + V_o\\ + \end{bmatrix} = + \begin{bmatrix} + z_i & z_r \\ + z_f & z_o\\ + \end{bmatrix} + \begin{bmatrix} + i_i \\ + i_o\\ + \end{bmatrix} +\end{equation*} +En annulant le courant $i_i$ le circuit devient : +\begin{center} + \begin{circuitikz} + \draw + (6,-0.5) node[op amp] (opamp) {} + (0,0) node[above]{$V_i$}{} to[R, l_=$1k\Omega$,o-](3,0) + to[R,l_=$500\Omega$] (3,-2) node[ground]{} + (3,0) to [C,l=$0.1\mu F$](3,3) + (3,0) to [C,l_=$0.1\mu F$](opamp.-) + (opamp.+) to[short] (4.5,-1)--(4.5,-1.5)node[ground]{} + (4.5,0) to[R,l_=$10k\Omega$] (4.5,3) + (3,3)--(7,3)--(7,-0.5)--(opamp.out) + (7,-0.5) to[short,i<_=$i_o$,-o] (8,-0.5)node[above]{$V_o$}{}; + \end{circuitikz} +\end{center} +Comme nous avons un amplificateur opérationnel que l'on considère comme idéal, son impédance d'entrée vaut $0\Omega$. Le courant $i_0$ passe donc entièrement dans l'amplificateur opérationnel et aucun courant ne passe dans le reste du circuit. Dès lors tout le circuit est à 0V. Nous obtenons donc : +\begin{equation*} +\left \{ + \begin{array}{rcl} + V_i &=& 0 \cdot i_i \\ + V_o &=& 0 \cdot i_o \\ +\end{array} +\right. +\Leftrightarrow +\left \{ + \begin{array}{rcl} + z_r &=& 0 [\Omega]\\ + z_o &=& 0 [\Omega]\\ +\end{array} +\right. +\end{equation*} +En annulant le courant $i_o$ le circuit devient : +\begin{center} + \begin{circuitikz} + \draw + (6,-0.5) node[op amp] (opamp) {} + (0,0) node[above]{$V_i$}{} to[R, l_=$R_1$,i>_=$i_i$,v^=$ $,o-*](3,0) node[above left]{$V_x$}{} + to[R,l_=$R_2$,v^=$ $] (3,-2) node[ground]{} + (3,0) to [C,l=$C$,v_=$ $](3,3) + (3,0) to [C,l_=$C$,v^=$ $](opamp.-) + (opamp.+) to[short] (4.5,-1)--(4.5,-1.5)node[ground]{} + (4.5,0)node[below]{$V^-$}{} to[R,l_=$R_3$,v^=$ $,*-] (4.5,3) + (3,3)--(7,3)--(7,-0.5)--(opamp.out) + (7,-0.5) to[short,-o] (8,-0.5)node[above]{$V_o$}{}; + \end{circuitikz} +\end{center} +Comme l'amplificateur opérationnel est idéal et en rétroaction négative, on sait que $V^+=V^-=0$V. Dès lors: +\begin{equation*} + (V_x-0)j\omega C=\frac{0-V_{o}}{R_3Cj\omega} \Leftrightarrow V_x=-\frac{V_{o}}{R_3Cj\omega} +\end{equation*} +Nous pouvons faire une loi des noeuds au niveau du noeud $V_x$ : +\begin{align*} + i_i &= (V_x-V_o)k\omega C+(V_x-0)j\omega C+\frac{V_x-0}{R_2}\\ + \Leftrightarrow i_i &= V_x(2j\omega C+\frac{1}{R_2})-V_oj\omega C\\ + \Leftrightarrow i_i &= -V_o(\frac{1}{R_3Cj\omega}(2j\omega C+\frac{1}{R_2})+j\omega C)\\ + \Leftrightarrow i_i &= - V_o \left[ \frac{1+2j\omega CR_2+R_2R_3C^2(j\omega)^2}{R_2R_3Cj\omega}\right]\\ + \Leftrightarrow V_o &= -\frac{CR_2R_3j\omega}{1+2R_2Cj\omega+R_2R_3C^2(j\omega)^2}i_i \\ + \Rightarrow z_f &= -\frac{CR_2R_3j\omega}{1+2R_2Cj\omega+R_2R_3C^2(j\omega)^2} [\Omega] +\end{align*} +Nous pouvons calculer $V_i$ selon : +\begin{align*} + V_i &= R_1i_i+V_x\\ + \Leftrightarrow V_i &= R_1i_i -\frac{V_o}{R_3Cj\omega}\\ + \Leftrightarrow V_i &= R_1i_i+ i_i \frac{R_2}{1+2R_2Cj\omega+R_2R_3C^2(j\omega)^2}\\ + \Leftrightarrow V_i &= i_i \frac{R_1+R_2+2R_1R_2Cj\omega+R_1R_2R_3C^2(j\omega)^2}{1+2R_2Cj\omega+R_2R_3C^2(j\omega)^2}\\ + \Rightarrow z_i &= \frac{R_1+R_2+2R_1R_2Cj\omega+R_1R_2R_3C^2(j\omega)^2}{1+2R_2Cj\omega+R_2R_3C^2(j\omega)^2}[\Omega] +\end{align*} +La matrice Z est donc : +\begin{equation*} + Z = \begin{bmatrix} + \frac{R_1+R_2+2R_1R_2Cj\omega+R_1R_2R_3C^2(j\omega)^2}{1+2R_2Cj\omega+R_2R_3C^2(j\omega)^2} & 0 \\ + -\frac{CR_2R_3j\omega}{1+2R_2Cj\omega+R_2R_3C^2(j\omega)^2} & 0\\ + \end{bmatrix} +\end{equation*} +Grâce à la table des caractéristiques externes nous pouvons calculer l'impédance d'entrée $Z_{in}$ : +\begin{align*} + Z_{in} &= z_i\left(1+\frac{z_r}{z_i}A_{if,o}\right)~~\mbox{avec}~A_{if,o} =-\frac{z_f}{z_o+z_L}\\ + &= z_i(1+\frac{0}{z_i}\cdot-\frac{z_f}{z_l+0})\\ + &=z_i\\ + \Rightarrow Z_{in} &= \frac{R_1+R_2+2R_1R_2Cj\omega+R_1R_2R_3C^2(j\omega)^2}{1+2R_2Cj\omega+R_2R_3C^2(j\omega)^2} [\Omega]\\ +\end{align*} +\end{solution} +\section{Question Dehez : circuit magnétique couplé} +Soit le circuit suivant : +\begin{center} + \begin{circuitikz} + \draw + (0,0.45) to [american voltage source,l^=$12\angle 0^\circ$](0,2.55) + to[R,l_=$3\Omega$](3,2.55) + (4,1.5) node[transformer] (T) {} + (T.outer dot A1) node[circ]{} + (T.outer dot B1) node[circ]{} + (T-L1.south) node[left]{$3j\Omega$}{} + (T-L2.south) node[right]{$3j\Omega$}{} + (3,0.45) --(0,0.45) + (4.5,0.45)--(9.5,0.45) + (7,0.45)to[C,l=$-6j\Omega$](7,1) + (5.5,1)--(8,1) + to[R,l_=$6\Omega$] (8,2.55) + (5.5,1) to [R,l_=$6\Omega$] (5.5,2.55)--(5,2.55) + (5.5,2.55) to[R,l_=$6\Omega$] (8,2.55) --(9.5,2.55)to[C,l=$-6j\Omega$](9.5,0.45); + \draw[<->,>=stealth] ($(3.6,2.7)$) to [bend left] node[pos=0.5,above] {$3j\Omega$} ($(4.4,2.7)$); + \end{circuitikz} +\end{center} +On demande : +\begin{enumerate} + \item Calculer le facteur de dispersion. Expliquer ce que signifie ce résultat. + \item Calculé le courant délivré par la source de tension. +\end{enumerate} +On souhaite mesure le courant délivré par la source à l'aide d'un ampèremètre. Cependant, l'ampèremètre possède un fond d'échelle de 100[mA]. On utilise un transformateur afin de pallier à ce problème. +\begin{center} + \begin{circuitikz} + \draw + (-3,0.45) to [american voltage source,l^=$12\angle 0^\circ$](-3,2.55) to[L](0,2.55) + to[R,l_=$3\Omega$](3,2.55) + (4,1.5) node[transformer] (T) {} + (T.outer dot A1) node[circ]{} + (T.outer dot B1) node[circ]{} + (T-L1.south) node[left]{$3j\Omega$}{} + (T-L2.south) node[right]{$3j\Omega$}{} + (3,0.45) --(-3,0.45) + (4.5,0.45)--(9.5,0.45) + (7,0.45)to[C,l=$-6j\Omega$](7,1) + (5.5,1)--(8,1) + to[R,l_=$6\Omega$] (8,2.55) + (5.5,1) to [R,l_=$6\Omega$] (5.5,2.55)--(5,2.55) + (5.5,2.55) to[R,l_=$6\Omega$] (8,2.55) --(9.5,2.55)to[C,l=$-6j\Omega$](9.5,0.45) + (-3,3.5)to[L](0,3.5)--(0,4.5) to node[meter]{\small A}(-3,4.5)--(-3,3.5) + (-2,3)--(-1,3) + (-2,3.1)--(-1,3.1); + \draw (-2.3,2.5)node[below right]{$\bullet$}; + \draw (-2.3,4)node[below right]{$\bullet$}; + \draw[white] (-3,3) to[] node[black,pos=0.5,rotate = 90] {$n:1$} (-3,3.1); + \draw[<->,>=stealth] ($(3.6,2.7)$) to [bend left] node[pos=0.5,above] {$3j\Omega$} ($(4.4,2.7)$); + \end{circuitikz} +\end{center} +\begin{enumerate}[resume] + \item Calculer la valeur idéale que devrait prendre le rapport de transformation $n$ de ce transformateur pour avoir une mesure correspondant au fond d’échelle de l'ampèremètre. +\end{enumerate} +\begin{solution} +\begin{enumerate} +\item Le facteur de dispersion s’écrit comme +\begin{equation*} + \sigma = 1 -\frac{M^2}{L_1L_2} = 1 - \frac{(3j)^2}{3j\cdot 3j} = 0 +\end{equation*} +Le couplage magnétique est parfait, il n'y a aucun flux de fuite. +\item Nous allons modifier le circuit en utilisant la dualité étoile-triangle : +\begin{center} + \begin{circuitikz} + \draw + (0,0.45) to [american voltage source,l^=$12\angle 0^\circ$](0,2.55) + to[R,l_=$3\Omega$](3,2.55) + (4,1.5) node[transformer] (T) {} + (T.outer dot A1) node[circ]{} + (T.outer dot B1) node[circ]{} + (T-L1.south) node[left]{$3j\Omega$}{} + (T-L2.south) node[right]{$3j\Omega$}{} + (3,0.45) --(0,0.45) + (5,2.55)--(8,2.55) + to[R,l^=$R_a$](8,1) + to[R,l_=$R_b$](7,0)to[C,l_=$-6j\Omega$](6,-1) + (8,1) to[R,l^=$R_c$](9,0) to[C,l^=$-6j\Omega$](10,-1)--(5.05,-1)--(5.05,0.45); + \draw[<->,>=stealth] ($(3.6,2.7)$) to [bend left] node[pos=0.5,above] {$3j\Omega$} ($(4.4,2.7)$); + \end{circuitikz} +\end{center} +\begin{equation*} + R_a=R_b=R_c=\frac{6\cdot6}{6+6+6}=2[\Omega] +\end{equation*} +On remarque que les 2 branches du bas de l'étoile sont en parallèle. Dès lors: +\begin{equation*} + Z=(2-6j)||(2-6j)=\left(\frac{1}{2-6j}+\frac{1}{2-6j}\right)^{-1}=1-3j[\Omega] +\end{equation*} +Le circuit devient : +\begin{center} + \begin{circuitikz} + \draw + (0,0.45) to [american voltage source,l^=$12\angle 0^\circ$](0,2.55) + to[R,l_=$3\Omega$,i>_=$I_1$,v^<=$ $](3,2.55) + (4,1.5) node[transformer] (T) {} + (T.outer dot A1) node[circ]{} + (T.outer dot B1) node[circ]{} + (T-L1.south) node[left]{$3j\Omega$}{} + (T-L2.south) node[right]{$3j\Omega$}{} + (3,0.45) --(0,0.45) + (5.05,0.45) to[european resistor,l_=$1-3j$,i>_=$I_2$,v^<=$ $](7,0.45) + to[R,l_=$2\Omega$,v^<=$ $](7,2.55)--(5.05,2.55); + \draw[<->,>=stealth] ($(3.6,2.7)$) to [bend left] node[pos=0.5,above] {$3j\Omega$} ($(4.4,2.7)$); + \end{circuitikz} +\end{center} +Nous pouvons effectuer 2 lois des mailles sur les 2 uniques mailles du circuit : +\begin{equation*} +\left \{ + \begin{array}{rcl} + 12 &=& (3+3j)I_1 +3jI_2 \\ + 0 &=& 3jI_1 +3I_2 \\ +\end{array} +\right. +\Leftrightarrow +\left \{ + \begin{array}{rcl} + I_1 &=& \frac{8}{5} -\frac{4}{5}j =1.788\angle -26.565^\circ [A]\\ + I_2 &=& -\frac{4}{5} -\frac{8}{5}j =1.788\angle -116.565^\circ [A]\\ +\end{array} +\right. +\end{equation*} +\item Le fond d'échelle correspond à la valeur maximale que peut 'lire' l'appareil de mesure. Il faut donc que le courant mesurée par l'ampèremètre ne dépasse pas 100ma. La formule qui lie le courant et le nombre de spires est donnée par : +\begin{equation*} + \frac{I_{primaire}}{I_{secondaire}} = \pm \frac{N_{secondaire}}{N_{primaire}} + \Leftrightarrow \frac{I_1}{I_{ampèremètre}}= \frac{1}{n} ~~\mbox{avec}~~I_{ampèremètre}< 0.1[A] + \end{equation*} + \begin{equation*} + \Leftrightarrow I_{1} \cdot n< 0.1 = I_{ampèremètre} \Leftrightarrow n <\frac{0.1}{I_{1}} = \frac{0.1}{1.788} = 0.056 + \end{equation*} +Il faut que le nombre de spires soit inférieur à 0.056. Il est donc impossible d'avoir un courant inférieur à 0.1[A] dans le circuit secondaire. Si l'on avait inversé le rapport c'est-à-dire $N_1=1$ et $N_2=n$ nous aurions pu trouvé un nombre minimum de spires égale à 17.8. +\end{enumerate} +\end{solution} +\section{Question Craeye : transitoire} +Soit le circuit suivant dont l'interrupteur change de position en $t=0$. +\begin{center} + \begin{circuitikz} + \draw (0,3) node[ground]{} to[american voltage source,l=$2V$] (0,6) to[short,-o] (1,6) coordinate (B); + \draw (2,3) node[ground]{}to[american voltage source,l=$1V$] (2,4) to[short,-o] (2,5) coordinate (A); + \draw (1.4,5.4) to[short,o-] (2,6); + \draw + (8,5.5) node[op amp] (opamp) {} + (2,6) to[L,l_=$L$] ++(3,0) to[R,l_=$R$](opamp.-) + (opamp.-) to[short,*-] ++(0,2.5) + to[R,l^=$R$]++(3,0) to [short]++(0,-3) + (opamp.+)--(6.8,4.5)node[ground]{} + (6.8,7)to[C,l_=$C$](9.8,7) + (9.8,5.5) to [R,l^=$R$](9.8,3)node[ground]{} + (opamp.out)to[short,-*](9.8,5.5) to [short,-o] (11,5.5)node[above]{$V_o$}{}; + \draw [->,>=stealth] ($ (A) + (-0.2,0) $) to [bend left] node[pos=0.5,below left] {$t=0$} ($ (B) + (0,-0.2) $); + \draw (A) node[right]{$A$}; + \draw (B) node[above]{$B$}; + \end{circuitikz} +\end{center} +On demande la tension $V_o(t)$ avec les données numériques suivantes: +$R=1k\Omega$, $L=1mH$, $C =1nF$ +\begin{solution} +En $t<0$, on peut réécrire le circuit comme : +\begin{center} + \begin{circuitikz} + \draw (2,3) node[ground]{}to[american voltage source,l=$1V$] (2,6) + (8,5.5) node[op amp] (opamp) {} + (2,6) to[short,i>_=$I_L(0^-)$] ++(3,0) to[R,l_=$R$,v^=$ $](opamp.-) + (opamp.-) to[short,*-] ++(0,2.5) + to[R,l^=$R$,v_=$ $]++(3,0) to [short]++(0,-3) + (opamp.+)--(6.8,4.5)node[ground]{} + (6.8,7)to[open,v^=$V_C(0^-)$,*-*](9.8,7) + (9.8,5.5) to [R,l^=$R$](9.8,3)node[ground]{} + (opamp.out)to[short,-*](9.8,5.5) to [short,-o] (11,5.5)node[above]{$V_o$}{}; + \end{circuitikz} +\end{center} +Étant donner que l'amplificateur opérationnel est considéré comme parfait et est en rétroaction négative ($V^+=V^-=0V$) nous obtenons: +\begin{equation*} + \left \{ +\begin{array}{rcl} +I_L(0^-) &=& \frac{1}{R}=10^{-3} [A]\\ +V_{C}(0^-) &=& R I_L(0^-)=1[V]\\ +\end{array} +\right. +\end{equation*} +En $t>0$, on peut réécrire le circuit comme : +\begin{center} + \begin{circuitikz} + \draw + (0,3) node[ground]{} to[american voltage source,l=$2V$] (0,6) to[short] (2,6) + (8,5.5) node[op amp] (opamp) {} + (2,6) to[L,l_=$L$,i>_=$I_1(s)$,v^=$V_L(s)$] ++(3,0) to[R,l_=$R$,v^=$V_R(s)$](opamp.-) + (opamp.-) to[short,*-] ++(0,3.5) + to[R,l_=$R$,i>_=$I_2(s)$,v^=$V_{R^{'}}(s)$]++(3,0) to [short]++(0,-4) + (opamp.+)--(6.8,4.5)node[ground]{} + (6.8,7)to[C,l_=$C$,v^=$V_C(s)$](9.8,7) + (9.8,5.5) to [R,l^=$R$](9.8,3)node[ground]{} + (opamp.out)to[short,-*](9.8,5.5) to [short,-o] (11,5.5)node[above]{$V_o(s)$}{}; + \end{circuitikz} +\end{center} +On peut remarquer que la résistance en bas à droite n'a pas d'impact sur la tension $V_o(s)$. +Nous pouvons effectuer une loi des mailles sur la maille de gauche: +\begin{align*} + \frac{2}{s} &= V_L(s) + V_R(s) ~~\mbox{avec}~~V_L(s) = sLI_1(s) -LI_L(0^-)\\ + \Leftrightarrow \frac{2}{s} &= sLI_1(s) -LI_L(0^-) + RI_1(s)\\ + \Leftrightarrow \frac{2}{s} &= I_1(s)(R+sL) -LI_L(0^-) +\end{align*} +\begin{equation*} + \Rightarrow I_1(s) = \frac{2+sLI_L(0^-)}{s(R+sL)} +\end{equation*} +Nous pouvons effectuer une loi des mailles sur la maille tout en haut : +\begin{align*} + V_C(s)&=V_{R{'}}\\ + \Leftrightarrow V_C(s) &= RI_2(s)\\ + \Leftrightarrow I_2(s) &= \frac{V_C(s)}{R} +\end{align*} +Sachant que $ V_C(s) = \frac{I_1(s)-I_2(s)}{Cs}+\frac{V_C(0^-)}{s}$ nous pouvons calculer : +\begin{align*} + V_C(s) &= \frac{I_1(s)-I_2(s)}{Cs}+\frac{V_C(0^-)}{s} \\ + \Leftrightarrow V_C(s) &= \frac{2+sLI_L(0^-)}{s^2C(R+sL)}-\frac{V_C(s)}{sRC}+V_C(0^-)\\ + \Leftrightarrow V_C(s) (1+\frac{1}{sRC}) &= \frac{2+sLI_L(0^-)+sC(R+sL)V_C(0^-)}{s^2C(R+sL)}\\ + \Leftrightarrow V_C(s) &= \frac{2+s(LI_L(0^-)+CRV_C(0^-))+s^2LRCV_C(0^-)}{s^2C(R+sL)}\frac{RsC}{1+sRC}\\ + \Leftrightarrow V_C(s) &= \frac{2R+Rs(LI_L(0^-)+CRV_C(0^-))+s^2LRCV_C(0^-)}{s(R+s(R^2C+L)+s^2RCL)}\\ +\end{align*} +Sachant que $V_C(s) = -V_o(s)$ : +\begin{equation*} + V_o(s) =- \frac{2+s(LI_L(0^-)+RCV_C(0^-))+s^2LCV_C(0^-)}{s(1+s(RC+\frac{L}{R})+s^2LC)} +\end{equation*} +En substituant les paramètres par leurs valeurs nous obtenons : +\begin{equation*} + V_o(s) = -\frac{2+2\cdot 10^{-3}s+10^{-12}s^2}{s(1+2\cdot 10^{-6}+10^{-12}s^2)}=-\frac{2\cdot 10^{12}+2\cdot 10^6+s^2}{s(10^{12}+2\cdot 10^6+s^2)}=-\frac{2\cdot 10^{12}+2\cdot 10^6+s^2}{s(s+10^6)^2} +\end{equation*} +En effectuant la décomposition en fraction simple nous avons : +\begin{equation*} + V_C(s) = \frac{A}{s} + \frac{B}{s+10^6}+\frac{C}{(s+10^{6})^2} \Leftrightarrow +\begin{dcases} + A +B =-1\\ + 2\cdot 10^6 A+ 10^6B+C = -2\cdot 10^6\\ + 10^{12}A = -2 \cdot 10^{12}\\ +\end{dcases} +\Leftrightarrow +\begin{dcases} + A= -2\\ + B= 1\\ + C=10^{6}\\ +\end{dcases} +\end{equation*} +Nous obtenons donc dans le domaine de Laplace : +\begin{equation*} + V_o(s) = -\frac{2}{s} +\frac{1}{s+10^6}+ \frac{10^6}{(s+\cdot 10^{6})^2}) +\end{equation*} +Cela correspond dans le domaine temporel à : +\begin{equation*} + V_o(t) = \left[-2+e^{-10^6t}(1+10^{6}t)\right]u(t) +\end{equation*} +Le graphe ressemble à +\begin{center} + \begin{tikzpicture} + \begin{axis}[enlargelimits=true,grid=major,ylabel=$V_o(t)$,xlabel=$t(s)$] + \addplot [blue,domain=0:0.00001,samples=200]{-2+e^(-1*10^6*x)*(1+10^6*x)}; + \end{axis} + \end{tikzpicture} +\end{center} +\end{solution} +\end{document} \ No newline at end of file diff --git a/src/q4/circmes-ELEC1370/exam/2018/Juin/Juin.mk b/src/q4/circmes-ELEC1370/exam/2018/Juin/Juin.mk new file mode 100644 index 000000000..491eba8f4 --- /dev/null +++ b/src/q4/circmes-ELEC1370/exam/2018/Juin/Juin.mk @@ -0,0 +1,2 @@ +MONTH=Juin +include ../../2018.mk diff --git a/src/q4/circmes-ELEC1370/exam/2020/2020.mk b/src/q4/circmes-ELEC1370/exam/2020/2020.mk new file mode 100644 index 000000000..d209b81a7 --- /dev/null +++ b/src/q4/circmes-ELEC1370/exam/2020/2020.mk @@ -0,0 +1,2 @@ +YEAR=2020 +include ../../../exam.mk diff --git a/src/q4/circmes-ELEC1370/exam/2020/Juin/All/Makefile b/src/q4/circmes-ELEC1370/exam/2020/Juin/All/Makefile new file mode 100644 index 000000000..a519d4a83 --- /dev/null +++ b/src/q4/circmes-ELEC1370/exam/2020/Juin/All/Makefile @@ -0,0 +1,2 @@ +MINMAJ=All +include ../Juin.mk diff --git a/src/q4/circmes-ELEC1370/exam/2020/Juin/All/circmes-ELEC1370-exam-2020-Juin-All.tex b/src/q4/circmes-ELEC1370/exam/2020/Juin/All/circmes-ELEC1370-exam-2020-Juin-All.tex new file mode 100644 index 000000000..aaaee1e50 --- /dev/null +++ b/src/q4/circmes-ELEC1370/exam/2020/Juin/All/circmes-ELEC1370-exam-2020-Juin-All.tex @@ -0,0 +1,589 @@ +\documentclass[fr]{../../../../../../eplexam} +\usepackage{../../../../../../eplunits} +\usepackage[oldvoltagedirection]{circuitikz} +\usepackage{bodegraph} +\usepackage{pgfplots} +\usepackage{amsmath} +\usepackage{enumitem} +\pgfplotsset{compat=newest} +\tikzset{meter/.style={draw,thick,circle,fill=white,minimum size =0.75cm,inner sep=0pt}} +\hypertitle{circmes-ELEC1370}{4}{ELEC}{1370}{2020}{Juin}{Maj} +{Brieuc Balon \and Ottilie Bonfanti} +{Claude Oestges, Bruno Dehez and Christophe Craeye} + +\section{Question Oestges : Bode} +On considère le circuit de filtrage suivant où l'amplificateur opérationnel est considéré idéal. Les composants ont les valeurs suivantes : $R_1 = 10k\Omega$, $R_2 = 20k\Omega$, $R_3=20k\Omega$, $C_1=10nF$. +\begin{center} + \begin{circuitikz} + \draw + (5,3) node[op amp] (opamp) {} + (opamp.-) --(3.8,4.5) + to[R,l^=$R_3$](7.5,4.5)--(7.5,3) + (opamp.out) to [short,-o](8,3) node[right]{$V_{out}$}{} + (1,4.5) node[left]{$V_{in}$} to [R,l^=$R_2$,o-](4.5,4.5) + (1.5,4.5) --(1.5,2.5) + to[C,l_=$C_1$] (opamp.+) + (3.5,2.5) to [R,l^=$R_1$](3.5,0)node[ground]{}; + \end{circuitikz} +\end{center} +\begin{enumerate} + \item On veut déterminer l'expression analytique (sans remplacer par les valeurs) du gain en tension $G(\omega)=\frac{V_{in}}{V_{out}}$. Dans cette première sous-question, indiquez les équations des mailles et des noeuds nécessaires à la résolution du problème. + \item Quelle est l'expression analytique (sans remplacer par les valeurs) du gain $G(\omega)=\frac{V_{in}}{V_{out}}$ ? + \item Tracer le diagramme de Bode (amplitude) de $G(\omega)$ pour les valeurs données des éléments. + \item Tracer le diagramme de Bode (phase) de $G(\omega)$ pour les valeurs données des éléments. Quelle est la fonction de ce circuit ? + \item Quelle est l'expression analytique de l'impédance d'entrée $Z_{in}$ si l'on considère que la charge est une impédance de $10k\Omega$ (limiter les calculs : il n'est pas nécessaire de calculer les matrices du quadripôles pour répondre). + \item Quelle est l'expression analytique de l'impédance de sortie $Z_{out}$ si l'on considère que la source en entrée a une résistance interne égale à $75\Omega$ (limiter les calculs : il n'est pas nécessaire de calculer les matrices du quadripôles pour répondre). +\end{enumerate} +\begin{solution} +\begin{enumerate} + \item Comme l'amplificateur opérationnel est en rétroaction négative et est supposé idéal ($i^-=i^+=0[A]$) nous obtenons: + \begin{equation*} + \begin{dcases} + V^+ = V^-\\ + \frac{V_{in}-V^-}{R_2} = \frac{V^- - V_{out}}{R_3}\\ + (V_{in} -V^+)j\omega C_1 = \frac{V^+}{R_1} + \end{dcases} + \end{equation*} + \item En prenant la dernière équation: + \begin{equation*} + V^+(\frac{1}{R_1}+j\omega C_1)= V_{in}j\omega C_1 \Leftrightarrow V^+ = \frac{R_1j\omega C_1}{1+j\omega C_1R_1} + \end{equation*} + En prenant la deuxième équation: + \begin{align*} + -\frac{V_{out}}{R_3} &= \frac{V_{in}}{R_2}-V^-(\frac{1}{R_2}+\frac{1}{R_3})\\ + -\frac{V_{out}}{R_3} &= V_{in} (\frac{1}{R_2}-\frac{R_1C_1j\omega}{1+R_1C_1j\omega}\frac{R_2+R_3}{R_2R_3})\\ + -\frac{V_{out}}{R_3} &= V_{in} (\frac{R_3+j\omega C_1R_1R_3 -(R_2+R_3)R_1j\omega C_1}{R_2R_3(1+j\omega C_1R1)})\\ + -\frac{V_{out}}{R_3} &=V_{in}\frac{R_3-j\omega C_1R_1R_2}{R_2R_3(1+j\omega C_1R_1)}\\ + \Rightarrow G(\omega) &= -\frac{R_3}{R_2}(\frac{1-j\omega C_1\frac{R_1R_2}{R_3}}{1+j\omega C_1R_1}) + \end{align*} + \item La fonction de transfert du gain $g_f$ peut être réécrite sous la forme générale : + \begin{equation*} + H(j\omega) = K \frac{1-j\frac{\omega}{\omega_0}}{1+j\frac{\omega}{\omega_1}}~~ + \mbox{avec}~~ + \left\{\begin{matrix*}[l] + K= -\frac{R_3}{R_2} =-1\\ + \omega_0 = \frac{R_3}{R_1R_2C_1}= 10^{4}[rad/s]\\ + \omega_1 = \frac{1}{R_1C_1}= 10^4[rad/s] \\ + \end{matrix*}\right. + \end{equation*} + \begin{center} + \begin{tikzpicture}[ + gnuplot def/.append style={prefix={}}, + ] + % Grid Style + \tikzset{ + semilog lines/.style={black}, + semilog lines 2/.style={gray,dotted}, + semilog half lines/.style={gray, dotted}, + semilog label x/.style={below,font=\tiny}, + semilog label y/.style={above,font=\tiny} } + % Magnitude Plot + \begin{scope}[xscale=7/5, yscale=3/50] + \UnitedB + \semilog{0}{8}{-30}{30} + %Asymp + \BodeGraph[green,samples=1000]{0:5.5}{\POAmpAsymp{1}{0.0001}} + \BodeGraph[blue,samples=1000]{0:5.5}{-\POAmpAsymp{-1}{-0.0001}} + %Real + \BodeGraph[red,samples=1000]{0:8}{\POAmp{1}{0.0001}-\POAmp{-1}{-0.0001}} + \end{scope} + % Phase plot + \begin{scope}[yshift=-7cm,xscale=7/5,yscale=3/180] + \UniteDegre + \OrdBode{30} + \semilog{0}{8}{-90}{270} + %Asymp + \BodeGraph[green,samples=1000]{0:8}{\POArgAsymp{1}{0.0001}} + \BodeGraph[blue,samples=1000]{0:8}{-\POArgAsymp{1}{-0.0001}-\IntArg{-100000}-\IntArg{-100000}} + %Real + \BodeGraph[red,samples=1000]{0:8}{\POArg{1}{0.0001}-\POArg{-1}{-0.0001}-\IntArg{-100000}-\IntArg{-100000}} + \end{scope} + \end{tikzpicture} + \end{center} + \item C'est un filtre passe-tout qui déphase de $180^\circ$ avant la fréquence de coupure. + \item Pour déterminer l'impédance d'entrée $Z_{in}$, nous allons utiliser la représentation Z du quadripôle. + \begin{equation*} + \begin{bmatrix} + V_i \\ + V_o + \end{bmatrix} = + \begin{bmatrix} + z_i & z_r \\ + z_f & z_o + \end{bmatrix} + \begin{bmatrix} + i_i \\ + i_o + \end{bmatrix} + \end{equation*} + En annulant le courant d'entrée $i_i$, le circuit devient : + \begin{center} + \begin{circuitikz} + \draw + (5,3) node[op amp] (opamp) {} + (opamp.-) --(3.8,4.5) + to[R,l^=$R_3$](7.5,4.5)--(7.5,3) + (opamp.out) to [short,-o](8,3) + (7.5,3) to[short, i_<=$i_o$](8,3) + (1,4.5) to [R,l^=$R_2$,o-](4.5,4.5) + (1.5,4.5) --(1.5,2.5) + to[C,l_=$C_1$] (opamp.+) + (3.5,2.5) to [R,l^=$R_1$](3.5,0)node[ground]{} + (3.5,0) to[short,-o](8,0) + to [open,v>=$V_{out}$](8,3) + (3.5,0)to[short,-o](1,0) + to [open,v^>=$V_{in}$](1,4.5); + \end{circuitikz} + \end{center} + Comme nous avons un amplificateur opérationnel que l'on considère comme idéal, son impédance d'entrée vaut 0$\Omega$. Le courant $i_0$ passe donc entièrement dans l'amplificateur opérationnel et aucun courant ne passe dans le reste du circuit. Dès lors tout le circuit est à 0V. Nous obtenons donc : + \begin{equation*} + \left \{ + \begin{array}{rcl} + V_i &=& 0\cdot i_o \\ + V_o &=& 0 \cdot i_o\\ + \end{array} + \right. + \Leftrightarrow + \left \{ + \begin{array}{rcl} + z_r &=& 0 [\Omega]\\ + z_o &=& 0 [\Omega]\\ + \end{array} + \right. + \end{equation*} + On remarque que les termes de droite de la matrice permettent de simplifier le reste des calculs. Calculons $Z_{in}$ : + \begin{align*} + Z_{in} &= Z_i (1+\frac{z_r}{z_i}A_{if,o}) ~~\mbox{avec}~~A_{if,o} = -\frac{z_f}{z_o+z_L}=-\frac{z_f}{z_L}\\ + &=z_i(1+\frac{0}{z_i}\frac{-z_f}{z_L})\\ + &=z_i + \end{align*} + L'impédance d'entrée ne dépend que de $z_i$. Pour calculer cela nous annulons le courant d'entrée $i_o$ et le circuit devient : + \begin{center} + \begin{circuitikz} + \draw + (5,3) node[op amp] (opamp) {} + (opamp.-) --(3.8,4.5) + to[R,l^=$R_3$](7.5,4.5)--(7.5,3) + (opamp.out) to [short,-o](8,3) + (7.5,3) to[short](8,3) + (1,4.5) to [short,i=$i_i$,o-](1.5,4.5) to[R,l^=$R_2$,i>_=$i_2$,*-](4.5,4.5) + (1.5,4.5) to[short,i>=$i_1$](1.5,2.5) + to[C,l_=$C_1$] (opamp.+) + (3.5,2.5) to [R,l^=$R_1$](3.5,0)node[ground]{} + (3.5,0) to[short,-o](8,0) + to [open,v>=$V_{out}$](8,3) + (3.5,0)to[short,-o](1,0) + to [open,v^>=$V_{in}$](1,4.5); + \end{circuitikz} + \end{center} + Nous pouvons effectuer une loi des mailles sur la maille tout à gauche, nous obtenons : + \begin{equation*} + V_{in} =i_1(R_1+\frac{1}{j\omega C_1}\Leftrightarrow i_1 = \frac{j\omega C_1}{1+R_1C_1j\omega}V_{in} + \end{equation*} + Nous pouvons exprimer le courant $i_2$ avec : + \begin{align*} + i_2 &= \frac{V_{in}-V^-}{R_2}~~\mbox{avec}~~V^-=V^+=R_1i_1\\ + &=\frac{V_{in}-R_1i_1}{R_2}\\ + &=\frac{V_{in}}{R_2}(1-\frac{j\omega R_1C_1}{1+R_1C_1j\omega})\\ + &=\frac{V_{in}}{R_2}(\frac{1}{1+R_1C_1j\omega}) + \end{align*} + Nous avons exprimé $i_1$ et $i_2$ au moyen de $V_{in}$. Nous allons maintenant lier $V_{in}$ à $i_i$ au moyen de la relation : + \begin{align*} + i_i &= i_1 +i_2\\ + &=V_{in}(\frac{1+R_2C_1j\omega}{R_2(1+R_1C_1j\omega)})\\ + \Leftrightarrow V_{in} &= \frac{R_2(1+R_1C_1j\omega)}{1+R_2C_1j\omega} i_i + \end{align*} + \begin{equation*} + \Rightarrow Z_{in}=z_i = \frac{R_2(1+R_1C_1j\omega)}{1+R_2C_1j\omega} [\Omega] + \end{equation*} + \item Comme nous avons un amplificateur opérationnel que l’on considère comme idéal, nous pouvons supposer que $Z_{out}=$ 0 $\Omega$. Si cela ne vous apparaît pas vous pouvez toujours calculer l'impédance de sortie au moyen de la table des caractéristiques externes: + \begin{align*} + Z_{out} &= z_o-z_r\frac{z_f}{z_G+z_i}~~ \mbox{avec}~~ z_G = 75[\Omega]\\ + &= 0-0\cdot \frac{z_f}{z_G+z_i}\\ + &=0 + \end{align*} + \begin{equation*} + \Rightarrow Z_{out} = 0 [\Omega] + \end{equation*} +\end{enumerate} +\end{solution} +\newpage +\section{Question Craeye : transitoire} +On considère le circuit suivant où $L=2mH$, $C=0.25nF$ et $R=6k\Omega$ et où l'interrupteur passe de A à B au temps $t=0$. +\begin{center} + \begin{circuitikz} + \draw (0,2) node[ground]{} to[american voltage source,l=$1V$] (0,4) to [R,l=$R$] (0,6) to[short,-o] (1,6) coordinate (B); + \draw (2,2) node[ground]{}to[american voltage source,l=$3V$] (2,3) to[R,l=$R$,-o] (2,5) coordinate (A) + (1.4,5.4) to[short,o-] (2,6) + (8,5.5) node[op amp] (opamp) {} + (2,6) to [L,l_=$L$](opamp.-) + (opamp.-) to[short,*-] ++(0,2.5) + to[R,l^=$R$]++(3,0) to [short]++(0,-3) + (opamp.+)to[R,l^=$R$](6.8,3)node[ground]{} + (6.8,7)to[C,l_=$C$](9.8,7) + (opamp.out)to[short,-*](9.8,5.5) to [short,-o] (11,5.5)node[above]{$V_o$}{}; + \draw [->,>=stealth] ($ (A) + (-0.2,0) $) to [bend left] node[pos=0.5,below left] {$t=0$} ($ (B) + (0,-0.2) $); + \draw (A) node[right]{$A$}; + \draw (B) node[above]{$B$}; + \end{circuitikz} +\end{center} +\begin{enumerate} + \item Dans le domaine de Laplace, donner les relations qui lient la tension de source, la tension $V_o$ et le courant dans l'inductance. + \item Donner l'expression de la tension $V_o$ en fonction du temps pour $t>0$ +\end{enumerate} +\begin{solution} +\begin{enumerate} + \item En $t<0$, on peut réécrire le circuit comme : +\begin{center} + \begin{circuitikz} + \draw (2,3) node[ground]{}to[american voltage source,l=$3V$] (2,6) + (8,5.5) node[op amp] (opamp) {} + (2,6) to[R,l_=$R$,v^=$ $] ++(3,0) to [short,i>_=$I_L(0^-)$](opamp.-) + (opamp.-) to[short,*-] ++(0,2.5) + to[R,l^=$R$,v_=$ $]++(3,0) to [short]++(0,-3) + (opamp.+)to[R,l^=$R$](6.8,3)node[ground]{} + (6.8,7)to[open,v^=$V_C(0^-)$,*-*](9.8,7) + (opamp.out)to[short,-*](9.8,5.5) to [short,-o] (11,5.5)node[above]{$V_o$}{}; + \end{circuitikz} +\end{center} +Étant donner que l'amplificateur opérationnel est considéré comme parfait ($i^+=i^-=0A$) et est en rétroaction négative ($V^+=V^-=0V$) car la masse ne délivre aucune tension et aucun courant, nous obtenons: +\begin{equation*} + \left \{ +\begin{array}{rcl} +I_L(0^-) &=& \frac{3}{R}=5\cdot 10^{-4} [A]\\ +V_{C}(0^-) &=& 0-V_o = R I_L(0^-)=3[V]\\ +\end{array} +\right. +\end{equation*} +En $t>0$, on peut réécrire le circuit comme : +\begin{center} + \begin{circuitikz} + \draw (2,3) node[ground]{}to[american voltage source,l=$1V$] (2,6) + (8,5.5) node[op amp] (opamp) {} + (2,6) to[R,l_=$R$,v^=$V_R(s)$] ++(3,0) to [L,l_=$L$,i>_=$I_1(s)$,v^=$V_L(s)$](opamp.-) + (opamp.-) to[short,*-] ++(0,3.5) + to[R,l_=$R$,v^=$V_{R^{'}}(s)$,i>_=$I_2(s)$]++(3,0) to [short]++(0,-4) + (opamp.+)to[R,l^=$R$](6.8,3)node[ground]{} + (6.8,7)to[C,l_=$C$,v^=$V_C(s)$](9.8,7) + (opamp.out)to[short,-*](9.8,5.5) to [short,-o] (11,5.5)node[above]{$V_o(s)$}{}; + \end{circuitikz} +\end{center} +Nous pouvons effectuer une loi des mailles sur la maille de gauche: +\begin{align*} + \frac{1}{s} &= RI_1(s)+V_L(s)~~\mbox{avec}~~V_L(s) = LsI_1(s)-LI_L(0^-)\\ + \Leftrightarrow \frac{1+sLI_L(0^-)}{s} &= I_1(s)(R+sL)\\ + \Rightarrow I_1(s) &= \frac{1+sLI_L(0^-)}{s(R+sL)} +\end{align*} +Nous pouvons effectuer une loi des mailles sur la maille supérieure : +\begin{align*} + V_C(s) &= V_{R^{'}}(s) ~~\mbox{avec}~~V_C(s) =-V_o(s)\\ + \Leftrightarrow V_C(s) &= -RI_2(s) \\ + \Rightarrow I_2(s) &= -\frac{V_o(s)}{R} +\end{align*} +Le courant dans la capacité peut s'écrire : +\begin{align*} + I_1(s) -I_2(s) &= sCV_C(s) -CV_C(0^-) ~~\mbox{avec}~~V_C(s) =-V_o(s)\\ + \Leftrightarrow \frac{1+sLI_L(0^-)}{s(R+sL)}+\frac{V_o(s)}{R} &= -scV_o(s) -cV_c(0^-)\\ + \Leftrightarrow -V_o(s) (\frac{1}{R}+sC) &= \frac{1+sLI_L(0^-)+sRCV_C(0^-)+s^2LCV_C(0^-)}{s(R+sL)}\\ + \Leftrightarrow V_o(s) &= -\frac{R+sR(LI_L(0^-)+RCV_C(0^-))+s^2RLCV_C(0^-)}{s(R+sL)(1+sRC)} +\end{align*} +\begin{equation*} + \Rightarrow V_o(s) = -\frac{\frac{1}{LC}+\frac{s}{LC}(LI_L(0^-)+RCV_C(0^-))+s^2V_C(0^-)}{s(s+\frac{R}{L})(s+\frac{1}{RC})} +\end{equation*} +\item En substituant les paramètres par leurs valeurs nous obtenons : +\begin{equation*} + V_o(s) = -\frac{2\cdot 10^{12}+ 11\cdot 10^{6}s+3s^2}{s(s+3\cdot 10^{6})(s+\frac{2\cdot10^6 }{3})} +\end{equation*} +En effectuant la décomposition en fraction simple nous avons : +\begin{equation*} + V_C(s) = \frac{A}{s} + \frac{B}{s+3\cdot10^6}+\frac{C}{s+\frac{2\cdot10^6 }{3}} \Leftrightarrow +\begin{dcases} + A + B + C = -3\\ + (3+\frac{2}{3}) 10^6 A+ \frac{2}{3}10^6B+ 3\cdot 10^6C = -11\cdot 10^6\\ + 2\cdot 10^{12}A = -2 \cdot 10^{12}\\ +\end{dcases} +\Leftrightarrow +\begin{dcases} + A= -1\\ + B= \frac{4}{7}\\ + C=-\frac{18}{7}\\ +\end{dcases} +\end{equation*} +Nous obtenons donc dans le domaine de Laplace : +\begin{equation*} + V_o(s) = -\frac{1}{s} +\frac{4}{7} \cdot \frac{1}{s+3\cdot10^6}- \frac{18}{7}\cdot \frac{1}{s+\frac{2\cdot10^6}{3}} +\end{equation*} +Cela correspond dans le domaine temporel à : +\begin{equation*} + V_o(t) = \left[-1+\frac{4}{7}\cdot e^{-3\cdot 10^6t}-\frac{18}{7}\cdot e^{-\frac{2}{3}\cdot 10^6t}\right]u(t) +\end{equation*} +Le graphe ressemble à +\begin{center} + \begin{tikzpicture} + \begin{axis}[enlargelimits=true,grid=major,ylabel=$V_o(t)$,xlabel=$t(s)$] + \addplot [blue,domain=0:0.00001,samples=200]{-1+(4/7)*e^(-3*x*10^6)-(18/7)*e^(-(2/3)*x*10^6}; + \end{axis} + \end{tikzpicture} +\end{center} +\end{enumerate} +\end{solution} +\newpage +\section{Question Dehez : triphasé} +On considère le circuit suivant, alimenté par une source de tension triphasée dont la fréquence est de 50Hz. +\begin{center} + \begin{circuitikz}[scale=0.9] + \coordinate (s1) at (0,0); + \coordinate (s2) at ($ (s1) + ({6*cos(60)},{-6*sin(60)}) $); + \coordinate (s3) at ($ (s1) + ({-6*cos(60)},{-6*sin(60)}) $); + \coordinate (c3) at ($ (s1) + (0,{-sqrt(9)}) $); + + %sources + \draw (c3) to[V,l=$220\angle 0^\circ \,V$,i=$I_1$,*-] (s1); + \draw (c3) to [V,l=$220\angle 120^\circ \,V$,i_=$I_2$] (s2); + \draw (c3) to [V,l_=$220\angle 240^\circ \,V$,i=$I_3$] (s3); + + \draw (s1) to [R,l=$1\Omega$] ++(9,0) coordinate (l11); + \draw (s3) to[short] ++(0,-2) coordinate (p1); + + \coordinate (l12) at ($ (l11) + ({6*cos(60)},{-6*sin(60)}) $); + \coordinate (l13) at ($ (l11) + ({-6*cos(60)},{-6*sin(60)}) $); + \coordinate (c1) at ($ (l11) + (0,{-sqrt(10)}) $); + + %inductances + \draw (l11) to[L,l_=$1j\Omega$] (9,-1.5) coordinate (l111); + \draw (l13) to[L,l_=$1j\Omega$] (7,-4.5) coordinate (l133) ; + \draw (l12)to[L,l_=$1j\Omega$] (11,-4.5) coordinate (l122) ; + + %résistances + \draw (l13) to[R,l=$1\Omega$] (s2); + \draw (l12) to[short] ++(0,-2) to[R,l=$1\Omega$] (p1); + + %Résistances triangle + \draw (l111) to [R,l^=$60\Omega$](l122); + \draw (l122) to [R,l^=$60\Omega$](l133); + \draw (l111) to [R,l_=$60\Omega$](l133); + + %bullets inductances + \draw (l11) node[below left]{$\bullet$}; + \draw (l12) node[above]{$\bullet$}; + \draw (l13) node[below right]{$\bullet$}; + + %inductances mutuelles + \draw [<->,>=stealth] ($ (l11) + (0.3,-0.2) $) to [bend left] node[pos=0.5,above right] {$j\Omega$} ($ (l12) + (0.2,0.2) $); + \draw [<->,>=stealth] ($ (l12) + (-0.2,-0.2) $) to [bend left] node[pos=0.5,above left] {$j\Omega$} ($ (l13) + (0,-0.4) $); + \draw [<->,>=stealth] ($ (l13) + (-0.2,0.2) $) to [bend left] node[pos=0.5,above] {$j\Omega$} ($ (l11) + (-0.4,-0.2) $); + + \end{circuitikz} +\end{center} +\begin{enumerate} + \item Calculer le facteur de dispersion entre les inductances couplées magnétiquement. + \item Calculer le module et la phase du courant $I_1$. + \item Calculer la valeur de la puissance active fournie par la source triphasée. + \item Calculer la valeur de la puissance réactive fournie par la source triphasée. + \item Calculer la valeur la valeur de la capacité C et de ces condensateurs qui permettra d'annuler la puissance réactive fournie par la source.\footnote{Ce n'est pas un noeud là où la capacité coupe le câble avec le courant $I_2$} + +\begin{center} + \begin{circuitikz}[scale=0.9] + \coordinate (s1) at (0,0); + \coordinate (s2) at ($ (s1) + ({6*cos(60)},{-6*sin(60)}) $); + \coordinate (s3) at ($ (s1) + ({-6*cos(60)},{-6*sin(60)}) $); + \coordinate (c3) at ($ (s1) + (0,{-sqrt(9)}) $); + + %sources + \draw (c3) to[V,l=$220\angle 0^\circ \,V$,i=$I_1$,*-] (s1); + \draw (c3) to [V,l=$220\angle 120^\circ \,V$,i_=$I_2$] (s2); + \draw (c3) to [V,l_=$220\angle 240^\circ \,V$,i=$I_3$] (s3); + + \draw (s1) to [R,l=$1\Omega$] ++(6,0) coordinate (c11) to [short] ++(5,0)coordinate (l11); + \draw (s3) to[short] ++(0,-2) coordinate (p1); + + \coordinate (l12) at ($ (l11) + ({6*cos(60)},{-6*sin(60)}) $); + \coordinate (l13) at ($ (l11) + ({-6*cos(60)},{-6*sin(60)}) $); + \coordinate (c1) at ($ (l11) + (0,{-sqrt(10)}) $); + + %inductances + \draw (l11) to[L,l_=$1j\Omega$] (11,-1.5) coordinate (l111); + \draw (l13) to[L,l_=$1j\Omega$] (9,-4.5) coordinate (l133) ; + \draw (l12)to[L,l_=$1j\Omega$] (13,-4.5) coordinate (l122) ; + + %résistances + \draw (l13) to [short] ++(-3,0) coordinate(c22) to[R,l=$1\Omega$] (s2); + \draw (l12) to[short] ++(0,-2) to[short] ++(-6,0) coordinate (c33) to[R,l=$1\Omega$] (p1); + + %Résistances triangle + \draw (l111) to [R,l^=$60\Omega$](l122); + \draw (l122) to [R,l^=$60\Omega$](l133); + \draw (l111) to [R,l_=$60\Omega$](l133); + + %bullets inductances + \draw (l11) node[below left]{$\bullet$}; + \draw (l12) node[above]{$\bullet$}; + \draw (l13) node[below right]{$\bullet$}; + + %inductances mutuelles + \draw [<->,>=stealth] ($ (l11) + (0.3,-0.2) $) to [bend left] node[pos=0.5,above right] {$j\Omega$} ($ (l12) + (0.2,0.2) $); + \draw [<->,>=stealth] ($ (l12) + (-0.2,-0.2) $) to [bend left] node[pos=0.5,above left] {$j\Omega$} ($ (l13) + (0,-0.4) $); + \draw [<->,>=stealth] ($ (l13) + (-0.2,0.2) $) to [bend left] node[pos=0.5,above] {$j\Omega$} ($ (l11) + (-0.4,-0.2) $); + %Capacités + \draw (c11) to [C,l=$C$,*-*]++(0,-3)coordinate (c00); + \draw (c00) to [C,l=$C$,-*] (c22); + \draw (c33) to [C,l=$C$,*-] ++(-1,2) coordinate (c000); + \draw (c000) to [short] (c00); + \end{circuitikz} +\end{center} +\end{enumerate} +\begin{solution} +\begin{enumerate} + \item Le facteur de dispersion s’écrit comme + \begin{equation*} + \sigma = 1 -\frac{M^2}{L_1L_2} = 1 - \frac{(j)^2}{2j*2j} = 0.75 + \end{equation*} + Nous obtenons bien un facteur de dispersion $\in$ [0,1]. + \item Nous allons commencer par changer la charge triangle formée par les résistance en une charge étoile en sachant que $R_{étoile}$ = $\frac{R_{triangle}}{3}$. + On obtient le schéma simplifié ci-dessous + +\begin{center} + \begin{circuitikz}[scale=0.9] + \coordinate (s1) at (0,0); + \coordinate (s2) at ($ (s1) + ({6*cos(60)},{-6*sin(60)}) $); + \coordinate (s3) at ($ (s1) + ({-6*cos(60)},{-6*sin(60)}) $); + \coordinate (c3) at ($ (s1) + (0,{-sqrt(9)}) $); + + %sources + \draw (c3) to[V,l=$220\angle 0^\circ \,V$,i=$I_1$,*-] (s1); + \draw (c3) to [V,l=$220\angle 120^\circ \,V$,i=$I_2$] (s2); + \draw (c3) to [V,l=$220\angle 240^\circ \,V$,i=$I_3$] (s3); + + \draw (s1) to [R,l=$1\Omega$] ++(9,0) coordinate (l11); + \draw (s3) to[short] ++(0,-2) coordinate (p1); + + \coordinate (l12) at ($ (l11) + ({6*cos(60)},{-6*sin(60)}) $); + \coordinate (l13) at ($ (l11) + ({-6*cos(60)},{-6*sin(60)}) $); + \coordinate (c1) at ($ (l11) + (0,{-sqrt(10)}) $); + + %inductances + \draw (l11) to[L,l_=$1j\Omega$] (9,-1.5); + \draw (9,-1.5) to[R,l=$20\Omega$,-*] (c1); + \draw (c1) to[R,l=$20\Omega$] (7,-4.5); + \draw (7,-4.5) to[L,l_=$1j\Omega$] (l13); + \draw (c1) to[R,l=$20\Omega$] (10.5,-4.3); + \draw (10.5,-4.3) to[L,l_=$1j\Omega$] (l12); + + %résistances + \draw (l13) to[R,l=$1\Omega$] (s2); + \draw (l12) to[short] ++(0,-2) to[R,l=$1\Omega$] (p1); + + %bullets inductances + \draw (l11) node[below left]{$\bullet$}; + \draw (l12) node[above]{$\bullet$}; + \draw (l13) node[below right]{$\bullet$}; + + %inductances mutuelles + \draw [<->,>=stealth] ($ (l11) + (0.3,-0.2) $) to [bend left] node[pos=0.5,above right] {$j\Omega$} ($ (l12) + (0.2,0.2) $); + \draw [<->,>=stealth] ($ (l12) + (-0.2,-0.2) $) to [bend left] node[pos=0.5,above left] {$j\Omega$} ($ (l13) + (0,-0.4) $); + \draw [<->,>=stealth] ($ (l13) + (-0.2,0.2) $) to [bend left] node[pos=0.5,above] {$j\Omega$} ($ (l11) + (-0.4,-0.2) $); + + \end{circuitikz} +\end{center} + + Nous allons ensuite enlever l'inductance mutuelle entre chaque pair d'inductance en sachant que la somme des courants est nulle. On a donc + \begin{align*} + 2jI_1 + 1jI_2 + 1jI_3 &= 2jI_1 + 1j(I_2 + I_3) \\ + &= 2jI_1 + 1j(-I_1) \\ + &= 1jI_1 + \end{align*} + + + +On peut dès lors travailler sur l'équivalent monophasé de ce circuit et calculer le courant $I_1$. + +\begin{center} + \begin{circuitikz}[scale=0.9] + \coordinate (s1) at (0,0); + \coordinate (c3) at ($ (s1) + (0,{-sqrt(10)}) $); + + %sources + \draw (c3) to[V,l=$220\angle 0^\circ \,V$,i=$I_1$] (s1); + + \draw (s1) to [R,l=$1\Omega$] ++(9,0) coordinate (l11); + \coordinate (c1) at ($ (l11) + (0,{-sqrt(10)}) $); + + %inductances + \draw (l11) to[L,l_=$1j\Omega$] (9,-1.5); + \draw (9,-1.5) to[R,l=$20\Omega$] (c1); + + %bullets inductances + \draw (l11) node[below left]{$\bullet$}; + \draw (c1) to (c3); + + \end{circuitikz} +\end{center} +\begin{align*} + 220\angle 0^\circ \ &= (21 + j)I_1\\ + \Leftrightarrow I_1 &= 10.4643\angle -2.7263^\circ [A] +\end{align*} +\item La puissance active de la source est la contribution des puissances actives des 3 circuits équivalents monophasés. +\begin{align*} + P = 3\cdot \Re(V \cdot I_1^*) = 3\cdot \Re( 220\angle 0^\circ \ \cdot 10,4643\angle 2,7263^\circ \ ) = 6898.64 [W] +\end{align*} +\item La puissance réactive de la source est la contribution des puissances réactives des 3 circuits équivalents monophasés. +\begin{align*} + P = 3 \cdot \Im(V \cdot I_1^*) = 3 \cdot \Im( 220\angle 0^\circ \ \cdot 10.4643\angle 2.7263^\circ \ ) = 328.506 [VAR] +\end{align*} +\item L'équivalent monophasé peut être réécris comme suit +\begin{center} + \begin{circuitikz}[scale=0.9] + \coordinate (s1) at (0,0); + \coordinate (c3) at ($ (s1) + (0,{-sqrt(10)}) $); + + \draw (6,0) to [C,l_=$\frac{1}{j\omega C}$] (6,-{sqrt(10)}); + + %sources + \draw (c3) to[V,l=$220\angle 0^\circ \,V$,i=$I_1$] (s1); + + \draw (s1) to [R,l=$1\Omega$] ++(9,0) coordinate (l11); + \coordinate (c1) at ($ (l11) + (0,{-sqrt(10)}) $); + + %inductances + \draw (l11) to[L,l_=$1j\Omega$] (9,-1.5); + \draw (9,-1.5) to[R,l=$20\Omega$] (c1); + + %bullets inductances + \draw (l11) node[below left]{$\bullet$}; + \draw (c1) to (c3); + + \end{circuitikz} +\end{center} +ou encore, +\begin{center} + \begin{circuitikz}[scale=0.9] + \coordinate (s1) at (0,0); + \coordinate (c3) at ($ (s1) + (0,{-sqrt(10)}) $); + %sources + \draw (c3) to[V,l=$220\angle 0^\circ \,V$,i=$I_1$] (s1); + \draw (s1) to [R,l=$1\Omega$] ++(9,0) coordinate (l11); + \coordinate (c1) at ($ (l11) + (0,{-sqrt(10)}) $); + %inductances + \draw (l11) to[european resistor,l=Z] (c1); + \draw (c1) to (c3); + \end{circuitikz} +\end{center} +\begin{align*} + Z &= (20 + j) || \frac{1}{j\omega C}\\ + &= \frac{20 + j}{1+20j\omega C - \omega C} [\Omega] +\end{align*} +Pour que la puissance réactive soit nulle, il faut que la partie imaginaire de Z soit nulle. +\begin{align*} + \Rightarrow Z &= \frac{20 + j}{1-\omega C+20j\omega C} \cdot \frac{(1-\omega C) - 20j\omega C}{(1-\omega C)-20j\omega C}\\ + &= \frac{20+j-20\omega C-j\omega C-400j\omega C+20\omega C}{(1-\omega C)^2+400\omega ^2C^2}\\ + &= \frac{20+j(1-401\omega C)}{(1-\omega C)^2+400\omega ^2C^2}\\ + \Rightarrow \Im(Z) &= 0\\ + \Leftrightarrow 0 &= (1-401\omega C)\\ + \Leftrightarrow C &= \frac{1}{401\omega } + = \frac{1}{401 \cdot 2 \cdot \pi \cdot 50} + = 7.938 \mu F +\end{align*} + +\end{enumerate} +\end{solution} +\end{document} \ No newline at end of file diff --git a/src/q4/circmes-ELEC1370/exam/2020/Juin/Juin.mk b/src/q4/circmes-ELEC1370/exam/2020/Juin/Juin.mk new file mode 100644 index 000000000..6584a9746 --- /dev/null +++ b/src/q4/circmes-ELEC1370/exam/2020/Juin/Juin.mk @@ -0,0 +1,2 @@ +MONTH=Juin +include ../../2020.mk